1. Trang chủ
  2. » Giáo án - Bài giảng

50 đề chuyên toán 2018 2019

0 21 0

Đang tải... (xem toàn văn)

THÔNG TIN TÀI LIỆU

SỞ GIÁO DỤC VÀ ĐÀO TẠO NGHỆ AN KỲ THI TUYỂN SINH LỚP 10 THPT NĂM HỌC 2018-2019 Môn thi: TOÁN CHUYÊN Ngày thi: 03/06/2018 Câu a) Giải phương trình : x    x  x2  5x 1  xy  y  x b) Giải hệ phương trình:  2   y  y   x  8x Câu a) Tìm số nguyên x; y; z cho x2  y  z   xy  3x  4z b) Cho hai số nguyên dương m, n thỏa mãn m  n  ước nguyên tố  m2  n2   CMR m.n số phương Câu Cho a, b, c thực dương thỏa mãn abc  Chứng minh rằng: a  a3  ab   b4  b3  bc   c  c3  ac   Câu Cho tam giác ABC vuông A  AB  AC  nội tiếp đường tròn (O) đường cao AH Gọi D điểm đối xứng với A qua BC Gọi K hình chiếu vng góc A lên BD Qua H kẻ đường thẳng song song với BD cắt AK I Đường thẳng BI cắt đường tròn (O) N (N khác B) a) Chứng minh AN.BI  DH BK b) Tiếp tuyến (O) D cắt đường thẳng BC P Chứng minh đường thẳng BC tiếp xúc với đường tròn ngoại tiếp tam giác ANP c) Tiếp tuyến (O) C cắt DP M Đường tròn qua D tiếp xúc với CM M cắt OD Q (Q khác D) Chứng minh đường thẳng qua Q vng góc với BM ln qua điểm cố định BC cố định A di động đường tròn (O) Câu Để phục vụ cho lễ khai mạc World Cung 2018, ban tổ chức giải đấu chuẩn bị 25000 bóng, bóng đánh số từ đến 25000 Người ta dùng màu: Đỏ, Da cam, Vàng, Lục, Lam, Chàm, Tím để sơn bóng (mỗi sơn màu) Chứng minh 25000 bóng nói tồn bóng màu đánh số a, b, c mà a chia hết cho b, b chia hết cho c abc  17 ĐÁP ÁN Câu a) Giải phương trình : x    x  x2  5x 1 Điều kiện xác định:  x  Ta có x2  5x      x  1    x 1  x 3 x 3  0 x  1  x 1 1     x  3  x    0 x  1  x 1     x  1 x  3  1    1 0 1  Do  x    2x 1  0 x  1 x  1  x 1 2  x    x    x  3(tm) Vậy phương trình có nghiệm x  b) Hệ cho tương đương với 2 xy  y  x  xy  y  x    2 2  y  y   x  x  x  y  y   xy  y  x  x   xy  y  x  xy  y  x    x  y    x  y     x  y   x  y  1    2  13 5  13 ;y  x  y  x  3    3 x  x    2  13 5  13 ;y  x  3     5  22 26  22  ;y x  x  y   3    3 x  10 x  21   5  22 26  22  ;y x   3  Vậy hệ phương trình cho có nghiệm Câu a) Do x, y, z số nguyên nên x  y  z    xy  y  z  x  y  z   xy  y  z  2   y    x  y     1   z      2   x  y  x   y    1    y  2  z  z     Vậy x  1; y  z  số nguyên cần tìm b) Giả sử m  n Theo ta có: m  n    m  n  1 m  n  1  m  n  1   m  n     m  n   1  m  n  1     2m  2n  m  2mn  n   m  n  1   m  n  m  n  1 Do m  n  số nguyên tố  m  n  ước m  n Mà m  n  m  n  vơ lý Vậy giả sử sai  m  n  m.n  m2 số phương Ta có điều phải chứng minh Câu Ta có:  a  1 a  a  1    a  2a  1 a  a  1   a  a3  a    a  a3   a  a  a  ab   ab  a   a  a  ab   ab  a  Chứng minh hồn tồn tương tự ta có: b4  b3  bc   1 ;  bc  b  c  c3  ac  ac  c  Như VT  1 1 1          ab  a  bc  b  ac  c   ab  a  bc  b  ac  c   (Áp dụng BĐT Bunhiacopxki cho số) Lại có 1 1 a ab               ab  a  bc  b  ac  c    ab  a  abc  ab  a a bc  abc  ab  a ab         ab  a  1  ab  a a  ab   Vậy ta có điều phải chứng minh Dấu “=” xảy a  b  c  Câu A I N P C H J M D O Q K B a) Chứng minh AN.BI  DH BK Ta có chắn cung AB nên BDA  BNA  IHA  BNA  INA Suy tứ giác ANHI nội tiếp (Tứ giác có hai đỉnh nhìn cạnh góc nhau) Do đó: AHN  AIN  BIK (hai góc nội tiếp chắn cung AN ) Ta có : AK  BD  AK  IH  AIH  900 Do tứ giác AHNI tứ giác nội tiếp (cmt)  AIH  ANH  1800  ANH  900  IBK  NAH  ANH BKI ( g.g )  BK BI BI    AN BI  DH BK AN AH DH b) Gọi O1 tâm đường tròn ngoại tiếp tam giác ANP, I trung điểm NP Vì A; D đối xứng qua BC nên PA tiếp tuyến (O) Ta có: PAN  PO1 N  PO1I1 (góc nội tiếp góc tâm chắn cung NP đường trịn  O1  ) Lại có: PAN  ADN (góc nội tiếp góc tạo tiếp tuyến dây cung chắn cung AN  O  )  PO1I1  ADN Hơn ANHI nội tiếp (cmt) nên ANH  AIH  900  NAH  NHP (cùng phụ với NHA ) Ta có : NAH  NIH  NBD  NDP  NHP  NDP  tứ giác PDNH nội tiếp nên NPH  NDA  NPH  PO1I1 Mặt khác : PO1I1  O1PI1  900  NPH  O1PI1  900  O1PH  900 Suy BC tiếp xúc với đường tròn ngoại tiếp tam giác ANP c) Gọi J trung điểm OM, G trung điểm OC, E giao điểm QG BM Dễ thấy MQ đường kính đường tròn qua D tiếp xúc với MC (Do MDQ  900 )  MQ  MC Mà MC  BC  MQ / / BC Do MQ / / BC  QMO  MOP (so le trong)  QOM  Tam giác QOM cân Q  QJ  OM (trung tuyến đồng thời đường cao)  BOM  GJQ (góc có cạnh tương ứng vng góc) Mặt khác OGJ OJG( g.g )  GJ OG  JQ OJ OGJ  OCM  OG OC OB   (OC  OB) OJ OM OM GJ OB   GJQ JQ OM BOM (c.g.c)  OMB  QJM  900 (hai góc nội tiếp chắn cung QM)  QE  EM  QE  BM Vậy đường thẳng qua Q vng góc với BM qua trung điểm G OC cố định Câu 5: Xét tập A  1; 2;3; .; 2500 tập B  1;3;3.2;3.22 ; ;3.213 Do 3.213  24576  250000  B  A Tập B có 15 phần tử Do bóng sơn màu mà có màu nên theo nguyên lý Dirichle tập B tồn bóng màu Giả sử bóng đánh số a  b  c a chia hết cho b, b chia hết cho c abc  18  17 Vậy ta có điều phải chứng minh SỞ GIÁO DỤC – ĐÀO TẠO TỈNH KHÁNH HÒA KỲ THI TUYỂN SINH LỚP 10 THPT NĂM HỌC 2018-2019 Mơn thi: TỐN CHUN Ngày thi : 03/06/2018 Câu a) Giải phương trình : x2  x   3x x  b) Có số tự nhiên có chữ số abc cho a, b, c độ dài cạnh tam giác cân Câu a) Chứng minh với số thực a, b, c ta ln có: a  b  c  a  b2  c2   ab  ac  bc  1 1 1  2  4;    2 x y xyz x y z b) Cho số x, y, z khác thỏa mãn : x  y  z  ; Tính Q   y 2017  z 2017  z 2019  x2019  x2021  y 2021  Câu Cho đường trịn (O) đường kính BC H điểm nằm đoạn thẳng BO (điểm H không trùng với hai điểm B O) Qua H vẽ đường thẳng vng góc với BC, cắt đường trịn (O) A D Gọi M giao điểm AC BD, qua M vẽ đường thẳng vuông góc với BC N a) Chứng minh tứ giác MNBA nội tiếp BO  OH b) Tính giá trị: P      AB  BH c) Từ B vẽ tiếp tuyến với đường tròn (O), cắt hai đường thẳng AC AN K E Chứng minh đường thẳng EC qua trung điểm I đoạn thẳng AH H di động đoạn thẳng BO Câu Cho a, b,c số thực dương thỏa mãn điều kiện a  b  c  abc Chứng minh  a2  b2    c2  a b Câu Để tiết kiệm chi phí vận hành đồng thời du khách tham quan hết 18 danh lam, thắng cảnh tỉnh K, công ty du lịch lữ hành KH thiết lập tuyến chiều sau: Nếu từ tỉnh A đến B từ B đến C khơng có tuyến từ A đến C Hỏi có cách thiết lập để hết 18 địa danh ? ĐÁP ÁN Câu a) Giải phương trình Điều kiện xác định x  1 x2  x   3x x   x   x  1  3x x   u  x Đặt   v  x  u  v u  2v Phương trình  u  3uv  2v    u  v  u  2v     TH 1: u  v  x2  x 1  1 x  x 1   x x  TH : u  2v x  x  x 1    x  2 x  4x   Vậy nghiệm phương trình cho : x  1 ;x  22 2 b) TH1:Tam giác a  b  c   có số lập TH2: Xét a  b  c Vì a  b  c (bất đẳng thức tam giác) nên: c  )a  b     khơng có giá trị c c  c   có cách chọn c +) a  b    c  c  )a  b     có cách chọn c c  c  )a  b     có cách chọn c c  c  10 )a  b     có cách chọn c c  c  12 )a  b     có cách chọn c c  c  14 )a  b     có cách chọn c c  c  18 )a  b     có cách chọn c c  c  18 )a  b     có cách chọn c c  Vậy trường hợp có 52 số thỏa mãn Do vai trò a, b, c nên : 52.3  156 (số) Vậy có tất  156  165 số thỏa mãn Câu a) VT   a  b  c    a  b  c  a  b  c   a  ab  ac  ab  b  bc  ac  bc  c  a  b  c   ab  bc  ca   VP b) Ta có: x yz   xyz xyz 1 1 2         xy yz xz xyz xy yz xz xyz 1 2 1 1  2 2 2    2 2 2 4 x y z xy yz xz x y z xyz x yz  1 1 1     4   2 x y z x y z Từ 1 1    x y z x yz   xy  yz  xz  x  y  z   xyz   x  y  x  z  y  z   x   y   y   z  z   x Hơn mũ Q lẻ nên có thừa số Vậy Q  Câu 3: M K A E I B N H C O D a) Ta có : BAC  900  BAM  900 (góc nội tiếp chắn nửa đường tròn) MNB  900 ( gt )  BAM  MNB  1800 Do tứ giác MNBA nội tiếp đường trịn đường kính MB (Tứ giác có tổng hai góc đối 1800 ) b) Do tam giác ABC vuông A nên áp dụng hệ thức lượng ta có: AB AB  BC BO OH BO.OH BO  BO  BH     BH AB AB BH  BO  BH BC BO  AB  BO       1 2 AB AB  AB  2  BO  OH  P  2 1    AB  BH Vậy giá trị P P  c) Ta dễ dàng có : Do tứ giác DBAC nội tiếp:   MBN  DBC  MBN  DAC  900  MBN  900  DAC  NMB  BCA (1)    DBC  DAC Tứ giác MNBA nội tiếp (cmt)  NMB  NAB (2) (hai góc nội tiếp chắn cung NB) Tam giác OAC cân O (OA = OC)  BCA  OAC (3) (hai góc đáy) Từ (1) (2) (3) suy NAB  OAC  OAC  BAO  NAB  BAO  BAC  NAO  NAO  900  OA  NA  NA tiếp tuyến đường trịn (O) A  EA  EB Theo tính chất hai tiếp tuyến cắt ta có:   EAB  EBA Trong tam giác vuông KAB ta chứng minh AE đường trung tuyến  EA  EB  EK  EAK cân E  BKA  EAK  AH  BC CI AI  AH / / BK Do theo định lý Ta lét ta có:  CE KE  BK  BC Ta có:  CI HI AI HI    CE BE KE BE Mà KE  EB  AI  HI HI / / EB  Từ suy I trung điểm AH Vậy ta có điều phải chứng minh Câu 4: Ta có: a  b  c  abc  a b c 1   1 ab ac bc Đặt x  ; y  ; z  bất đẳng thức cho trở thành : xy  xz  yz  1  a2  b2    c2  a b 1 1  1  c 1  a b c  1 z2 1 z   x2   y  1 z2 0 z    x2   y       1   1   x2 1  y2 1   x2  y  1 x 1 y 2 1 z2 0 z  z  z  x2  y 0 z Ta lại có: 1  xy    x  y   x2  y2   x2  y  x2 y    xz  yz    x  y   bdt         z   z  z( x  y)  z 0 z 1 y2 1    1  1   z  (1  xy )  z  1  z xy  z  1  0 z 1 y2 1   x2 1 y2 1 y2  1  x  y    x  y   z  x2  1  x2 1  x2 2  z  ( xz  yz )  z 0 z 2 0 Từ ta có điều phải chứng minh Câu 5: Gọi A địa điểm có nhiều tuyến đường (gồm ca đường xuất phát từ A đến A) Các địa điểm lại ta chia thành loại: Loại 1: Các đường xuất phát từ A có n(1)  m tuyến đường Loại 2: Các tuyến đến A có n    n tuyến Loại 3: Khơng có tuyến đến A có n(3)  p tuyến Do m  n  p  17 và: Số tuyến liên quan đến A có m  n tuyến Số tuyến khơng liên quan đến A không vượt m  n Gọi S số cách thiết lập hết 18 địa danh thì: S  m  n  p  m  n   mn  mn   p  1 m  n  p  1  m  n  p  1  đẳng thức Cosi) Dấu xảy m  p  6, n  Vậy có tối đa 108 cách thiết lập hết 18 địa danh  108 (Áp dụng bất SỞ GIÁO DỤC VÀ ĐÀO TẠO NAM ĐỊNH ĐỀ THI TUYỂN SINH LỚP 10 TRƯỜNG THPT CHUN ĐỀ CHÍNH THỨC Mơn thi: TOÁN (chuyên) Năm học: 2018 - 2019 Thời gian làm bài: 150 phút (Đề thi gồm: 01 trang) Câu (2,0 điểm) a) Rút gọn biểu thức P  b) Chứng minh Câu (2,0 điểm) x2 y2 x2 y   ( x  y )(1  y ) ( x  y )(1  x) (1  x)(1  y ) 1  1 1 1          2018 12 22 22 32 20172 20182  a) Giải phương trình 1  x  x  x   x  x   x  y   y ( x  y  1)  x   b) Giải hệ phương trình  4y 3  x  y    x  14 y   Câu (3,0 điểm) Cho đoạn thẳng AB C điểm nằm hai điểm A, B Trên nửa mặt phẳng bờ đường thẳng AB, vẽ nửa đường trịn đường kính AB nửa đường trịn đường kính BC Lấy điểm M thuộc nửa đường trịn đường kính BC ( M  B; M  C ) Kẻ MH vng góc với BC ( H  BC ), đường thẳng MH cắt nửa đường trịn đường kính AB K Hai đường thẳng AK CM giao E a) Chứng minh BE  BC AB b) Từ C kẻ CN  AB (N thuộc nửa đường trịn đường kính AB), gọi P giao điểm NK CE Chứng minh tâm đường tròn nội tiếp tam giác BNE PNE nằm đường thẳng BP c) Cho BC  2R Gọi O1 , O2 tâm đường tròn nội tiếp tam giác MCH MBH Xác định vị trí điểm M để chu vi tam giác O1HO2 lớn Câu (1,5 điểm) a) Tìm tất cặp số nguyên ( x; y ) thỏa mãn x2  y  41  xy b) Có số tự nhiên n khơng vượt q 2019 thỏa mãn n3  2019 chia hết cho Câu (1,5 điểm) a) Cho số thực dương a, b thỏa mãn a  b  Chứng minh  a  b    a  b   4ab   a  3b b  3a  b) Cho 100 điểm mặt phẳng cho bốn điểm có ba điểm thẳng hàng Chứng minh ta bỏ điểm 100 điểm để 99 điểm cịn lại thuộc đường thẳng -HẾT Họ tên thí sinh: Họ tên, chữ ký GT 1: Số báo danh: Họ tên, chữ ký GT 2: Trang 1/5 SỞ GIÁO DỤC VÀ ĐÀO TẠO NAM ĐỊNH ĐỀ CHÍNH THỨC ĐÁP ÁN VÀ HƯỚNG DẪN CHẤM THI ĐỀTHI TUYỂN SINH LỚP 10 TRƯỜNG THPT CHUYÊN Năm học: 2018 - 2019 Mơn : TỐN (chun) (Hướng dẫn chấm gồm: 05 trang) Câu 1: (2,0 điểm) Điểm Nội dung a) (1,0 điểm) Điều kiện: x   y; x  1; y  P 0,25 x3  x  y  y  x3 y  x y x  xy  y  x  y  x y  ( x  y )(1  y )(1  x) (1  y )(1  x) 0,25 x2  x2 y  x  y  1 x  x  xy  y b) (1,0 điểm) Đặt S   0,25 0,25 1 1 1         12 22 22 32 20172 20182 0,25 Ta có  1  1        n (n  1)  n n   n(n  1) (n  * )  1   1    1  n n  n n    1   1  1  Áp dụng đẳng thức ta S  1     1      1     2  3  2017 2018  = 2018   2018 (điều phải chứng minh) 2018 0,25 0,25 0,25 Câu 2: (2,0 điểm) Nội dung Điểm a) (1,0 điểm) Điều kiện: x2  x     1  x  x  x   x  x   2(1  x) x  x   x  x  (1) Đặt 0,25 x  x   y ( y  0) y  PT (1) trở thành y  2(1  x) y  x     y  2 x Với y  x2  x    x  1  (thỏa mãn điều kiện) Với y  2 x x  x   2 x (vô nghiệm) Trang 2/5 0,25 0,25   Phương trình có tập nghiệm 1  6; 1  0,25 2) (1,0 điểm) Điều kiện x  8; y  1; x  y   x  y   ( x  y )( y  1)  (1)  Hệ cho tương đương  4y  x   x  14 y  (2)  y 1 1  Nhận xét: y  1 y  khơng thỏa mãn, (1)  x y x y  20  y 1 y 1 0,25 x y   x  y  Thế vào (2) ta phương trình y 1 y    y  y  10 y  11      y 1     y   y  10 y   0,25     y  3    y  1  (3)  y 1    y 1   Với 1  y   2 2 3  ;  ;2 y   1 y 1   2  y 1   2y 1  y 1   2y 1 Do (3)  y    y   x  thỏa mãn điều kiện Vậy nghiệm hệ ( x; y)  (7;3) 0,25 0,25 Câu 3: (3,0 điểm) Điểm Nội dung a) (1,0 điểm) Ta có BME  BKE  900 nên tứ giác BMKE nội tiếp 0,25  HKB  CEB mà HKB  BAE (vì phụ với HKA )  BAE  CEB 0,25 BEC đồng dạng với BAE (vì ABE chung BAE  CEB ) 0,25 Do BE BC   BE  BC AB AB BE b) (1,0 điểm) Xét tam giác vng ABN có CN  AB  BN  BC AB mà BE  BC AB suy BN  BE hay BNE cân tai B suy BNE  BEN (1) Trang 3/5 0,25 0,25 Mặt khác, theo câu ta có CEB  BAE BAE  BNP suy CEB  BNP (2) Từ (1) (2) suy PNE  PEN hay PNE cân P  NP  PE Vì NP  PE BN  BE nên BP  NE Suy BP đường phân giác góc EBN EPN Do tâm đường tròn nội tiếp tam giác BNE PNE nằm đường thẳng BP c) (1,0 điểm) Gọi giao điểm O1O2 với MB, MC I J Ta có CMH  MBH (vì phụ MCB ) Suy O1MH  O2 BH Mặt khác O1HM  O2 HB  450 O2 Suy MO1H đồng dạng với BO2 H O1 O H MH MH MC Do dó  mà  O2 H HB HB MB O’ O H MC   O2 H MB O H MC ) O1HO2 đồng dạng với CMB (vì O1HO2  CMB  900  O2 H MB Suy HO2O1  MBC  MBC  HO2 I  1800 0,25 0,25 0,25 0,25 0,25 Suy tứ giác BHO2 I nội tiếp  MIJ  O2 HB  450 Suy MIJ cân M  MI  MJ Ta có MO2 I  MO2 H (g.c.g) suy MI  MH O2 I  O2 H Tương tự có O1H  O1 J Chu vi tam giác O1HO2 O1H  HO2  O1O2  JO1  O1O2  O2 I  2MI  2MH Ta có MH  R Suy chu vi tam giác O1HO2 lớn 2R MH  R , hay M nằm nửa 0,25 0,25 đường trịn đường kính BC Câu 4: (1,5 điểm) Nội dung Điểm a) (0,75 điểm) Phương trình cho tương đương x2  xy  y  41  (1) 82 Mặt khác từ (1) ta có y số lẻ, nên y  1;9 Với y   x  x  36   x  Ta có 'x  82  y   y  Với y  1  x2  x  36   x  0,25 0,25 x  Với y   x  x      x  2 Trang 4/5  x  1 Với y  3  x  x      x  2 Vậy có cặp số nguyên ( x; y ) thỏa mãn là: (1;3),(2;3),(1; 3),(2; 3) b) (0,75 điểm) Đặt n  6q  r , r 0,1,2,3,4,5 Khi n3  2019 chia hết cho r  chia hết cho Nếu r chẵn r  lẻ, r  khơng chia hết cho Suy r  1,3,5 Với r   r   không chia hết cho Với r   r   30 Với r   r   128 không chia hết cho Suy n  6q  Mà  n  2019   q  336 Vậy có tất 337 số tự nhiên n thỏa mãn đề 0,25 0,25 0,25 0,25 Câu 5: (1,5 điểm) Nội dung Điểm a) (0,75 điểm) Bất đẳng thức cho tương đương 1   a  3b b  3a Áp dụng BĐT Cô si cho số dương ta có a a ab 1 a ab       (1) a  b a  3b  a  b a  3b  a  3b 0,25 b 2b 11 2b       (2) a  3b  a  3b  a  3b a  b 13 a  13 a          (3) a  3b  a  b  a  3b  a  b  0,25 13 b  Chứng minh tương tự ta có     (4) b  3a  a  b  1 Từ (3) (4) suy   (điều phải chứng minh) a  3b b  3a 0,25 Dấu "  " xảy a  b  b) (0,75 điểm) 0,25 Nếu tất 100 điểm thuộc đường thẳng tốn hiển nhiên Nếu 100 điểm thẳng hàng Ta chọn bốn điểm A, B, C, D mà tất thẳng hàng Theo giả thiết điểm A, B, C, D phải có điểm thẳng hàng, giả sử điểm A, B, C thuộc đường thẳng d , điểm D nằm đường thẳng d Ta 0,25 chứng minh 96 điểm lại thuộc đường thẳng d phương pháp phản chứng Giả sử 96 điểm lại, tồn điểm E nằm đường thẳng d Xét bốn điểm A, B, D, E phải có điểm thẳng hàng Do điểm A, B, D không thẳng hàng, điểm A, B, E không thẳng hàng nên điểm A, D, E thẳng hàng điểm B, D, E thẳng hàng Từ (1) (2) suy Trang 5/5 Trường hợp điểm A, D, E thẳng hàng điểm B, D, E khơng thẳng hàng, điểm C, D, E không thẳng hàng, điểm B, C, D, E khơng có điểm thẳng hàng, trái với giả thiết Trong trường hợp B, D, E thẳng hàng tương tự, điểm A, C, D, E khơng có 0,25 điểm thẳng hàng, trái với giả thiết Như điểm A, B, C thuộc đường thẳng d , phải có 96 điểm thuộc d Bài toán chứng minh Chú ý: - Nếu thí sinh làm đúng, cách giải khác với đáp án, phù hơp kiến thức chương trình THCS tổ chấm thống cho điểm thành phần đảm bảo tổng điểm hướng dẫn quy định - Tổng điểm tồn khơng làm trịn HẾT Trang 6/5 ĐẠI HỌC QUỐC GIA HÀ NỘI TRƯỜNG ĐẠI HỌC KHOA HỌC TỰ NHIÊN ĐỀ THI TUYỂN SINH LỚP 10 Môn: TOÁN CHUYÊN Năm học: 2018-2019 Câu  xy ( x  y )  a) Giải hệ phương trình :  3 3   x  y  x y   x  1 y  1  31 b) Giải phương trình:  x   x   x   x Câu a) Cho x, y số nguyên cho x2  xy  y ; xy  y  x chia hết cho Chứng minh x2  y  x  y chia hết cho b) Cho a1 , a2 , , a50 số nguyên thỏa mãn:  a1  a2  a50  50 , a1  a2   a50  100 Chứng minh từ số cho chọn vài số có tổng 50 Câu Cho ngũ giác lồi ABCDE nội tiếp (O) có CD / / BE Hai đường chéo CE BD cắt P Điểm M thuộc BE cho MAB  PAE Điểm K thuộc AC cho MK song song AD, điểm L thuộc đường thẳng AD cho ML // AC Đường tròn ngoại tiếp tam giác KBC cắt BD, CE Q S (Q khác B, S khác C) a) Chứng minh điểm K, M, Q thẳng hàng b) Đường tròn ngoại tiếp tam giác LDE cắt BD, CE tai T R (T khác D, R khác E) Chứng minh M, S, Q, R,T thuộc đường tròn c) Chứng minh đường tròn ngoại tiếp tam giác PQR tiếp xúc (O) Câu Cho a, b, c số thực dương Chứng minh  ab bc   1       2 b  c  a  b bc   ab ĐÁP ÁN Câu  xy ( x  y )  a) Giải hệ phương trình :  3 3   x  y  x y   x  1 y  1  31 Ta có hệ phương trình:  xy  x  y    2 ( x  y )( x  xy  y )   xy   7( x  y  xy  1)  31  xy ( x  y )     ( x  y )  x  y   3xy    xy    x  y   xy  1  31  ab  2  a  a  3b   b   a  b  1  31 Đặt a  x  y; b  xy hệ trở thành:  ab   3 a  3ab  b   a  b  1  31   a  b   a  b   3ab   3ab   a  b  1  31     a  b   3ab(a  b)  3ab  7( a  b)  24    a  b   6(a  b)  3.2   a  b   24    a  b    a  b   30    a  b   27  (a  b)  3  (a  b  3)  a  b   3(a  b)  10       a  b  a  b  3(a  b)  10    a  b  a  2 (do a   x  y   xy  4b)    ab  b  x  y    x  y 1  xy  Vậy hệ có nghiệm  x; y   1;1 b) Giải phương trình:  x   x   x   x Điều kiện xác định:  x  Đặt a  x , b   x  a, b  0 Khi phương trình tương đương với: 9  3ab  a  5b  2a  b   3ab  7a  5b  2  2a  b   2a  2ab  4a  ab  b  2b  3a  3b    2a(a  b  2)  b  a  b     a  b      a  b   2a  b  3   a  b   b   a   3a (2  a)  2a  10   2a  b   b   2a   3a   2a   a    2a  1   3a  1 a  1  a   x  (tm)      a  1   a   x  1(tm) Vậy phương trình có tập nghiệm S   ;1 9  Câu a) Cho x,y… Ta có: x  xy  y    xy  y  x   x  xy  y  x  x  xy   xy  y    x  y    x  y  x  y  1 Lại có: x2  xy  y, xy  y  x chia hết cho   x  y  x  y  1 chia hết cho TH1: Nếu x  y chia hết cho5 y   x  mod5   x2  xy  y  x  x  x  x  3x  1 (mod5) , x chia hết cho chia dư +)Nếu x chia hết cho y vậy, toán chứng minh +)Nếu x chia cho dư y chia dư 2, x2  y  x  y  2.9   2.3  30  0(mod5) Ta có điều phải chứng minh TH2) Nếu x  y  chia hết cho x  y  1 mod5   x  xy  y   y  1  y( y  1)  y  y  1 mod5 Do y chia dư x chia dư nên: x2  y  x  y  2.16  16  2.4   60   mod5 Vậy ta có điều phải chứng minh b) Cho……… Nếu tồn n :1  n  50 : a1  a2   an  50 kết luận tốn hiểu nhiên a1  a2   an  49  an1  a1  a2   an1  51 Xét:  n  49 :  TH 1: an1   a1  a2   an  49 an  an3   a50  49 Nên n  24  a1  an2 ; a2  an3 ; ; an  a2n1  49  a1  a2   an  an2  an3   a2n1  an2   a49  a50 Điều vô lý nên: n  25  49  a1  a2   an  na1  25a1  a   a   a2  .an  48; a2   an1  50 TH2: an1  an  an3   a50  100   a1  a2   an1   49  49   49  n  an   49  n   n  33  49   a1  a2   a16    a17   an   16   n  16  a17  16  17a17  a17   a17   a1  a2  a17  Nếu an1  18 đặt a1  a2   an1  50  k  k  1  18  an1   50  k   49  k   k  17  ak 1   an1  50 Nếu an1  19  49   49  n  an   49  n 19  n  47  a1  a2   a45  Vì a45    a1  a2   a44    a45   an   44   n  44 a45  44   47  44  49 Đặt an1  50  k   k  31  a1   ak  an1  50 Vậy ta có điều phải chứng minh a1   ak  1 Câu 3: a) Chứng minh điểm K, M, Q thẳng hàng Do tứ giác BCKQ BCDA nội tiếp nên: CKQ  CBQ  CAD  KQ / / AD Mặt khác MK / / AD nên K, M, Q thẳng hàng b) Đường tròn ngoại tiếp tam giác………… Chứng minh tương tự ta có: R, M , L thẳng hàng MQ / / AD nên RMQ  RLD  ETD  tứ giác RTMQ nội tiếp Chứng minh tương tự RMSQ nội tiếp đó: M , S , Q, R, T thuộc đường tròn c) Chứng minh đường tròn ngoại tiếp Bổ đề: cho tam giác ABC, M nằm d / / BC lấy E khác M d, AM cắt BC I Đường qua M / / AB cắt BE J , IJ / / AE Chứng minh MJ cắt AE, AC S T, ME cắt AC G Ta có MG//BC suy MS AP AG MA MA AG , ME cắt AB P ta có:     AE / / IJ  MJ PB GC MI MI GC Quay trở lại toán: AM cắt BC, (O) I J khác A Áp dụng bổ đề ta có: IR / / AE, IQ / / AB Do IRE  AEC  AJC  nên RIJC tứ giác nội tiếp Chứng minh tương tự ta có DQIJ tứ giác nội tiếp Do đó: RJI  IJQ  RPD  2PCD  CPD  1800 nên RPQJ nội tiếp Kẻ tiếp tuyến Jx (O) Ta có: xJR  xJA  RJA  ADJ  PDC  ADP  MAC  ADP  PAD  APB  PEJ  MAC  PED Suy : Jx tiếp xúc với  PQR  hay ta thu được:  PQR  tiếp xúc với  O  Vậy ta có điều phải chứng minh Câu 4: Áp dụng BĐT Cauchy-Schwarz ta có:  ab bc   1  bc    ab              b  c  a  b bc   ab bc ab bc  ab c  b b   a c   b b   a 2            a  b b  c  a  b b  c   a  b b  c   a  b b  c  b   c b   a     2  ab ab bc bc Vậy ta có điều phải chứng minh Dấu "  " xảy a  b  c SỞ GIÁO DỤC VÀ ĐÀO TẠO NAM ĐỊNH ĐỀ THI TUYỂN SINH LỚP 10 TRƯỜNG THPT CHUN ĐỀ CHÍNH THỨC Mơn: TỐN (chun) Năm học: 2016 - 2017 Thời gian làm bài: 150 phút (Đề thi gồm: 01 trang) Câu (2,0 điểm) a) Đơn giản biểu thức x   x   x   x  với x  b) Cho a, b, c số thực thỏa mãn điều kiện a  b  c  ; Tính giá trị biểu thức 1 47    a  b b  c c  a 60 a b c   bc ca ab Câu (2,0 điểm) x  3x    3x  x  2   x  y  3x   b) Giải hệ phương trình  2   x  y  x  y   a) Giải phương trình Câu (3,0 điểm) Cho tam giác ABC có ba góc nhọn, nội tiếp đường trịn  O  Các đường cao AK , BM , CN tam giác ABC cắt H a) Chứng minh NKH  MKH b) Đường thẳng MN cắt đường tròn  O  hai điểm I , J Chứng minh AO qua trung điểm IJ c) Gọi P trung điểm BC , diện tích tứ giác AMHN S Chứng minh 2.OP  S Câu (1,5 điểm) a) Chứng minh tồn vô hạn ba số nguyên  x, y, z  thỏa mãn xyz  x5  y3  7z  b) Tìm tất số ngun khơng âm a, b, c thỏa mãn  a  b    b  c    c  a   6abc 2 a3  b3  c3  chia hết cho a  b  c  Câu (1,5 điểm) a) Cho x, y, z số thực thỏa mãn  x  y  x  z   1; y  z Chứng minh  x  y   y  z   z  x  b) Trên bảng ban đầu ghi số số Ta thực cách viết thêm số lên bảng sau: bảng có hai số, giả sử a, b ; a  b , ta viết thêm lên bảng số có giá trị a  b  ab Hỏi với cách thực vậy, bảng xuất số 2016 hay khơng? Giải thích -HẾT Trang 1/4 SỞ GIÁO DỤC VÀ ĐÀO TẠO NAM ĐỊNH ĐÁP ÁN VÀ HƯỚNG DẪN CHẤM THI ĐỀTHI TUYỂN SINH LỚP 10 TRƯỜNG THPT CHUYÊN Năm học: 2016 - 2017 ĐỀ CHÍNH THỨC Mơn: TỐN (chun) (Hướng dẫn chấm gồm: 04 trang) Câu 1: (2,0 điểm) Điểm Nội dung a) (1,0 điểm) x   x 1  x   x 1  x 1 x 1 1  x 1 x 1 1     x 1 1  x 1 1   x 1 1    x 1 1 0,25 0,25 x 1 1 0,25  0,25 x    b) (1,0 điểm) Do a  b  c  nên  b  c   c  a    a  b  a b c      bc ca a b bc ca a b 6    3 bc ca ab 1    6   3 bc ca ab 47 47 17    3 60 10 10 0,25 0,25 0,25 0,25 Câu 2: (2,0 điểm) Điểm Nội dung a) (1,0 điểm) Điều kiện x  3x   0;  3x  Đặt a  x  3x  1; b  x  1; a; b  Khi ta 2b2  a   3x 0,25 Phương trình cho trở thành: a  2b2  a  2b  2b2  a  2b  a  2b2  a   2b  a  0,25   a  2ab  b2    a  b x   x  3 Với a  b ta x  3x   x   x  3x    0,25 Thử lại ta nghiệm phương trình là: x  0; x  3 0,25 Trang 2/4 2) (1,0 điểm) 1  2 2   x  y  3x    2  x  y  x  y   0,25 Cộng vế với vế (1) (2) ta x  y  x  y   Phương trình (3) tương đương với  x  y    x  y  2 2  3 0 x  y x     x  y   y  Ta thấy x  y  thỏa mãn (1) (2) Hệ cho có nghiệm  x; y   1;1 0,25 0,25 0,25 Câu 3: (3,0 điểm) Điểm Nội dung a) (1,0 điểm) A I Chứng minh tứ giác BNHK tứ giác CMHK tứ giác nội tiếp M 0,25 N H J O C B K Chứng minh tứ giác BNMC nội tiếp P 0,25 S Chứng minh NBM  NKH ; MCN  MKH ; NBM  MCN 0,25 Từ chứng minh MKH  NKH b) (1,0 điểm) 0,25 Kẻ đường kính AS  O; R  , nối BS Ta có BSA  ACB (hai góc nội tiếp chắn cung) Tứ giác BNMC nội tiếp nên ANM  ACB (cùng bù với BNM )  ANM  ASB Trong tam giác ABS ta có ABS  900 (góc nội tiếp chắn nửa đường trịn) nên BSA  BAS  900 Suy ANM  BAS  900  AO  MN hay AO  IJ Tam giác OIJ cân O  OI  OJ  , AO  IJ suy AO qua trung điểm IJ Trang 3/4 0,25 0,25 0,25 0,25 c) (1,0 điểm) Ta có ABS  SCA  900 (góc nội tiếp chắn nửa đường trịn)  SC  AC; SB  AB  CH song song SB (cùng vng góc BC )  BH song song SC (cùng vng góc AB )  BHCS hình bình hành  P trung điểm HS (vì P trung điểm BC ) Do OP đường trung bình tam giác AHS  OP  0,5 AH Trong tứ giác ANHM ta có AH  NA2  NH  MA2  MH NA2  NH  NA.NH hay NA2  NH  4SNAH (Vì NA.NH  4SNAH ) 0,25 MA2  MH  2MA.MH  MA2  MH  4SMAH (Vì 2MA.MH  4SMAH )  2.HA2  4S ANHM  8.OP2  4S hay 2.OP2  S Dấu xảy NH  NA; MH  MA , MAN  900 hay BAC  900 (mâu thuẫn với tam giác ABC nhọn) Do không xảy dấu bằng, suy 2.OP  S 0,25 Câu 4: (1,5 điểm) Điểm Nội dung a) (0,75 điểm) Với số nguyên k  ta có: x5  y3  7z2   k 30 x5  k 30 y3  k 30 7z2    k x    k 10 y    k 15.z   Do  x0 ; y0 ; z0  số nguyên thỏa mãn điều kiện đề với số nguyên k  ta có  k x0 ; k 10 y0 ; k 15 z0  số nguyên khác thỏa mãn điều kiện đề 0,25 0,25 Ta thấy 1; 2;3 thỏa mãn điều kiện đề Từ suy tồn vơ hạn ba số nguyên  x, y, z  thỏa mãn xyz  x5  y3  7z2    Lưu ý: Học sinh k ; k 10 ; k 15 với k số nguyên khác tùy ý số nguyên thỏa mãn đề b) (0,75 điểm) Giả sử a; b; c số ngun khơng âm thỏa mãn đề bài, ta có:  a  b  b  c    c  a  2 0,25 0,25  6abc  a  b  c  ab  bc  ca  3abc (1) 2  Phân tích a3  b3  c3  3abc   a  b  c  a  b2  c  ab  bc  ca  (2) 0,25 Từ (1) (2) a3  b3  c3  3abc  3abc  a  b  c  hay a3  b3  c3  3abc  a  b  c  1 Do a3  b3  c3  chia hết cho a  b  c  nên ta chia hết cho a  b  c  Suy a  b  c  Thử lại: a  b  c  thỏa mãn Vậy có số  a; b; c    0;0;0  thỏa mãn đề Trang 4/4 0,25 Câu 5: (1,5 điểm) Điểm Nội dung a) (0,75 điểm) Đặt x  y  a; x  z  b ta ab  1; a  b Bất đẳng thức cần chứng minh trở thành: 1 a b 1  2 4 2    a  b2  4 2 a b  a  b ab a  b  2ab a  b2  2  a  b2   0,25  a  b2  0,25 Do ab  1; a  b nên a  b2  2ab hay a  b2     Mặt khác a  b   a  b2   2 a  b2  2  Vậy a  b2  a  x  y 2  b2     y  z tức  a  b2     z  x 0,25  b) (0,75 điểm) Đặt k  ab  a  b   a  1 b  1  0,5 Nếu số a, b tồn số chia dư k chia dư Ban đầu bảng gồm có số số (một số chia dư 1; số chia dư 2) Suy thời điểm, bảng ln có số chia dư số lại chia dư Do với cách thực đề bài, bảng khơng thể xuất số 2016 (Vì số 2016 chia hết cho 3) Lưu ý: Học sinh dùng bất biến theo modun 10 cách nhận xét chữ số tận số viết bảng; sử dụng cách liệt kê số viết bảng 0,25 Chú ý: - Nếu thí sinh làm đúng, cách giải khác với đáp án, phù hơp kiến thức chương trình THCS tổ chấm thống cho điểm thành phần đảm bảo tổng điểm hướng dẫn quy định - Tổng điểm tồn khơng làm tròn HẾT Trang 5/4 Họ tên thí sinh: Số báo danh: Họ tên, chữ ký GT 1: Họ tên, chữ ký GT 2: Trang 6/4 SỞ GIÁO DỤC – ĐÀO TẠO TỈNH HÀ TĨNH KỲ THI TUYỂN SINH LỚP 10 THPT NĂM HỌC 2018-2019 MƠN THI: TỐN CHUN 1 Câu Cho x, y, z số hữu tỉ thỏa mãn   Chứng minh x2  y  z số x y z hữu tỉ Câu a) Giải phương trình : x2  3x   x   xy  x  y  5 b) Giải hệ phương trình:  1  x2  2x  y  y   Câu a) Cho phương trình x2  2mx 1  2m  Chứng minh phương trình ln có hai nghiệm x1; x2 với m Tìm m để P  x1 x2  đạt giá trị nhỏ x  2mx2   2m b) Cho x, y, z  thỏa mãn x  y  z  Chứng minh xy  xy  z yz xz   yz  x xz  y Câu Cho đường tròn tâm (O) dây cung AB cố định khơng phải đường kính Điểm C khác A, B di động AB Đường tròn tâm P qua C tiếp xúc với (O) A, đường tròn tâm Q qua C tiếp xúc với (O) B Các đường tròn (P), (Q) cắt điểm thứ hai M Các tiếp tuyến đường tròn (O) A B cắt I a) Chứng minh MC phân giác AMB điểm A, M, O, B, I thuộc đường tròn b) Chứng minh điểm C thay đổi tâm đường trịn ngoại tiếp tam giác MPQ thuộc đường thẳng cố định Câu Cho a1  a2   an , n số tự nhiên không âm, a số ngun dương khơng có số liên tiếp Đặt S  a1  a2   an Chứng minh luon tồn số phương b thỏa mãn Sn  b  Sn1 ĐÁP ÁN Câu 1 1 Từ giả thiết cho ta có: x  y  z  xz  yz  xy  xy  xz  yz   x  y  z  x  y  z  xy  xz  yz  x  y  z  x yz  x  y  z số hữu tỉ Vậy ta có điều phải chứng minh Câu 2 a) Giải phương trình x  3x   x  Điều kiện xác định: x  2 Phương trình cho tương đương với 16 x  12 x   x     16 x  x    x    x     x  1  x   2  4 x  x    41 4 x   x    x   x    x   x      x   2 x    x   2 x  4 x  x    x   33   2  x   Vậy nghiệm phương trình cho x   41  33 ;x  8  xy  x  y  5  b) Giải hệ phương trình:    x2  2x y  y    x  0; x  Điều kiện xác định :  y  0; y   Từ phương trình (1) ta có: xy  x  y  5   x  1 y  1  4  x2  2x  a2  x 1  a  2 Đặt  y   b   y  y  b   ab  4  1 1      x  2x y  y a 1 b 1 a2  b2  2 a2  b2  2    a 2b  a  b  17  a  b   a  b    34   a  b    a  b    a  b   2ab   a  b   2ab    4    b   a  a   a  2 a  2  x  1 TH 1:   b  y  a  x  TH :   b  2  y  1 Vậy nghiệm hệ cho  1;3  3; 1 Câu 3: a)  '  m2  2m    m  1  nên phương trình ln có hai nghiệm với m  x1  x2  2m Theo định lý Vi et ta có:  x1 x2  2m   x1 x2  P x1   x1  x2  x2   2m Ta có P P x1 x2   x1  x2   x1 x2  2m 4m  4m   11 4m  4m   2m  1 P 4m    1 Vậy giá trị nhỏ P 1 đạt m Câu b Áp dụng bất đẳng thức Cơ si ta có: 1 x y      z  x  y  z   x  z y  z  Chứng minh hoàn toàn tương tự ta có: xy  xy  z xy  xy  z  x  y  z  xy yz  y z  xz 1 z x        ;  yz  x  x  y z  x  xz  y  y  z x  y  1 x y z z x y  P        2 x z x y x z y  z x y y  z   x z   y x   z y   P              x  z x  z   x  y x  y   y  z y  z    P   2 x yz Vậy giá trị nhỏ P đạt Câu O Q M P A B C J I a) Chứng minh MC phân giác góc AMB điểm A, M, O, B, I thuộc đường tròn P O Q O Ta có: IA tiếp tuyến chung     IB tiếp tuyến chung      P, A, O Q, B, O thẳng hàng thẳng hàng P AMC  BAI Xét đường trịn   có (góc nội tiếp góc tạo tiếp tuyến dây cung chắn cung AC) Xét đường tròn chắn cung BC) Mà  Q  có: (góc nội tiếp góc tạo tiếp tuyến dây cung BAI  ABI IAB ( cân A)  AMC  BMC  MC Ta có: Mà BMC  ABI phân giác AIB  BAI  ABI  1800 AMB (tổng ba góc tam giác) BAI  ABI  AMC  BMC  AMB  AIB  AMB  1800  Lại có: Tứ giác AMBI nội tiếp (Tứ giác có tổng hai góc đối OAI  OBI  900 ( gt )  OAI  OBI  1800  Tứ giác AOIB 1800 ) nội tiếp (Tứ giác có 180 ) tổng hai góc đối Vậy điểm A, M, O, B, I thuộc đường tròn b) Chứng minh điểm C thay đổi tâm đường trịn ngoại tiếp tam giác MPQ thuộc dường thẳng cố định Gọi J trung điểm OI PA  PM  MPO  PAM  PMA  2PAM  2OAM Ta có tam giác AMP cân P  nên: (góc ngồi tam giác tổng hai góc khong kề với nó) Tương tự ta có: Tam giác BMQ cân Q Mà OAM  OBM  QM  QB  nên MQO  2OBM (hai góc nội tiếp chắn cung OM)  MPO  MQO  Tứ giác PMOQ tứ giác nội tiếp (Tứ giác có hai góc nội tiếp chắn cung nhau) Do đường trịn ngoại tiếp tam giác MPQ đường tròn ngoại tiếp tứ giác PMOQ A, M , O, B, Q JM  JB, QM  QB Các điểm thuộc đường trịn đường kính OI nên  JMQ  JOQ JMOQ hay tứ giác nội tiếp Suy P,M,O,Q,J thuộc đường trịn  Ta có I, O cố định nên JO cố định Trung trực JO cố định JO MPQ Vậy tâm đường tròn ngoại tiếp tam giác thuộc trung trực cố định Câu S  a1  a2   an  Sn1  Sn  an1 Vì n Ta có: S n1  S n   Sn1    Sn   Sn  an 1  Sn  S n   an1  Sn  Vì dãy số khơng có hai số liên tiếp nên an1  an  an  an1   an1  an1  2.2 a2  a1   an 1  a1  n.2  n.an1  a1  a2   an  1     n   nan1  n(n  1)  S n  nan1  n(n  1)   S n  Ta chứng minh : an1  nan1  n  n  1   an21  2an1   4nan1  4n  n  1   an1  2n  1  0(luondung ) k Sn1  Sn  Do ta ln có: nên tồn số thỏa mãn b  k2 Vậy số phương cần tìm Sn1  k  Sn SỞ GIÁO DỤC VÀ ĐÀO TẠO BÌNH ĐỊNH Đề thức KỲ THI TUYỂN SINH VÀO LỚP 10 THPT CHUN NĂM HỌC 2018 – 2019 Mơn thi: TỐN (Chun Toán) Ngày thi: 03/ 6/ 2018 Thời gián làm bài: 150 phút (không kể thời gian phát đề) Bài 1: (2,0điểm)  Cho biếu thức : T  a b   ab  a  b a  b3 :   a b ab a b    , với a  b,a  0, b    a) Rút gọn biểu thức T b) Chứng tỏ T > Cho n sô tự nhiên chẵn, chứng minh số 20n  3n  16n  1chia hết cho số 323 Bài 2: (2,0 điểm) Giải bất phương trình: 3x   7x  4  x  y  x  y   Giải hệ phương trình:   x  y   5  xy  Bài 3: (1,0 điểm) Cho phương trình: (m  1)x  2(2m  3)x  5m  25  (m tham số) Tìm giá trị m số nguyên cho phương trình có nghiệm số hữu tỉ Bài 4: (4 điểm) Cho tam giác ABC có góc nhọn AB  BC; BC  CA Xác định vị trí điểm M thuộc miền tam giác ABC (gồm cạnh miền tam giác) cho tổng khoảng cách từ M đến ba cạnh nhỏ Cho tam giác ABC (AB < AC) có goc nhọn, đường cao AD, BE, CF cắt H Đường thẳng EF cắt đường thẳng BC AD K I Qua F kẻ đường thẳng song song với AC cắt AK, AD M N Gọi O trung điểm BC Chứng minh: a) DA phân giác FDE b) F trung điểm MN c) OD  OK  OE2 BD  DC  OD  DK Bài 5: (1,0 điểm) Cho hai số dương a, b thỏa mãn a   Chứng minh rằng: b 2 1  1 25  a   b    a  b  Lượt giải: Bài 1: (2,0điểm) a) Rút gọn T: Với a  b,a  0, b  , ta có: T a  b  ab a  a b  b a  b3  a  b3 a  b  ab :   a b a b a b a b  Vậy : T     a b ab   a b a b    ab ab ab a  b  ab , với a  b, a  0, b  ab b) Chứng tỏ T > Ta có: T  a  b  ab , với a  b, a  0, b  (kết câu 1.a) ab  T a b  ab  ab   a b  ab   (vì ab  0, a  b  với a  b, a  0, b  ) Vậy T > Ta có: a n  bn  (a  b)(a n 1  a n  b  a n3 b2   abn  bn1 )  a n  bn  m(a  b) (a, b, n, m  ) (*) Vì n số tự nhiên chẵn nên n = 2k ( k  )  A = 20n  3n  16n 1  400k  9k  256k 1 Áp dụng (*), có: A  (400k  1k )  (256k  9k )  399x  247y  19  21x  19.13y (x, y  )  A 19 với số tự nhiên n chẵn (1) có: A  (400k  9k )  (256k  1k )  391p  255q  17  23p  17 15q (p,q  )  A 17 với số tự nhiên n chẵn (2) mà 17 19 hai số nguyên tố nên từ (1) (2) suy ra: A 17 19 với số tự nhiên n chẵn Vậy 20n  3n  16n 1 323 với số tự nhiên n chẵn Bài 2: (2,0 điểm) Giải bất phương trình: 3x   7x  (1) 2 2 2 2     x  x  x  x  (1)      3 3 2 9x  12x   7x  9x  5x     (x  1)(9x  4)  (9x  9)(9x  4)    2  2 x   x  2    x   9x    9x  1  x     | 4  x  y  x  y   Giải hệ phương trình:   x  y   5  xy  (2) Vậy bất phương trình (1) có tập nghiệm S   x  2 4 x  9  4S S2  5S    S  P  S  2; P      Đặt S = x + y  0; P = xy  0, ta có: (2)   4S  P  S   5  S  3; P  S3   S 8 3  0) Khi đó: S = 2; P  x, y nghiệm phương trình: t  2t   vô nghiệm (  '  5 S = – 3; P = x, y nghiệm phương trinh: t  3t    t1  1; t  2 Vai trò x, y hệ (2) nhau, hệ (2) có hai nghiệm: (x = – 1; y = – 2), (x = – 2; y = – 1) Bài 3: (1,0 điểm) Phương trình: (m  1)x  2(2m  3)x  5m  25  (3) Có  '   (2m  3)  (m  1)(5)(m  5)  9m2  42m  34  (3m  7) 15 (3) có nghiệm hữu tỉ với m  ' phương, suy ra: (3m  7)2  15  n (n  )  (3m – – n)(3m – + n) = 15 (m, n  ) (4) Phương trình (4) tương đương với hệ phương trình: 3m   n  15 (4.1),  3m   n  1 3m   n  1 (4.2),  3m   n  15 3m   n  5 (4.3),  3m   n  3 3m   n  3 (4.4)  3m   n  5 3m   n  15  3m   n  3m   n   3m   n  15 3m   n   3m   n  3m   n   3m   n  (4.5), (4.6) (4.7), (4.8) Giải hệ trên, suy hệ phương trình (3) có nghiệm hữu tỉ khi: m = m = Bài 4: (4 điểm) Gọi khoảng cách từ M đến BC, CA, AB là: x, y, z Ta có: 2SABC  x  BC  y  CA  z  AB  (x  y  z)AB (vì AB  BC  CA )  xyz  2S AB + Nếu AB > BC dấu “=” xãy khi: M  C + Nếu AB = BC > CA dấu “=” xãy khi: M thuộc cạnh AC + Nếu AB = BC = CA M điểm tam giác ABC Vậy tổng khoảng cách từ M đến ba cạnh tam giác nhỏ chiều cao cạnh lớn khi: M trùng C (nếu AB > BC = CA), M nằm cạnh AC (nếu AB = BC > CA) M điểm tam A giác ABC AB = BC = CA a) DA phân giác FDE M E P F I H N K B D O Q C Dễ chứng minh tứ giác BDHF nội tiếp đường trịn đường kính HB (1) tứ giác ABDE nội tiếp đường trịn đường kính AB (2) (1)  HDF  HBF (nội tiếp chắn cung HF) (1’), (2)  HBF  HDE (2’) (nội tiếp chắn cung AF) (1’) (2’) suy ra: HDF  HDE Vậy DA phân giác FDE b) F trung điểm MN Qua B kẻ đường thẳng song song với AC cắt AK, AD P, Q  PQ // MN // AC Ta có: FC phân giác DFE (tương tự chứng minh câu a) mà FB  FC nên PB phân giác FC phân giác  KFD  BK FK CK KB DB =   = BD FD CD KC DC (3) Theo hệ định lí Ta-let, ta lại có: BP KB (4) (vì BP // AC) = CA KC BQ DB và: (5) (BQ // AC) = CA DC BP BQ Từ (3), (4), (5) suy ra:  BP = BQ = CA AC Khi đó, áp dụng hệ định lí Ta-let hai tam giác ABP ABQ với MF // PQ, NF // BQ, có: A MF AF FN MF FN = =  =  MF = NF  F trung điểm MN BP AB BQ BQ BQ c) Chứng minh OD  OK = OE BD  DC = OD  DK M E Từ kết câu a)  DFE = 2CFE (6) P Dễ chứng minh tứ giác BCEF nội tiếp đường tròn (O) đường kính BC, nên EOC = 2CFE F I H (7) N K B D O C Từ (6) (7) suy ra: DFE = EOC  Tứ giác DFEO nội tiếp 1 sđ OE = sđ OF = OEK (vì OE = OF = BC) 2 OE OD Từ suy ra:  ODE  OEK (g.g)   OD  OK = OE OK OE  ODE  Q S Khi đó: BD  DC =  OB  OD  OC + OD   OB2  OD2  OD  OK  OD2  OD  OK  OD   OD  DK Bài 5: 1     a  ab   b v(a > 0, b > 0) b b (x  y) lại có HĐT: 2(x  y )  (x  y)  (x  y)  x  y  (1) , dấu”=” xãy x= y có HĐT: (x  y)2  (x  y)2  4xy  (x  y)  4xy (2), dấu”=” xãy x= y Ta có: a  1    ab 1   b   a   b   1  b   1  2   1   1  1  a b  a  a   a  - Áp dụng (1), ta có:  a     b       a  b 2 2  1 dấu “=” xãy khi: a   b  a   a b b 2 2 (1’), 1 1 a a b  - Áp dụng (2), ta có:  a        (2’), dấu “=” xãy khi: a  a   b b b b b a  Từ (1’) (2’) suy ra: 1 1   (1  4)  Dấu “=” xảy khi: a  hay b  a   b       b a a  b    1 a   Vậy  a     b  1 25 , dấu “=” xãy khi: a = b =   b SỞ GIÁO DỤC – ĐÀO TẠO TỈNH CÀ MAU KỲ THI TUYỂN SINH LỚP 10 THPT NĂM HỌC 2018-2019 Mơn thi: TỐN CHUN Thời gian: 150 phút Ngày thi: 03/06/2018 ĐỀ THI CHÍNH THỨC Câu 1: Rút gọn biểu thức sau a) A  20  45  125  405 b) B     Câu 2: Cho parabol  P  : y  x đường thẳng d : y  x  a) Vẽ đồ thị  P  d mặt phẳng tọa độ b) Tìm m để d  P  đường thẳng    : y   2m  3 x  1cùng qua điểm có hồnh độ lớn Câu 3: Cho phương trình x2   2m  1 x  m2   (1) (x ẩn số) a) Tìm m để phương trình có nghiệm phân biệt b) Gọi x1; x2 nghiệm phân biệt (1) Tìm m để x1; x2 thỏa mãn  x1  x2   x1 Câu 4: Cho tam giác ABC cân A Gọi I tâm đường tròn nội tiếp tam giác, K tâm đường trịn bàng tiếp góc A O trung điểm IK a) CMR điểm B, I , C, K thuộc (O) b) CMR: AC tiếp tuyến (O) c) Tính tổng diện tích hình viên phân giới hạn cung nhỏ CI , IB, BK , KC dây cung tương ứng (O) biết AB  20, BC  24 Câu 5: Giải phương trình :   x 1   x 1   x ĐÁP ÁN Câu Ta có ngay: A  20  45  125  405    15  18  B  94  94    2   2.2 2.1    2   2  1   2  1 2  2.2 2.1   2   2   2   2   ( do2   0) Câu a) Học sinh tự vẽ b) Giao điểm (d) (P) hồnh độ phương trình: x  x  x   x  x     x   x  1     x  1 x   x   y     M  2;4  Do Đường thẳng Vậy m  qua điểm M (2;4)    2m  3   m  11 giá trị cần tìm Câu a) Để phương trình cho có nghiệm phân biệt     2m  1   m2  1   4m2  4m   4m2    4m    m  11 Vậy m phương trình có hai nghiệm phân biệt b) x1; x2 Với phương trình có hai nghiệm phân biệt  x1  x2  2m  Theo hệ thức Vi-et ta có:  x1 x2  m2  m   x1  x2   x12  x22  x1 x2   x1  x2   x1 x2 2   2m  1   m  1  4m   x1  x2  2m   x1  2m   4m    2m ) x1 x2  m    4m  3  2m   m   16m  8m  12  6m  m   9m  22m  13    m  1 9m  13   m  1(tm) m      m  13 (tm) m  13    13 m  1; m  Vậy thỏa mãn điều kiện toán Câu 4: A H I B C O K a) CMR: điểm … Vì I tâm đường tròn ngoại tiếp tam giác ABC nên IC phân giác góc C Vì K tâm đường tròn ngoại tiếp tam giác ABC góc A nên CK phân giác ngồi góc C Theo tính chất phân giác ngồi tam giác ta có IC vng CK nên Chứng minh hồn tồn tương tự ta có: Xét tứ giác  BICK BICK ta có: IBK  900 IBK  ICK  900  900  1800 tứ giác nội tiếp (tổng hai góc đối diện 1800 ) ICK  900 Do O trung điểm IK nên theo tính chất trung tuyến ứng với cạnh huyền nửa cạnh huyền Vậy O OC  OI  OK tâm đường tròn ngoại tiếp tứ giác IBKC b) CMR: AC tiếp tuyến… OIC  OCI Ta có: Tam giác IOC cân O nên Mặt khác, theo tính chất góc ngồi tam giác ta có: 1 1 BAC  ACB  BAC  ABC 2 2 1 1  ICO  ICA  BAC  ABC  ACB  1800  900 2 2  OC  CA OIC  IAC  ACI  Do AC tiếp tuyến (O) C (đpcm) c) Tính tổng diện tích …… Gọi diện tích hình cần tính S, diện tích hình trịn (O) S’, gọi giao điểm BC IK M Ta có: S  S ' S ICKB   IO  S IBK  S IKC IK BM IK CM IK IK BC.IK      2 Ta có: S ABC   AB  BC  CA AM BC  IM 2 AB  BM 24   AB  BC  CA  IM  24   20    24   20.2  24  IM    IM  Áp dụng hệ thức lượng tam giác IBM vng B có đường cao BM ta có: BM 122   24 IM  IK  IM  MK   24  30 BM  IM IK  MK  1 1  S   IK  BC.IK   302  24.30 4  225  360  346,86  dvdt  Câu Điều kiện xác định: x 1 t  x   t  0  x  t  Đặt Phương trình trở thành:  t  1  2t    t  1  t  3t  3t   2t   t  t  4t  5t   t  t  4t     t  t    1     t   x 1   x  Vậy phương trình cho có nghiệm x 1 SỞ GIÁO DỤC – ĐÀO TẠO TỈNH THANH HÓA TRƯỜNG THPT CHUYÊN LAM SƠN KỲ THI TUYỂN SINH LỚP 10 THPT NĂM HỌC 2018-2019 Mơn thi: TỐN CHUN Thời gian: 150 phút ĐỀ THI CHÍNH THỨC Câu     1) Tính giá trị biểu thức P  1  1   1   1 1      2018     a3  3a  5a  17  2) Cho a, b số thực dương thỏa mãn biểu thức   b  3b  5b  11  Chứng minh a  b   Câu 1) Giải phương trình : x2  x   1  x  x  1 1  x2  y  2) Giải hệ phương trình :   x   y   xy   Câu 1) Tính tất cặp số nguyên dương  x; y  thỏa mãn: x2019  y 2019  y1346  y 673  2) Cho n số nguyên dương tùy ý, với số nguyên k ta đặt Sk  1k  2k   nk Chứng minh S2019 S1 Câu 4: Cho tam giác nhọn ABC có AB  AC Gọi D, E, F theo thứ tự chân đường cao kẻ từ A, B, C tam giác, P giao điểm đường BC EF Đường thẳng qua D song song với EF cắt cạnh AB, AC, CF Q, R, S 1) CMR: tứ giác BQCR tứ giác nội tiếp 2) Chứng minh PB DB với D trung điểm QS  PC DC 3) Khi B, C cố định A thay đổi thù chứng minh đường tròn ngoại tiếp tam giác PQR qua điểm cố định Câu Trong giải đấu thể thao có n độ tham dự n  , luật đấu sau: Hai đội đấu với trận Sau trận, đội thắng điểm, đội thua điểm hòa hai đội điểm Sau giải đấu đội xếp hạng théo thứ tự từ cao xuống thấp (bằng điểm xếp hạng) Hỏi điểm chênh lệch lớn đội xếp thứ hạng liền ? ĐÁP ÁN Câu 1) Tính giá trị biểu thức 2.3 Ta có:       2.3 3.4 1        3.4 2018.2019       2018 2018.2019      P  1  1       2.3  3.4   2018.2019  2.3  3.4  2018.2019   2.3 3.4 2018.2019 10 4074340  2.3 3.4 2018.2019 1.4 2.5 3.6 2016.2019 2017.2020  2.3 3.4 4.5 2017.2018 2018.2019 1.2 .2017   4.5 .2020   1.2020  2020  1010   2.3 .2018  3.4.5 .2019  2018.3 6054 3027 2)  a  13  2a  16  0(1) a  3a  5a  17    3 b  3b  5b  11   b  1  2b  12  0(2)     2018   1      a  1  2a  16   b  1  2b  12  3 2   a   b  1  a  1   a  1 b  1   b  1    a  b       a      a  b     b  1   b  1         a 1    a  b     b  1   b  1   0a, b        Vậy ta có điều phải chứng minh Câu 1) Giải phương trình x  x   1  x  x  Điều kiện xác định: x   a   x  a   Đặt   b  x   b    a  b2   x  x2  x   x2  x  Pt  a  b   2ab   a  b   1  x  x    1  x  x  a  b      a  b  1 1  x  x   2 3  x  x   ( x  1)  x  1(VN x  1)  3  x  x  x  1  x  1  x     3  x    x   x   x  x   x   x  x  10    x    Vậy phương trình cho có nghiệm x  1   x2 y  2) Giải hệ phương trình   x   y   xy    x2    x2     y 1  Điều kiện xác định :    y2   xy    xy  2   x, y   Hệ cho tương đương với 1  x  y  x y (1)  1 2 x  y   2  x   y   xy   x  y    x  1 y  1  xy  2(2)   (2)  x y   x y  x  y   xy   x y  xy    xy   xy     xy  1  2   x  y    x  y   1(ktm)  xy  2(tm)   xy   xy  1     xy   1( tm )    xy  2  2   x  y    x  y     xy   x  y     x  y  2    x  y    xy    x  y  2  Vậy hệ cho có nghiệm  x; y  thỏa mãn   2; ;  2;   Câu Bài Tính tất cặp số nguyên  x; y  thỏa mãn x2019  y 2019  y1346  y 673  Đặt : x673  a; y 673  b  a; b   Phương trình cho trở thành: a3  b3  b2  b  2(*)  a3  b3  3b2  3b   2b2  4b    b  1   2b2  4b  3   b  1 3 Lại có: a3  b3  6b2  12b   7b2  13b    b     7b2  13b     b   Từ (1) (2) ta có:  b  1  a3   b    b   a  b  3 a  b Vì a, b    a  b  +) Với a  b ta có: *  b3  b3  b2  b   b  b     b  1 b    b  a  b    b  2  a  b  2  x 673  y 673   x  y  1(tm)   673   673 673  x  y  2(ktm)  x  y  2 +)Với a  b   *   b  1  b3  b2  b   b3  3b  3b   b3  b  b   4b  4b    1  (ktm) b    1  (ktm) b   Vậy  x; y   1;1 Bài Ta có ngay: S1      n  n  n  1 n 1 Ta chứng minh S2019 chia hết cho n n 1 nguyên Sử dụng khai triển Newton ta có: a k 1  b2 k 1   a  b  a k  a k 1.b   b2 k (a  b) Giả sử n lẻ   Do vậy:      n    n  1   n        n  1    2.n 12019  22019   n2019   12019  n2019   22019   n  1  12019  22019   n2019   12019   n  1 2019   2 2019 2019 2019 2019 2019 2019 2019 Do  n; n  1  nên 12009  2209   n2009  chia hết cho n  n  1 Do S2019 S1 Vậy ta có điều phải chứng minh Câu A E F S P B D R C M Q 1) CMR: tứ giác BCQR tứ giác nội tiếp Do AB  AC nên Q nằm tia đối tia BA R nằm đoạn CA, từ Q, C nằm phía đường thẳng BR 2019 n Do tứ giác BFEC nội tiếp nên AFE  BCA (góc ngồi đỉnh góc đỉnh đối diện Vì QR / / EF  AFE  BQR (hai góc đồng vị)   BCA  BQR  AFE   BQCR tứ giác nội tiếp (hai đỉnh kề nhìn cạnh đối diện góc nhau) PB DB 2) Chứng minh với D trung điểm QS  PC DC Xét DHB EHA ta có: HDB  AEH  900 ; BHD  AHE (hai góc đối đỉnh) DB HB (các cặp cạnh tương ứng tỉ lệ)  DHB EHA( g  g )   AE HA Xét DHC FHA ta có: HDC  AFH  900 ; CHD  AHF (hai góc đối đỉnh) DC HC (các cặp cạnh tương ứng tỉ lệ)  DHC FHA( g  g )   AF HA HC DB AE HB AE FB  DC  AF    (1) HA DC AF HC AF EC Áp dụng định lý Menelaus cho tam giác ABC với cát tuyến PEF ta được: PB EC FA PB AE FB 1  (2) PC EA FB PC AF EC PB DB Từ (1) (2) ta :  (3)(dpcm) PC DC DQ BD DS CD Do QR song song với EF nên theo định lý Ta-let :   PF BP PF CP Kết hợp với (3) ta DQ  DS hay D trung điểm QS 3) Khi B, C cố định A thay đổi chứng minh đường trịn ngoại tiếp tam giác PQR qua điểm cố định Gọi M trung điểm BC Ta chứng minh DP.DM  DQ.DR Thật vậy, tứ giác BQCR nội tiếp  QBC  QRC (các góc nơi tiếp chắn cung QC )  QBD CRD( g  g )   DQ.DR  DB.DC (4) QD BD (các cặp cạnh tương ứng tỉ lệ)  CD RD  DC  DB  Tiếp theo ta chứng minh: DP.DM  DB.DC  DP.   DB.DC   DP. DC  DB   2DB.DC  DB. DP  DC   DC. DP  DB   DB.PC  DC.PB  PB DB (Đúng theo phần b) Do đó: DP.DM  DB.DC (5)  PC DC Từ (4) (5) ta DP.DM  DQ.DR  DP DQ  DR DM Xét DQP DRM ta có: DP DQ  (cmt ); PDQ  RDM (hai góc đối đỉnh) DR DM  DQR RMD(c  g  c)  PQR  RMP (hai góc tương ứng)  PQMR tứ giác nội tiếp (hai đỉnh kề nhìn cạnh đối diện góc nhau)  Đường trịn ngoại tiếp tam giác PQR qua trung điểm BC (đpcm) Câu Đội đứng thứ có điểm cao A   n  1 điểm (Đội đấu n  trận với đội lại thắng) Xét n  đội cịn lại ta có: Đội đứng số n  đội cịn lại có số điểm nhỏ xác định sau:  n  1 n   Gọi P tổng điểm n  đội đấu với nhau, số trận n  đội lại:  P   n  1 n  2 (Vì đội thắng hay hịa sau trận có điểm) Gọi Q số điểm đội n  đội lại  Q  n  1   n  1 n    Q  n   A  Q   n  1   n    n Vậy chênh lệch điểm số lớn đội xếp hạng liên n điểm SỞ GIÁO DỤC VÀ ĐÀO TẠO HƯNG YÊN KỲ THI TUYỂN SINH VÀO LỚP 10 THPT CHUN NĂM HỌC 2018-2019 Mơn thi : TỐN ĐỀ CHÍNH THỨC (Dành cho thí sinh dự lớp chuyên: Toán, Tin) Thời gian làm bài: 150 phút x 1 1 Câu (2 điểm) Cho biểu thức A  B  x4  5x2  8x  2025 với : x x  x  x x  x x  0, x  a) Rút gọn biểu thức A b) Tìm giá trị x để biểu thức T  B  A2 đạt giá trị nhỏ Câu (2 điểm) a) Tìm giá trị m để đồ thị hàm số y  x y  x  m cắt hai điểm phân 8 biệt A  x1; y1  , B  x2 ; y2  cho  x1  x2    y1  y2   162 b) Tìm giá trị nguyên x để M  x4   x  1  x2  x số phương Câu (2 điểm) a) Giải phương trình x3  108x  45  x 48x  20  3x2  x  y  x  y   x  1 y  1  b) Giải hệ phương trình  x 2  y 2     1  y    x   Câu (3 điểm) Cho đường tròn (O;R) đường thẳng d khơng có điểm chung với đường tròn Trên d lấy điểm M bất kỳ, qua M kẻ tiếp tuyến MA, MB với đường tròn (O) (A, B tiếp điểm) Kẻ đường kính AC đường trịn (O) Tiếp tuyến đường tròn (O) cắt đường thẳng AB E a) Chứng minh BE.MB  BC.OB b) Gọi N giao điểm CM với OE Chứng minh đường thẳng qua trung điểm đoạn thẳng OM CE vng góc với đường thẳng BN c) Tìm giá trị nhỏ dây AB M di chuyển đường thẳng d, biết R  8cm khoảng cách từ O đến đường thẳng d 10 cm Câu (1 điểm) Cho a, b hai số thay đổi thỏa mãn điều kiện a  a  b  Tìm giá trị nhỏ biểu thức A  8a  b b 4a ĐÁP ÁN Câu a) Điều kiện x  0; x  A   x 1 1 :  x x  x  x x  x  x 1  x x  x  x 1 x  x 1    x 1  x x  x 1  x2  x   x x x 1    x 1 x  x 1   x 1  x 1  x 1 b) Ta có: T  B  A2  x  x  x  2025   x  1  x  x  x  2025  x  x   x  x  x  2023  x  x  16  x  x   2003   x     x    2023 2 Vì  x    0,  x     T  2003 2  x   x2    Dấu “=” xảy      x  2  x  x   x   Vậy với Tmin  2003  x  Câu a) Phương trình hồnh độ giao điểm hai đồ thị là: x2  x  m  x  x  m  0(*) Hai đồ thị hàm số cắt hai điểm phân biệt  * có hai nghiệm phân biệt      4m   m  Gọi x1 , x2 hai nghiệm phương trình (*) Khi ta có: y1  x1  m,  x1  x2   x1 x2  m Áp dụng hệ thức Vi ét ta có:  y2  x2  m Theo đề ta có:  x1  x2    y1  y2   162 8   x1  x2    x1  m  x2  m   162 8   x1  x2    x1  x2   162 8   x1  x2   81   3 x  x   x  x2     x1  x2    x1  x2  +) Với x1  x2   x2    x2   x1 x2  m  1 1   (tm) 2 +)Với x1  x2   x2    x2   x1 x2  m  1 1  x1  2 1 1  x1  2 1 1   (tm) 2 2 Vậy m   thỏa mãn điều kiện tốn b) Ta có: M  x4   x  1  x  x M  x  x  3x  3x   x  x M  x  x3  x  x   4M  x  x3  x  x  +) Ta có:  2x  x   x  x  x  x  x  x  x   x    x  x  x  x   4M 2 Ta thấy dấu "  " xảy nên  x  x   4M (1) +) Với x   4M   M   M số phương Với x   4M  20  M   M không số phương Với x   4M  124  M  31  M không số phương  x 1  x  Với x  0;1;2 ta có:     x  1     x  1  x    x     Ta có: 2 4M  x  x3  x  x   x  x3  x  x   x  x    x  x  1   x  1  2  4M   x  x  1 (2) Từ (1) (2)   x  1  4M   x  x  1 Mà x   4M   x  x  1 2  x 1  x    x  1      x   2  x  1 Vậy có giá trị nguyên x thỏa mãn yêu cầu toán x  0; x  1 ; x  Câu Câu a Điều kiện: x   12 x  108 x  45  x 48 x  20  x  x3  12 x   x 12 x   x  x3  x  x 12 x   12 x   x (2 x  3)  12 x   x  3     x  3 x  12 x    2 x   x (ktm)      x  12 x   x  12 x  (1) 1  x  x   x  12 x    x     x  3 2 x  1  x2   x   x2  x       x    x   2 x   x  x   0(VN ) Vậy nghiệm phương trình x   b) Điều kiện: x  1; y  1 y  x  x  y  x  y   x  1 y  1  x( x  1)  y ( y  1)  ( x  1)( y  1)  y 1  x 1       x 2  y 2   x   y 2 2     x    y             1  y    x    y    x    y    x    Đặt a  x y ;b  Khi hệ phương trình trở thành: y 1 x 1 b   a a  b  b   a b   a     a   2 a  b  2a  2a   2a(a  1)  a    x   y     a    y  x        x  b  y 1     (tm)  a   x   x     y   b      y    y   x    Vậy nghiệm hệ phương trình ( x; y)  (1;0) ( x; y)  (0;1) Câu d M E B P A H Q N O C a) Xét tứ giác OAMB có OAM  OBM  900  900  1800  Tứ giác OAMB tứ giác nội tiếp  OAB  OMB (hai góc nội tiếp chắn cung OB) Mà OAB  OMB (cùng phụ với ACB)  OMB  BCE Xét tam giác OMB tam giác ECB có: OBM  EBC  900 ; OMB  BCE (cmt )  OMB  ECB( g.g ) BE BC   BE.MB  BC.OB (dpcm) OB MB b) Gọi P Q trung điểm OM CE OMB ECB (cmt )  CEB  MOB Xét tam giác EAC tam giác OMA có: ECA  OMA  900 ; CEA  CEB  MOB  MOA  EAC OMA( g.g )  EC AC EC AC    OA AM OC AM Xét tam giác COE tam giác AMC có OCE  CAM  900 CE AC  (cmt )  COE CO AM ACM (c.g.c)  AMC  COE (hai góc tương ứng) Mà COE  NOA  1800  AMC  NOA  1800  Tứ giác OAMN tứ giác nội tiếp (Tứ giác có tổng hai góc đối 180 )  ONM  1800  OAM  900  OMN vuông N  NP  OM (trung tuyến ứng với cạnh huyền tam giác vuông)  NP  BP  OM  P thuộc trung trực đoạn thẳng BN Chứng minh tương tự ta có : NQ  BQ  EC  Q thuộc trung trực đoạn thẳng BN Vậy PQ trung trực đoạn thẳng BN  PQ  BN c) Gọi H  AB  OM ta có OH  AB  ABmin  OH max Áp dụng hệ thức lượng tam giác vng OAM có: R2 OH OM  OA  R  OH   OH max  OM  M hình chiếu vuong góc O OM 82 đường thẳng d  OM  d  O; d   10  OH   6, (cm) 10 2 Xét tam giác vuông OAH có AH  82  6,42  4,8(cm)  AB  AH  9,6(cm) Vậy dây AM nhỏ 9,6cm Câu 5: Theo giả thiết ta có: a  b   b   a 8a   a 1 1  A  b  2a   b  a  a  b2  4a 4a 4a 1 1 a  a  a  2a    a   a2  a   a   a2  a   4a 4a 4a 2  1 1  a   a     1  4a  2 2 Co  si 1  ;a   a  Dấu xảy   ab 4a 2  b   a Vậy MinA   a  b  2 (tm) SỞ GIÁO DỤC VÀ ĐÀO TẠO HƯNG YÊN ĐỀ CHÍNH THỨC KỲ THI TUYỂN SINH VÀO LỚP 10 THPT CHUN NĂM HỌC 2018-2019 MƠN THI : TỐN Dành cho thí sinh chun: Tốn, Tin, Lý, Hóa, Sinh Câu (1,0 điểm)   a) Rút gọn biểu thức A  2    b) Tìm m để đường thẳng y  x  m2  đường thẳng y   m  2 x  11 cắt điểm trục tung Câu (2,0 điểm) x  y  m  (1) (m tham số) 2 x  y  m Cho hệ phương trình  a) Giải hệ phương trình (1) m  b) Tìm m để hệ (1) có nghiệm  x; y  cho P  98  x2  y   4m đạt giá trị nhỏ Câu (2,0 điểm) a) Giải phương trình: x    x   x  x2  b) Tìm m để phương trình x4  5x2   m  (m tham số) có hai nghiệm Câu (1,0 điểm) Quãng đường AB dài 120 km Một ô tô chạy từ A đến B với vận tốc xác định Khi từ B trở A, ô tô chạy với vận tốc nhỏ vận tốc lúc từ A đến B 10 km/h Tính vận tốc lúc ô tô, biết thời gian nhiều thời gian 24 phút Câu (3,0 điểm) Cho ba điểm A, B, C cố định thẳng hàng theo thứ tự Vẽ đường trịn (O;R) qua B C (BC < 2R) Từ A kẻ tiếp tuyến AM, AN với đường tròn (O) (M, N tiếp điểm) Gọi I trung điểm BC a) Chứng minh năm điểm A, M, I, O, N thuộc đường tròn b) Gọi J tâm đường tròn nội tiếp tam giác MBC, E giao điểm thứ hai đường thẳng MJ với đường tròn (O) Chứng minh EB  EC  EJ c) Khi đường tròn (O) thay đổi, gọi K giao điểm OA MN Chứng minh tâm đường trịn ngoại tiếp tam giác OIK ln thuộc đường thẳng cố định Câu (1,0 điểm) Cho số dương x, y, z thỏa mãn xy  yz  zx  3xyz Chứng minh x3 y3 z3 1 1        2 zx x y y z  x y z Câu a) A      1    1  2  2    3.1     1  2   1 1  b) Hai đường thẳng cắt a  a '   m   m  Giả sử hai đồ thị cắt điểm A  Oy  A  0; y A  Phương trình hồnh độ giao điểm hai đồ thị cho : x  m    m   x  11   m  3 x  m    m  3 x   m  3 m  3 (*) Hai đồ thị cắt A nên x  nghiệm phương trình (*)  0.(m  3)   m  3 (m  3)  (m  3)(m  3)  m   m    m    m  3 Với m  (loại) đường thẳng trùng Vậy với m  3 hai đường thẳng cắt điểm trục tung Câu 2: a) Thay giá trị m  1vào hệ phương trình ta có: x  y  x   2 x  y   y  I    Vậy với m  1thì hệ phương trình có nghiệm  x; y    2;1    I  ln có nghiệm (x;y) với m 3  x  y  2m  x  m   y  I    2 x  y  m 7 y  m  5m   x  x  m   y     m6  y  y  m   Theo đề ta có: P  98 x  y  4m b) Ta có     5m    m     P  98.    4m   49 49    2(26m  102m  117)  4m  52m  208m  234  52  m  4m    234  52.2  52  m    26  26  MinP  26 Dấu “=” xảy  m    m  2 Vậy m  2 thỏa mãn yêu cầu toán Câu x     3  x  a) Điều kiện : 2  x  6  x  x    x  3  x   1(*) Đặt x    x  t  t  0 t  x    x   x  3  x     x  3  x  Pt  x    x    x  3  x   t2  t2   (*)  t  1  2t  t     t  2t     t  1 t  3  t   t  1(ktm)   t   t  (tm) 32    x  3  x   2   x  x2   x2  x     x  1 x    x 1   x  (tm)   x    x  2 (tm) Vậy phương trình có tập nghiệm S  2;1 b) x  5x   m  0(*) Đặt x2  t (t  0) Phương trình cho  t  5t   m  0(1) Để phương trình (*) có hai nghiệm phương trình (1) phải có nghiệm dương  (1) phải có hai nghiệm trái dấu hai nghiệm kép dương 6  m   ac    5  4(6  m)    m         m6   x1 x2  VN 6  m      5    x1  x2  Vậy m  thỏa mãn yêu cầu toán Câu ( x  0) Gọi vận tốc lúc ô tô x(km / h) Khi vận tốc lúc tơ : x  10 (km / h) Thời gian thời gian ô tô hết quãng đường AB là: 120 120 (h); ( h) x x  10 Đổi 24 phút  0, Theo đề ta có phương trình: 120 120   0, x x  10  120( x  10)  120 x  0, x( x  10)  0, x  x  1200   0,  x  50  ( x  60)   x  50 (tm)   x  60 (ktm) Vậy vận tốc lúc ô tô 50km / h Câu M J O K A B H I C N E a) Ta có OMA  ONA  900 ( gt )  OIA  900 (quan hệ vng góc đường kính dây cung)  Các điểm M, I, N nhìn OA góc 900 nên thuộc đường trịn đường kính OA Vậy điểm A, M , O, I , N thuộc đường trịn đường kính OA b) Ta có MJ phân giác BMC  BME  EMC  sd BE  sdCE  EB  EC (1) (hai cung căng hai dây nhau) Ta có: EBC  EMC  BME; CBJ  JBM ( gt )  EBJ  EBC  CBJ  BME  JBM Xét tam giác BMJ có BME  JBM  BJE (góc ngồi tam giác tổng hai góc khơng kề với nó)  EBJ  BJE  EBJ cân E  EB  EJ (2) Từ (1) (2)  EB  EC  EJ c) Gọi H giao điểm AC MN, ta có: OKH  900 (Do AM, AN hai tiếp tuyến cắt nên OA trung trực MN) AIO  900 (quan hệ vng góc đường kính dây cung) Xét AHK AOI có: AKH  AIO  900 ; OAI chung  AHK AOI ( g.g )  AH AK   AH AI  AO AK AO AI (3) Xét tam giác vng AMO có AO AK  AM (4) (hệ thức lượng tam giác vng) Ta có: AMB  ACM (góc tạo tiếp tuyến dây cung góc nội tiếp chắn cung BM) Xét tam giác AMB ACM có: MAC chung; AMB  ACM (cmt ) AM AB   AM  AB AC (5) AC AM AB AC Từ (3) (4) (5) suy AH AI  AB AC  AH  AI Ta có AB, AC, AI khơng đổi  AH không đổi Mà A cố đinh nên H cố định Gọi O ' tâm đường tròn ngoại tiếp tam giác OIK, tâm đường trịn ngoại tiếp tứ giác OIHK  O ' trung điểm OH  O ' thuộc trung trực HI Mà H ; I cố định  Trung trực HI cố định Vậy (O) thay đổi tâm đường trịn ngoại tiếp OIK ln chạy trung trực HI , với H  AC  MN  AMB ACM ( g.g )  Câu Theo đề ta có: xy  yz  zx  3xyz  xy yz zx 1   3   3 xyz xyz xyz z x y Cosi Lại có: 3xyz  xy  yz  xz  3  xyz   xyz   x  y  z  Ta có x3 xz Cosi xz z z 1  x   x  x  x 2 zx zx zx z 1 z z 1 z z 1     ) 2 2  y3 x 1  x  y3  y   Tương tự ta có:   z  z  y 1  y  z ( Do z 1  z  z  Cộng vế với vế bất đẳng thức ta được: x3 y3 z3 x y  z 3 11 1    x y z         (dpcm) 2 zx x y yz 4 2 x y z  SỞ GIÁO DỤC – ĐÀO TẠO TỈNH VĨNH PHÚC KỲ THI TUYỂN SINH LỚP 10 THPT NĂM HỌC 2018-2019 Mơn thi: TỐN CHUN Thời gian: 150 phút Ngày thi: 03/06/2018 ĐỀ THI CHÍNH THỨC Câu a) Trong mặt phẳng tọa độ Oxy , cho ( P) : y  x2 đường thẳng (d ) : 2mx  m  Tìm tất giá trị m để (d ) cắt (P) điểm phân biệt A( x1; y1 ); B( x2 ; y2 ) thỏa mãn x1  x2  y1 y2  b) Giải phương trình : x  x    x2  x 1  x2  y  c) Giải hệ phương trình :   x  y  xy  Câu Cho phương trình x  y3  z  9!(1) với x; y; z ẩn 9! Là tích số nguyên dương liên tiếp từ đến a) Chứng minh có số nguyên x; y; z thỏa mãn (1) x, y, z chia hết cho b) Chứng minh không tồn số nguyên x, y, z thỏa mãn (1) Câu Cho số thực dương a, b, c Chứng minh rằng: a2 b2 c2   1 a  ab  b2 b2  bc  c c  ac  a Câu Cho hình thoi ABCD (AC > BD) Đường tròn nội tiếp (O) tứ giác ABCD theo thứ tự tiếp xúc với cạnh AB, BC, CD, DA E, F, G, H Xét K đoạn HA L đoạn AE cho KL tiếp xúc với đường tròn (O) a) Chứng minh LOK  LBO BL.DK  OB2 b) Đường tròn ngoại tiếp tam giác CFL cắt AB M khác L đường tròn ngoại tiếp tam giác CKG cắt AD N khác K Chứng minh điểm K, L, M, N nằm đường tròn c) Lấy điểm P, Q tương ứng FC, CG cho LP song song với KQ Chứng minh KQ tiếp xúc với (O) Câu Một bảng hình vng gồm n hàng n cột (n nguyên dương) Các hàng cột đánh số từ đến n ( từ xuống dưới, từ trái qua phải) Ơ vng nằm hàng i, cột j  i; j  1;2;3; n  bảng gọi ô  i; j  Tại ô bảng điền số cho ô  i; j  điền số  b j  n , số số hàng I b j số số cột j Gọi P tổng số ô bảng hình vng cho a) Xây dựng bảng hình vng thỏa mãn u cầu tốn trường hợp n  P   n2   n2  n2 b) Chứng minh P    , với   phần nguyên 2 2 ĐÁP ÁN Câu a) Ta có: phương trình hồnh độ giao điểm : x  2mx  m   x  2mx  m     4m2   4m  1   2m  1   m   x1  x2  2m  x1 x2  m  Theo định lý Vi-et ta có:   x1  x2  y1 y2   4m   x1 x2    4m  m  2m     m  1   m  1 Vậy giá trị cần tìm m  1 b) Giải phương trình: x  x    x2  x  Ta có điều kiện xác định : x  x     x    x  3 2 x    x  x     3  2  x   2 Ta có: x  x    x2  x 1  x  x   x( x  4)   x  x   x2  4x   x  x  4    x2  x   x2  x     x2  4x    x2  4x     x  x   Do x2  4x     x2  4x    x   13   x   13 (tm) (ktm) Vậy phương trình có nghiệm x   13  x2  y  c) Giải hệ phương trình   x  y  xy  u  x  y  DK : u  4v  v  xy  Đặt  u  2v  25  10v  v  2v  v  12v  20     u  v  u   v u   v  u  (tm)  v  10  v   x  1; y      v      u  5  x  2; y  u   v  ( ktm )    v  10 Vậy tập nghiệm hệ cho 1;2  ;  2;1 Câu a) Chứng minh rằng… Ta có: 9!  1.2.3.4.5.6.7.8.9 số chẵn  x3  x  x  2m  m   8m3  y3  z  9!  4m3  y3  z  1.3.4.5.6.7.8.9 số chẵn  y  y  y  2n n    4m3  8n3  z  1.3.4.5.6.7.8.9  2m3  4n3  z  1.2.3.5.6.7.8.9 số chẵn  z3  z  z  p  p    2m3  4n3  p3  1.2.3.5.6.7.8.9  m3  2n3  p3  1.3.5.6.7.8.9 m  Chứng minh hoàn toàn tương tự ta có n  m; n; p  p  Vậy ta có điều phải chứng minh b) Chứng minh không tồn tại… Theo ý a) ta đặt x  4a; y  4b; z  4c  a3  2b3  4c3   a  a  2u  x  2m    y  2n z  p   a; b; c   9! 1.2.3.4.5.6.7.8.9   1.3.5.6.7.9 số chẵn 43 43 u    8u3  2b3  4c3  1.3.5.6.7.9  4u  b3  2c3  1.3.3.5.7.9  1.5.7.34 Lại có:  1.5.7.34  34  1.5.7.34   (mod 9)  x  Z   x  0; 1  a; b; c   4u  b3  2c3  93 Nhưng 1.5.7.34 chia hết cho 93 nên ta có điều vơ lý Vậy ta có điều phải chứng minh Câu Ta có : Áp dụng bất đẳng thức Cauchy – Schaw thì: a  c a2 c2   2 2 a  ab  b c  a  b  c  a  b  c  ab  ac  bc b  c b2 c2   2 b(a  b  c) c  ac  a a  b  c  ab  ac  bc a  b a2 b2   a (a  b  c) b  bc  c a  b  c  ab  ac  bc a2 b2 c2 a2 b2 c2       a  ab  b b  bc  c c  ac  a a  a  b  c  b(a  b  c ) c (a  b  c ) a  c   b  c    a  b a  b  c  ab  ac  bc 2 a2 b2 c2 a  b  c  a  c   b  c    a  b       a  ab  b b  bc  c c  ac  a a  b  c a  b  c  ab  ac  bc  a  b  c  ab  ac  bc  a2 c2 b2    1  a  ab  b c  ac  a b  bc  c a  b  c  ab  ac  bc a2 c2 b2    1  a  ab  b c  ac  a b  bc  c a2 c2 b2     (dpcm) a  ab  b c  ac  a b  bc  c 2 Dấu xảy a=b=c 2 Câu a) Chứng minh… Gọi điểm tiếp xúc LK với (O) T Ta có: LOK  LOT  TOK  TOE TDH EOH    900  EOB  LBO 2 OLK  OLB (do LB, LK tiếp tuyến) Khi đó: tam giác OLK BLO đồng dạng Chứng minh hoàn tồn tương tự ta có: OLK DOK ( g.g )  DOK BLO OD DK    BL.DK  BO.DO  OB BL BO Vậy ta có điều phải chứng minh b) Đường tròn ngoại tiếp … Ta có : Do CFLM nội tiếp nên BM BL  BC.BF  BO2  BL.DK  BM  DK Do BMDK hình thang cân nên KM // BD Chứng minh hồn tồn tương tự ta có: LN//BD Do KMLN hình thang cân nên hiểu nhiên nội tiếp đường trịn Ta có điều phải chứng minh c) Lấy điểm P, Q… Ta có: Kẻ PQ ' tiếp xúc với (O) Q’ thuộc CD Tương tự phần a, chứng minh ta có: BP.DQ '  OB  BL.DK  BP DK  BL DQ ' LBP  KDQ '  BLP DQ ' K (c.g.c)  BLP  DQ ' K  AB / / CD  LP / / KQ '  Q  Q ' Vậy KQ tiếp xúc với (O) Câu a) Xây dựng … Ta có bảng thỏa mãn toán: 1 1 1 1 b) Chứng minh rằng… Khơng tính tổng qt, gọi cột hàng có số cột Giả sử cột có k số  k  n  Gọi hàng i hàng loại  i;1  Gọi hàng j hàng loại  j;1  Vậy có k hàng loại n  k hàng loại Khi tổng số hàng loại  k loại  n  k Như vậy: P  k   n  k  Áp dụng bất đẳng thức Cơ si ta có: P  k  n  k  2 k  n  k   2  n2  n2     Vậy ta có điều phải chứng minh SỞ GIÁO DỤC – ĐÀO TẠO TỈNH BẾN TRE KỲ THI TUYỂN SINH LỚP 10 THPT NĂM HỌC 2018-2019 Mơn thi: TỐN CHUN Thời gian: 150 phút ĐỀ THI CHÍNH THỨC a b  a b a  b Câu 1: Cho biểu thức P  với a, b hai số thực dương  ab a) Rút gọn biểu thức P : a  b a  b   b) Tính giá trị biểu thức P a  2019  2018 b  2020  2019 Câu 2: a) Cho p số nguyên tố lớn Chứng minh p  chia hết cho 24 b) Cho phương trình x2  2mx  m   với m tham số Tìm giá trị m để phương trình cho có hai nghiệm phân biệt x1; x2 thỏa đạt giá trị x  x22 lớn Câu a) Giải phương trình: x3   x  3x   x2  y   b) Giải hệ phương trình:    x  y 1  xy   Câu a) Tìm nghiệm nguyên phương trình x3  xy   x  y b) Cho hai số thực dương a, b thỏa a  b  Tìm giá trị nhỏ biểu thức T  a b Câu 5: Cho nửa đường trịn (O;R) có đường kính AB Vẽ đường thẳng d tiếp tuyến  O  B Trên cung AB lấy điểm M tùy ý (M khác A, B), tia AM cắt đường thẳng d N Gọi C trung điểm đoạn thẳng AM, tia CO cắt đường thẳng d D a) Chứng minh tứ giác OBNC nội tiếp b) Gọi E hình chiếu N đoạn AD Chứng minh N, O, E thẳng NE AD  2R ND  CO.CD c) Chứng minh CACN d) Xác định vị trí điểm M để 2AM  AN đạt giá trị lớn hàng ĐÁP ÁN Câu 1: a) Rút gọn biểu thức P :  Điều kiện : a  0, b  ab P  P:       a b   ab  a  b a  b a b  a b  P   a  b a  b   a  b  ab  ab  a b  a  b a  b  a  b  a  b    a  b  a  b   a  b b) Tính giá trị biểu thức… a  a  2019  2018    Ta có: b  2020  2019 b      P  a  b  2018    2019  1 2018  2   a  2018    b  2019   2019   2018  2019 Câu 2: a) Cho p số nguyên tố… Ta có nhận xét sau: Nếu p số nguyên tố lớn p  1(mod 24) (2) Lại có: 1  23 mod 24 Cộng vế theo vế 1 ;   ta : p   24(mod 24)   mod 24  Vậy p  chia hết cho 24 với p số nguyên tố lớn b) Cho phương trình…… Phương trình cho có nghiệm phân biệt   '  (1)  15   m  m     m      m  2  Vậy phương trình cho có hai nghiệm phân biệt x1; x2 với m  x1  x2  2m  x1 x2  m  Áp dụng hệ thức Vi et ta có:  Theo đề ta có 1    2 x  x2  x1  x2   x1 x2 4m  2m  1  31   2m    2    31 31 1 1  31 31         2m  Ta có: 4m  2m    2m2  2m    2 2     1   4m  2m   31 31   2m    2  1 Dấu “=” xảy  2m  0m 2 Vậy Max 2  m   x1  x2 31  Câu a) Giải phương trình :  x  1     13  13  x  3  13  x   x   x   2 Điều kiện          13  13  x  3x    x    x   13    x  2     x3   x  3x    x  1  x2  x  1  x2  x    x  1 Đặt a  x  1; b  x2  x  1(a  0; b  0) Khi ta có phương trình:  b  2a  ab  2a  ab  b    a  b  2a  b   a  b    2a  b  (ktm a  0; b  0)  2a  b  x   x  x   x   x2  x   x2  5x   Có   52  3.4  37   phương trình có hai nghiệm:   37 (tm) x     37 (tm) x    37  37 Vậy phương trình có hai nghiệm phân biệt: x  x  2  x  y  b) Giải hệ phương trình:   x  y 1  xy   2  x  y   x  xy  y  1  xy   x  y   2.1  xy     x  y  xy      x  y 1  xy     x  y  1  xy   a  x  y Khi ta có hệ phương trình tương đương: b   xy Đặt   a2 b   a  2b a     22   b  ab  a a   x  x   y x   y x  y        1  2  y y  y   y  y    1  xy      1 Vậy hệ phương trình có nghiệm  x; y   1;    Câu a) Tìm nghiệm nguyên… x3  xy   x  y  x3  xy  x  y   x  x  1  y  x  1  2   x  1  x  x  y   2   x   2   x  3  (tm)  y  11  x  x  y     x    x    (tm)   x  x  y  1  y     x     x  (tm)     y  x  x  y        x   1   x  2    y  (tm)   x  x  y   Vì x, y  Vậy hệ phương trình có nghiệm ngun  x; y    3;11 ; 1;1 ;  0;2  ;  2;4  b) Cho hai số thực dương…… Ta có: T  4(a  b) a  b 4b a 4b a     5   5  5  a b a b a b a b Áp dụng bất đẳng thức Cosi cho hai số thực dương ta có: T  5 4b a   5  a b 4b a 4b a  2 4 a b a b  a  (tm)  4b a 2  a  b   a  4b     Dấu “=” xảy   a b     a  2b a  b   b  (tm) a  b    Vậy MinT  a  ;b  3 Câu N M C B O A E D a) Chứng minh tứ giác OBNC nội tiếp Ta có: C trung điểm đoạn AM  OC  AM  C hay OCM  900 (mối quan hệ đường kính dây cung) Có: AB  BN  B hay OBN  900 (d tiếp tuyến đường trịn B) Xét tứ giác OBNC ta có: OCN  OBN  1800  OBNC tứ giác nội tiếp b) Gọi E hình chiếu… Xét ADN ta có: AB, DO hai đường cao tam giác Mà AB  CD  O  O trực tâm AND Lại có NE đường cao cịn lại AND nên ba điểm N , O, E thẳng hàng (đpcm) Ta có: S AND  1 NE AD AB.ND  NE AD  AB.ND  NE AD  AB   2R 2 ND (dpcm) c) Chứng minh rằng… Ta có: CAO  MBN (góc nội tiếp góc tạo tiếp tuyến dây cung chắn cung BM) Lại có: MB / /CD   CM   NBM  CDN (hai góc đồng vị)   CAO  CDN  MBN  Xét CAO CDN ta có: CAO  CDN (cmt ); ACO  NCD  900  CAO  CDN ( g.g ) CA CO   CA.CN  CD.CO (dpcm) CD CN d) Xác định… Áp dụng hệ thức lượng ABN vng B có đường cao MB ta có: AM AN  AB   2R   4R 2 Áp dụng bất đẳng thức Co si ta có: AM  AN  2 AM AN  8R2  2R (không đổi) AN Vậy Min(2 AM  AN )   AM   M điểm cung AB SỞ GIÁO DỤC VÀ ĐÀO TẠO ĐẮK LẮK KÌ THI TUYỂN SINH VÀO LỚP 10 TRUNG HỌC PHỔ THƠNG NĂM HỌC 2018 – 2019 ĐỀ CHÍNH THỨC Mơn thi: TỐN – CHUN Thời gian làm bài: 150 phút, không kể thời gian phát đề Câu (2,0 điểm) Cho đa thức f ( x)  x3  x2  (1  m) x  m 1) Khi m  , phân tích đa thức f ( x) thành nhân tử 2) Tìm tất giá trị tham số m để phương trình f ( x)  có ba nghiệm phân biệt x1 , x2 , x3 thỏa mãn x12  x22  x32  Câu (2,0 điểm) 15 ( x  1)2  15 x  1) Giải phương trình:  x  6x  x( x  x  4) 2  (2 x  y)( x  y )  x  x  xy  y 2) Giải hệ phương trình:  2   3( x  y )   x  y  14   x  x Câu (2,0 điểm) (1) (2) 1) Truyện kể hoàng tử cứu cơng chúa gặp rắn có 100 đầu Hồng tử có hai kiếm: Thanh kiếm cho phép chặt 21 đầu rắn Thanh kiếm cho phép chặt đầu rắn rắn lại mọc thêm 2018 đầu khác Biết rắn có 21 đầu đầu hồng tử không dùng kiếm kiếm tương ứng hồng tử cứu cơng chúa rắn bị chặt hết đầu Hỏi hoàng tử có cứu cơng chúa khơng? 2) Tìm số nguyên x, y, z thỏa mãn đồng thời: x2  y  z  xz  4( x  z )  396 x  y  3z Câu (1,0 điểm) 1) Cho số thực x, y không âm, chứng minh x3  y3  x2 y  xy 2) Cho số thực dương a, b, c thỏa mãn abc  Chứng minh rằng: ab bc ca  5   5 a  b  ab b  c  bc c  a5  ca Câu (3,0 điểm) 1) Cho tam giác ABC có ba góc nhọn  ABIE , DI cắt CE N Chứng minh NI.ND=NE.NC c) Gọi M giao điểm EF với IC Chứng minh MN vng góc với CH 2) Biết đường chéo ngũ giác lồi ABCDE cắt khỏi tam giác có diện tích Tính diện tích ngũ giác ABCDE Hết -Thí sinh khơng sử dụng tài liệu Giám thị khơng giải thích thêm Họ tên thí sinh: Số báo danh: Chữ kí giám thị 1: Chữ kí giám thị 2: SỞ GIÁO DỤC VÀ ĐÀO TẠO ĐẮK LẮK KÌ THI TUYỂN SINH VÀO LỚP 10 TRUNG HỌC PHỔ THÔNG NĂM HỌC 2018 – 2019 Mơn thi: TỐN – CHUN ĐÁP ÁN, BIỂU ĐIỂM VÀ HƯỚNG DẪN CHẤM ĐỀ THI CHÍNH THỨC (Đáp án, biểu điểm hướng dẫn chấm gồm tất 04 trang) A ĐÁP ÁN VÀ BIỂU ĐIỂM Câu Đáp án Cho đa thức f ( x)  x  x  (1  m) x  m 1) Khi m  , phân tích đa thức f ( x) thành nhân tử Điểm f  x   x3  x  x  0,25 f ( x)  ( x 1)( x  1)( x  2) 2) Tìm tất giá trị tham số m để phương trình f ( x)  có ba nghiệm phân biệt x1 , x2 , x3 thỏa mãn x12  x22  x32  x  Phân tích phương trình ( x  1)( x  x  m)    Câu  x  x  m  (*) (2,0 Phương trình f ( x)  có nghiệm phân biệt điểm)  Phương trình (*) có hai nghiệm phân biệt khác m  m     m     4m    Lúc đó: x1  1, x2  x3  1; x2 x3  m 0,25 0,25 0,25 0,25 0,25 Điều kiện: x  x  x    x2  x3   x2 x3   m  0,25 Vậy   m  1, m  0,25 2 2 15 ( x  1)2  15 x  1) Giải phương trình:  x  6x  x( x  x  4) Điều kiện: x  0; x   5; x   * Phương trình biến đổi thành: 1   x  x  x  x  15 x 1   (1) 4 15 x 6 x 2 Câu x x (2,0 điểm) Đặt x   t  t  2; t   x t  4 1    PT (1) trở thành: t  t  15 t  12 Với t  4 ta có x   4  x  2 thỏa mãn (*) x x   4 Với t  12 ta có x   12   thỏa mãn (*) x  x    0,25 0,25 0,25 0,25 2  (2 x  y)( x  y )  x  x  xy  y 2) Giải hệ phương trình:  2   3( x  y )   x  y  14   x  x Phương trình (1): (2 x  y)( x2  y  x  3)   2x  y Thế vào (2): 3x  x   5x  10 x  14   x  x Đánh giá vế trái (*): (1) (2) * 3( x  1)2   5( x  1)2   Và đánh giá vế phải (*):  x  x2   ( x  1)2  Dấu xảy x  1 Vậy hệ phương trình cho có nghiệm ( x; y)  (1; 2) 1) Giả sử rắn có n đầu (n số nguyên dương) Nếu dùng kiếm kiếm số đầu rắn sau bị chặt n  21 n  2009 Tức giảm tăng đại lượng bội số Mà 100 chia dư nên hoàng tử cứu công chúa 0,25 0,25 0,25 0,25 0,50 0,50 2) Tìm số nguyên x, y, z thỏa mãn đồng thời: x2  y  z  xz  4( x  z )  396 x  y  3z Từ điều kiện x  y  3z suy x  y chia hết cho hay x, y chia hết cho x2  y  z  xz  4( x  z )  396  ( x  z  2)2  4(100  y ) 2 Câu Suy ra: 100  y số phương y  100 Mặt khác y nên (2,0 y  0;36  y 0;6; 6 điểm)   x2 x2   x  3z z  z  Xét y  :    3 x  z   400        x  z   20  x  z   20 Tìm x  6, z  12 x  9, z  27   x  36  3z Xét y  y  6 :    x  z    256   x2 x2  z   12  z   12   3  x  z   16  x  z   16   0,25 0,25 0,25 0,25 Giải x, z  Vậy  x; y; z   6;0;12   9;0; 27  1) Cho số thực x, y không âm, chứng minh x3  y3  x2 y  xy Bất đẳng thức: x3  y3  x2 y  xy  x ( x  y)  y ( x  y)  Câu  ( x  y)2 ( x  y)  , x, y  (1,0 2) Cho số thực dương a, b, c thỏa mãn abc  Chứng minh rằng: điểm) ab bc ca  5   5 a  b  ab b  c  bc c  a5  ca Chứng minh a5  b5  a 2b3  a3b2  a3 (a  b2 )  b3 (a  b2 )   (a  b)2 (a  b)(a  ab  b2 )  0, a, b  (*) 0,25 0,25 Áp dụng (*): a5  b5  a 2b2 (a  b)  a5  b5  ab  ab abc c ab c  (1) 5 a  b  ab a  b  c bc a ca b Tương tự 5  (2) ;  b  c  bc a  b  c c  a  ca a  b  c Cộng (1), (2), (3) ta bất đẳng thức cần chứng minh Dấu xảy a  b  c 1) 0,25  (3) 0,25 0,25 Câu (3,0 a) Góc HDC  AEB  90 nên tứ giác DHEC nội tiếp đường trịn đường điểm) kính HC Tâm O trung điểm HC 0,25 b) Xét NIC NED ta có: END  INC (đối đỉnh); DEN  CIN (cùng chắn cung CD ) Suy ra: NIC ∽ NED NI NE    NI ND  NC.NE NC ND c) DIC  DHC (cùng chắn cung CD ) (1) DHC  ABC (cùng phụ góc BCF ) (2) Lại có: BFC  BEC  900 nên tứ giác BFEC nội tiếp, suy ABC  AEF (3) Mà AEF  MEC (đối đỉnh), từ MEC  DIC tứ giác MENI nội tiếp, suy EMN  EIN (4) 0,25 0,25 0,25 0,25 0,25 ACB  EIN (cùng chắn cung DE ) (5) ACB  AFE (tứ giác BFEC nội tiếp) (6) Suy AFE  EMN  AB / / MN Mà AB  CH nên MN  CH 0,25 2) 0,25 Các diện tích SABC  SABE nên C E cách AB hay AB // CE Tương tự đường chéo lại song song với cạnh tương ứng Gọi P giao điểm BD CE đặt diện tích SBCP  x  Do tứ giác ABPE hình bình hành nên SBPE  SABE  Lại có: SBCP BP SBEP x 1 , tức là:    x 1 SPCD PD SPED 1 x x   Diện tích ngũ giác: SABCDE  SABE  SBPE  SCDE  SBCP   x Vậy: SABCDE  5   0,25 0,25 0,25 B HƯỚNG DẪN CHẤM Điểm thi đánh giá theo thang điểm từ đến 10 Điểm thi tổng điểm thành phần khơng làm trịn Học sinh giải theo cách khác hợp lí cho điểm tối đa phần Hết SỞ GIÁO DỤC VÀ ĐÀO TẠO ĐẮK LẮK KÌ THI TUYỂN SINH VÀO LỚP 10 TRUNG HỌC PHỔ THÔNG NĂM HỌC 2018 – 2019 PHIẾU CHẤM BÀI THI Môn thi: TOÁN – CHUYÊN (Dùng cho lần chấm thứ nhất) Túi số: Phách số: Thang Câu Đáp án điểm 1) f  x   x3  x2  x  0,25 f ( x)  ( x  1)( x  1)( x  2) x  2)    x  x  m  (*) Phương trình (*) có hai nghiệm phân biệt khác 1  m  m   Lúc đó: x1  1, x2  x3  1; x2 x3  m 2,00 1    x  x  x  x  15 x 1    (1) 4 15 x 6 x 2 x x Đặt x   t  t  2; t   x t  4 1    (1): t  t  15 t  12 t  4  x  2 t  12  x   2) Phương trình  2x  y  x  x2   ( x  1)2  Dấu xảy x  1 ( x; y)  (1; 2) Tổng điểm câu 1) Số đầu rắn sau bị chặt n – 21 n + 2009 Mà 100 chia dư nên hồng tử khơng thể cứu công chúa 2) x, y chia hết cho ( x  z  2)2  4(100  y )  y  0;6; 6 y   x  6, z  12 ; x  9, z  27 y  6  x, z  Tổng điểm câu 1)  ( x  y)2 ( x  y)  2) Chứng minh a5  b5  a2b3  a3b2 ab c   (1) a  b5  ab a  b  c  0,25 0,25 0,25 0,25 2,00 0,50 0,50 0,25 0,25 0,25 0,25 2,00 0,25 0,25 0,25 NIC ∽ 0,25 0,25 0,25 NED NI NE   NI ND  NC.NE NC ND c) DIC  DHC (1) DHC  ABC (2) 0,25 0,25 1,00 b) 0,25 0,25 Tổng điểm câu 1a) HDC  AEB  90  DHEC nội tiếp đường trịn đường kính HC Tâm O trung điểm HC 0,25 Tổng điểm câu 0,25 0,25 0,25 3( x  1)2   5( x  1)2   0,25 Vậy   m  1, m  bc a  (2) ; b  c  bc a  b  c ca b  (3) 5 c  a  ca a  b  c Suy điều phải chứng minh Dấu xảy a  b  c Điểm Chấm 0,25 0,25 Thang điểm Đáp án Câu 0,25 x12  x22  x32   m  x  0; x   5; x   * Điểm Chấm 0,25 0,25 Lại có: BFC  BEC  900 nên tứ giác BFEC nội tiếp, suy ABC  AEF (3) Mà AEF  MEC (đối đỉnh), từ MEC  DIC tứ giác MENI nội tiếp, suy EMN  EIN (4) ACB  EIN (5) ACB  AFE (6) Suy AFE  EMN  AB / / MN Mà AB  CH nên MN  CH 0,25 0,25 0,25 2) SABC  SABE nên C E cách AB hay AB // CE Tương tự đường chéo lại song song với cạnh tương ứng 0,25 Gọi P giao điểm BD CE đặt diện tích SBCP  x  Do tứ giác ABPE hình bình hành nên SBPE  SABE  0,25 Lại có: SBCP BP SBEP   , tức SPCD PD SPED x 1  x 1 1 x x 0,25  là:  SABCDE  SABE  SBPE  SCDE  SBCP  3 x Vậy: SABCDE    0,25 5 Tổng điểm câu 3,00 Đánh dấu X vào ý cột Thang điểm Ghi điểm câu vào cột Điểm chấm Tổng điểm chấm: – Bằng số: – Bằng chữ: Ngày …… tháng 06 năm 2018 Cán chấm thi (Kí ghi rõ họ tên) SỞ GIÁO DỤC – ĐÀO TẠO TỈNH HÀ NAM KỲ THI TUYỂN SINH LỚP 10 THPT NĂM HỌC 2018-2019 Mơn thi: TỐN CHUN Thời gian: 150 phút ĐỀ THI CHÍNH THỨC Câu   1 a 1 a 1      a  2a    a  1  a a  a   a  a   a    Cho biểu thức Q   1) Rút gọn Q 2) So sánh Q, Q3 Câu 1) Giải phương trình:  x 9 3    x   2x 2) Trong mặt phẳng tọa độ Oxy cho ( P) : y  x2  d  : y  m, (d ') : y  m2 (0  m  1) Đường d cắt P hai điểm phân biệt A, B , đường d’ cắt P hai điểm phân biệt C, D (hồnh độ A D âm) Tìm m cho diện tích tứ giác ABCD gấp lần diện tích tam giác OCD Câu Tìm số nguyên dương x, y thỏa mãn x  3.2 y  Câu Cho đường tròn (O) đường thẳng d cố định (d (O) điểm chung) Lấy M điểm di động đường thẳng d Vẽ hai tiếp tuyến MA MB phân biệt cát tuyến MCD với (O) cho C nằm M D, CD không qua tâm O Vẽ dây cung DN song song với AB Gọi I giao điểm CN AB Chứng minh IC BC IA  IB  IA BD b) Điểm I thuộc đường cố định M di chuyển đường thẳng d a) Câu Một học sinh tùy ý chấm điểm phân biệt vào hình trịn có bán kính Chứng minh tồn điểm A, B điểm cho cho AB  Câu Cho số thực dương x, y, z thỏa mãn xy  yz  xz  x  y  z Chứng minh rằng: x2 x3   y2 y3   z2 z3  1 ĐÁP ÁN Câu 1)Rút gọn Q Điều kiện  a    1 a 1 a 1 Q     a  2a   a  a   a  a  1 a  1 a   1 a   a  a2        a  2a    1 a  1 a a2 a   a  a   a         a2  1 a 1 a      a  1   a  1 a  1 a  a  1 a  1 a  a   a   a2      a 1 (do a  0) a   a   a  a    a   a  a2 1  1  a  a 1 a  1 a     1 a  1 a   1 a  1 a 1 a  1 a  a 1  a  a   a   a2 2a   a2  (1  a )  a    (  a  1)  a2 1 1  a  a  a2 1 (1  a ) a  a2 1 (1  a ) a 2) So sánh … Điều kiện  a  Ta có: Q3   a  1 Q3  Q   a  1   a  1 Xét hiệu :   a  1  a  1  1   a  1 a   1 a   1  a(a  1)(a  2)   a    a   a    a  a  1 a    Mà a     Q3  Q   Q3  Q Vậy Q3  Q Câu 1) Giải phương trình:  x 9 3    x   x (*) ĐKXĐ: 9  x  a  x   b   x (a  0) a  x    b   b   x  a  b  18  b  18  a (1) *   a  3 b  3   a   Đặt   a  3 b  3   a  3 a  3   a  3 b   2a      a  3 b  2a  3  a   a    b  a   b  a  +)Với a   1  b2  18    b   x   a2     x  (tm)  x  b    +)Với b  2a   1   2a  3  18  a2 (do b  0)  4a  12a   18  a  5a  12a     a  3 5a  3   a  3 (ktm)   a  (tm)  b  2a     21 (tm) 5   3 x   a    216  5   x   (tm) 25 21    9  x  b     Vậy tập nghiệm phương trình cho S   216  ;0  25  2) Trong mặt phẳng tọa độ Oxy……… Điều kiện:  m  Phương trình hồnh độ giao điểm đường thẳng d đồ thị hàm số  P  là:    A  m; m x  m  x m 0 x  m     x   m  B m ; m  2   Phương trình hồnh độ giao điểm d ' đồ thị hàm số (P) là: C  m; m2  x  m  x m 0 x  m     x  m   D  m; m  2 2 Ta có d , d ' hai đường thẳng song song với Ox  d / / d ' hay AB / /CD  ABCD hình thang Khi ta có: S ABCD   AB  CD  d  d , d ' Có: d  d , d '  m  m2  m  m2 (do  m  1) AB   CD   m  m  m m  S ABCD   2 m  2m  AB  CD  d  d ; d '     m  2m  m  m   m  m  m  m  1 Lại có: SOCD  d  O; CD  CD  m2 2m  m3 2 Theo đề ta có: S ABCD  9.SOCD     m  m  m  m   9m m m   m  1  m   9m3  m  m m   m  9m m  10m m  m  m       m  5m  m   2 m   1   m  m (VN ) 5m  m   Vậy m  thỏa mãn toán Câu Phương trình : x  3.2 y  1(*) +)Xét x  ta có: *   3.2 y   y  +)Xét x  ta có: *  72  3.2 y   y  16  y  +)Xét x  y  ta có: Vì 3.2 y  0(mod8)  VP  1(mod8)  x  1(mod8)  x  x  2n  *  n1   3.2 y   n  1 n  1  3.2 y    1 3, n n 1 7 n   2a   a, b :  n a  b  y b 7   3.2  2a  3.2b   2b  2a b  3   2b  b    a b (VN )   a b  2   2   b b   2   (tm)     a b    a     n  a      n   x  2n  Vậy số nguyên dương x, y thỏa mãn  x; y   1;1 ;  2;4  Câu N A O I D K C M IC BC IA  IB  IA BD Xét đường trịn  O  ta có: a) B d H CAB  CDB (hai góc nội tiếp chắn cung BC) (1) Ta có: ND / / AB  sd cung AN  sd cung BD Mà ACN góc nội tiếp chắn cung AN  DCB góc nội tiếp chắn cung BD    ACN  BCD   sd AN  sd BD    Từ (1) (2)  ACI DCB ( g  g )  2 AI CI AC IC BC     (dpcm) DB CB DC IA BD IC AC Chứng minh tương tự ta có  IB AD Xét MBC MDB ta có:  DMB chung MBC  MDB (góc nội tiếp góc tạo tia tiếp tuyến dây cung chắn cung BC)  MBC  MDB ( g.g ) MB MC BC (các cặp cạnh tương ứng tỉ lệ)   MD MB BD Chứng minh tương tự ta có: MAC MDA( g.g )  MA MC AC   MD MA AD  IC MA  AC   IB  MD   AD       IC  MB   BC   IA MD  BD  Mà MA  MB (tính chất hai tiếp tuyến cắt nhau)  IA  IB b) Điểm I thuộc đường cố định… Kẻ OH  d  H  Gọi K  OH  AB (dpcm) Vì IA  IB(cmt )  I trung điểm AB  M , I , O thẳng hàng MO  AB  O Xét OIK OHM ta có: IOK chung; OIK  OHM  900  OIK OHM ( g.g ) OI OK   OI OM  OK OH OH OM Lại có: OB2  OI OM (Hệ thức lượng OBM vng B có đường cao BI)  OB  OB  OH OK   OI OM   OK  OH Mà OB  R , OH  d  O; d  không đổi  OK không đổi hay K cố định Vì OI  IB, O, K cố định nên ta có I thuộc đường trịn đường kính OK cố định (đpcm) Câu Ta xét hai trường hợp sau: TH1: Nếu điểm cho tồn điểm tâm đường trịn, tốn chứng minh TH2: Nếu sáu điểm khơng có điểm trùng với tâm đường trịn, ta xét hai khả xảy là: +)Trong sáu điểm có hai điểm nằm bán kính đường trịn, tốn chứng minh +)Trong sáu điểm cho khơng có hai điểm nằm mọt bán kính Khi ta vẽ sáu bán kính qua sáu điểm cho, hai bán kính gần tạo góc tâm Như ta có sáu góc tâm Theo nguyên lý cực hạn sáu góc tồn góc có số đo bé Mà tổng số đo góc 3600 nên góc bé khơng vượt q 60 Khơng tính tổng qt, ta giả sử góc AOB Đến ta có điều phải chứng minh Câu Áp dụng BĐT Cơ si ta có:  xy  yz  xz    x  y  z   3 xy  xz  yz   xy  xz  yz  Áp dụng BĐT Cauchy-Schwwarz ta có:  x  y  z  y  2  y  y  4   z  2  z   4 VT   x  2  x2  2x  4   VT   VT   x  y  z   y  z    x  y  z   x  y  z   12 x  y  z  xy  xz  yz  3  x  y  z   x  y  z   12  2 x  y  z  VT  2 x  y  z   x  y  z   15   xy  yz  xz   VT  2 x  y  z  x  y  z 2   x  y  z   15  Ta cần chứng minh: 2 x  y  z 1  x  y  z    x  y  z   15   x  y  z    x  y  z   15    x  y  z   x  y  z  3  Điều do: x  y  z  3 xy  yz  xz   Dấu “=” xảy  x  y  z  Vậy x2 x3   y2 y3   z2 z3  1 BỘ GIÁO DỤC – ĐÀO TẠO TRƯỜNG ĐẠI HỌC VINH ĐỀ THI CHÍNH THỨC KỲ THI TUYỂN SINH LỚP 10 THPT NĂM HỌC 2018-2019 Môn thi: TOÁN CHUYÊN Thời gian: 150 phút Câu Cho phương trình x   2m  3 x  3m   ( m tham số) a) Tìm tất số thực m để phương trình cho có hai nghiệm x1; x2 thỏa mãn điều kiện x12  x22  x1 x2  b) Tìm tất số nguyên m để phương trình cho có nghiệm ngun Câu a) Giải phương trình x  x   x2  x  1  x  x  y  y  b) Giải hệ phương trình   x2   y    x2 y2  Câu 3: Cho số tự nhiên n  số nguyên tố p thỏa mãn p  chia hết cho n đồng thời n3  chia hết cho p Chứng minh n  p số phương Câu Cho số thực khơng âm a, b thỏa mãn:  a  b   a  b  Chứng minh  rằng: 1     b3    9 3  a   b  1      a3 Câu Cho đường tròn (O; R)  O '; r  cắt điểm phân biệt A B  R  r ' cho O O’ phía AB, Gọi K điểm cho OAO ' K hình bình hành a) CMR: ABK tam giác vuông b) Đường trịn tâm K bán kính KA cắt (O; R) (O '; r ) theo thứ tự M N (khác A) Chứng minh ABM  ABN c) Trên đường tròn  O; R  lấy C thuộc cung AM không chứa B (C khác A, M) Đường thẳng CA vng góc với  O ', r  D CMR: KC  KD Câu 6: Cho 17 số tự nhiên mà chữ số số lấy từ tập hợp 0;1; 2;3; 4 Chứng minh ta chọn số 17 số cho cho tổng số chia hết cho ĐÁP ÁN Câu a) Tìm tất số thực m… Ta có:    2m  3   3m  1  4m2    m Do phương trình ln có hai nghiệm phân biệt  x1  x2  2m   x1 x2  3m  Theo định lý Vi-et, ta có:  Theo đề ta có: x12  x22  x1 x2    x1  x2   3x1 x2    2m  3   3m  1   m  1  4m  3m     m   Vậy giá trị cần tìm là: m  1; m  b) Tìm tất số ngun…… Để phương trình có nghiệm ngun   4m2  phải số phương Khi đó: 4m2   k  k  4m2    k  2m  k  2m    k  2m; k  2m  U (5)  1;5; 1; 5 Ta có bảng sau: k  2m k  2m m 5 1 Vậy giá trị cần tìm là: m  1; m  1 Câu 2: a) Giải phương trình: x  x   x2  x  Điều kiện xác định: x  Pt  x  x   x x   x  x   x  x  3  x  x  x  x  3  x  x  x  x  x  x  3  1 5 1 5 1 x  x  1  x  3x  3  x    x   x  3x    (VN ) Vậy phương trình cho có tập nghiệm S  0;1 b) Giải hệ phương trình: 1  x   y  3  x y    x2   y    x2 y2 Điều kiện : x; y  Ta có: 1  1  x  y 3 x   y     x y x y    (I )  2  x2   y    x     y         x2 y2 x  y  x Đặt a   x; b y với a  y Thay vào hệ (I) ta có:  a   a  b  b    a  b   2ab    2ab   ab      a  a  b    b  a  b  Mà a  nên   x  2  x  (tm)   x  x   x      y  1  y  y   y  1  y   (tm)  1   1   ; 1;      Vậy nghiệm hệ cho 1; Câu 3: n3    n  1  n2  n  1 p  p  1 n  p   n  p  n  Vì p  n    n  1 không chia hết cho p Do đó:  n  1  n2  n  1 p   n2  n  1 p Đặt : p   kn, k   p  kn  (*)   n  n  1  kn  1  kn   n  n   kn  n  n  k  n  k  n  n  1  n  kn  1  kn  1   k  1 n  k   kn  1 k    k  1 n  k    k  1 n  k  kn   k  n 1  k  n   p  kn   n  n   n  p  n  2n    n  1 Vậy n  p số phương Câu 4: a  b  ab2  a  2ab  b  a  b   a  b  a  b   2ab   a  a    b  b    ab  a  b  1  a  a  1  b  b  1   a  1 b  1  a  a  1  b  b  1 2  a  1 b  1 a b  2 b 1 a 1 a b Đặt x  ;y   x y 2 b 1 a 1  Ta có: 1  x 1  y   3    xy   x3  y    xy    x  y   x  y   3xy      xy  x y     x  y (do  xy   1)  xy   a  0; b   x  y   a  2; b  Dấu "  " xảy khi:  Câu a) CMR: ABK tam giác vuông Gọi I giao điểm KA OO ' Khi I trung điểm KA (tính chất hình bình hành) Mặt khác OO ' trung trực AB nên IA  IB (tính chất đường nối tâm giao tuyến chung hai đường trịn) Từ ta có: IA  IB  IK nên tam giác ABK vng B (tam giác có đường trung tuyến từ đỉnh B đến cạnh AK nửa cạnh AK tam giác tam giác vng B) Vậy ta có điều phải chứng minh b) Đường trịn tâm K… Ta có: KA  KM (cùng thuộc đường tròn  K ; KA) , OA  OM  R Suy OK trung trực AM (tính chất đường trung trực)  KO  AM Vì KO / / AO '  MA  AO ' (từ song song đến vng góc) Do đó: MA tiếp tuyến  O ' (định nghĩa) Suy : MAB  ANB; NAB  AMB Khi xét hai tam giác: AMB ABN ta suy ra: ABM  ABN c) Trên đường tròn (O;R) lấy C thuộc cung… Gọi E, F trung điểm CA, AD H trung điểm EF OE  CD (quan hệ đường kính dây cung) O ' F  CD  Khi ta có:   OE / /O ' F (từ vng góc đến song song)  OEFO ' hình thang vng E , F Lại có H trung điểm EF , I trung điểm OO ' (cách dựng)  HI / /OE / /O ' F (đường trung bình hình thang)  HI  CD (từ song song đến vng góc)  HI đường trung trực EF  IE  IF (tính chất đường trung trực) Lại có: EI đường trung bình ACK (E trung điểm AC, I trung điểm AK)  KC  2EI (tính chất đường trung bình tam giác) Mà FI đường trung bình ADK ( F trung điểm AD, I trung điểm AK)  KD  2FI (tính chất đường trung bình tam giác)  KD  KC   2EI  (đpcm) Câu 6: Ký hiệu A, B, C, D, E tập hợp số có chữ số tận 0;1;2;3;4 Nếu tập khác rỗng ta chọn từ tập hợp phần tử Khi tổng số chọn có tận nên chia hết cho Nếu có tập khác rỗng theo ngun lý Dirichle tập cịn lại ln có tập có phần tử Ta chọn số từ tập này, tổng số chọn chia hết cho Vậy trường hợp ta chọn số có tổng chia hết cho SỞ GIÁO DỤC – ĐÀO TẠO TỈNH BÌNH DƯƠNG KỲ THI TUYỂN SINH LỚP 10 THPT NĂM HỌC : 2018-2019 Mơn thi: TỐN CHUYÊN Thời gian: 150 phút ĐỀ CHÍNH THỨC Câu a) Giải phương trình :  x  x    x   x    b) Cho số thực x, y thỏa mãn x  2018  x y  2018  y  2018 Tính giá trị biểu thức Q  x2019  y 2019  2018  x  y   2020 Câu Gọi x1; x2 nghiệm phương trình x2   m  1 x  2m   Tìm tất giá trị 2 x  x  m nguyên dương để A       có giá trị nguyên  x2   x1  Câu a) Tính giá trị biểu thức P  1    1  2025 2024  2024 2025 b) Tìm tất số nguyên dương x, y thỏa mãn: x2  y   x  y  Câu Cho đường tròn (O) bán kính R điểm M nằm ngồi (O) Kẻ tiếp tuyến MA, MB với đường tròn (O) (A;B tiếp điểm) Trên đoạn AB lấy điểm C (C khác với A B) Gọi I, K trung điểm MA, MC Đường thẳng KA cắt (O) điểm thứ hai D a) CMR: KO2  KM  R2 b) CMR: Tứ giác BCDM tứ giác nội tiếp c) Gọi E giao điểm thứ hai MD với (O), N trung điểm KE Đường thẳng KE cắt (O) điểm thứ hai F CMR: điểm I, A, N, F thuộc đường tròn ĐÁP ÁN Câu a) Giải phương trình  x  x    x   x Điều kiện xác định:  x   PT   x  x   x   x a Đặt   7x  x a   (b  0)  7  x  b (a  0)   7x b  PT  b  2a    a  b  b  2b  2a  ab   b b  2  a   b     a  b   b    a  b  x   x  x  (tm)   b  (tm)   x   x  3(tm) Vậy nghiệm phương trình cho x  3; x  b) Cho số thực x,y thỏa mãn…… x  2018  x  y   x  2018  x   x  2018  x   2018  y  2018 2018 y  2018  y 2018  2018  y  y 2018  y  y  x  2018  x  2018  y  y Biến đổi tương tự ta có: 2018  x  x  2018  y  y (2) Cộng vế với vế (1) (2) ta được: 2018  x  2018  y  2018  x  2018  y x  y  x2  y   x   y  (1) +)Với x  y ta có: 1  x  2018  x  2018  x  x  2x   x   x  y   x 2019  y 2019   x  y   Q  x 2019  y 2019  2018( x  y )  2020  2020  x 2019  y 2019   Q  2020 +)Với x   y , ta có:  x  y   Vậy Q  2020 Câu Ta có:  '    m  1  2m    m2  4m     m     x 2 Suy phương trình cho ln có hai nghiệm phân biệt với m   x1  x2   m  1   x1 x2  2m  Áp dụng định lý Vi-et ta có:  Theo đề ta có: 2 2  x1   x2  x14  x24  x1  x2   x1 x2 A      2  x1 x2   x1 x2   x2   x1  2 2  x1  x2 2  x1 x2    x1 x2 2    x1 x2    m  12   2m      2m  2   2  2m    4m  8m   4m  12   2m   2 2  4m  12m  16   2m  6m     2  2 2m  m3     (m  3)  m  m         m3     A 2m  6m     2m  6m    m  3 m3 Ta có: 2m2  6m   2m(m  3)   Ta thấy: 2m  m  3  m  3 m    2m2  6m  8  m  3 m    2m2  6m  8  m  3   m  3 hay  m  3 U (8)   m  3 1; 2; 4; 8 Ta có bảng giá trị -8 -4 -2 -1 x 3 -5 -1 x Kết hợp với điều kiện m  ta có giá trị thỏa mãn tốn: m5; 1;1;2;4;5;7;11 Câu Dựa vào đề ta có phân số tổng quát dãy số  n  1 n  n n    n  1 n  n n  1  3 n  n n   n  1 n  n  n  1 n  2n  n  n  n  n  1  n  1  n  n n 1 1   n  n  1 n n 1 Từ ta có: 1    1  2025 2024  2024 2025 1 1 1        2 2024 2025 1 44  1  1  45 45 2025 P Vậy P  44 45 b) Tìm tất số nguyên dương x, y thỏa mãn… 11 x2  y  xy Áp dụng BĐT Co si ta có: x  y  xy   x  y x2  y x y   xy  x  y  2 2 x y 2  x  y  3( x  y )  2  6 x  y   x  y 6 x y Lại có: x, y   x  y  x  y    x  y   x  y   x  y  TH1: x  y   x  y   x2  y      ktm  x  2; y  TH : x  y     x  y  5(ktm)  x  1; y   x  3; y   x  y  10 (ktm)  TH : x  y    x  1; y   x  y  10 (ktm)  x  y   x2  y  (ktm)   x  3; y   x  y  13  x  2; y   x  y  13  TH : x  y    x  4; y   x  y  17   x  1; y   x  y  17 (ktm) (ktm) (ktm) (ktm)  x  4; y   x  y  20 (ktm)  2  x  2; y   x  y  20 (ktm) TH : x  y    x  5; y   x  y  26 (ktm)   x  y   x  y  18    3 (tm) Vậy nghiệm dương phương trình cho  3;3 Câu A E H D I F Q M P G O C K B a) CMR: KO2  KM  R2 Ta có I trung điểm AM , K trung điểm MC (gt)  KI / / AC (đường trung bình AMC ) Lại có: MO  AB (tính chất hai tiếp tuyến cắt nhau)  KI  MO (từ song song đến vng góc) Gọi P giao điểm MO Q giao điểm KI MO Xét OAM vng A có đường cao AP ta có: OA2  OP.OM  R2 Xét MQK vng Q ta có: KM  MQ2  QK Xét QKO vng Q ta có: KO2  QO2  QK  KO  KM  QO  QK  MQ  QK  QO  MQ   QO  MQ  QO  MQ   AI  MI  IQ / / AP Xét AMP có:  ( gt )  Q trung điểm MP (tính chất đường (cmt ) trung bình)  MQ  QP  KO  KM   QO  MQ  QO  MQ    QO  QP  MO  OP.OM  R Vậy KO2  KM  R b) CMR: tứ giác BCDM nội tiếp Gọi giao điểm KO với  O  G, H hình vẽ Xét KGA KDH ta có: AKG chung; KAG  KHD (hai góc nội tiếp chắn cung DG)  KGA KDH ( g  g )  KG KA   KG.KH  KA.KD (các cặp cạnh tương ứng tỉ lệ) KD KH Ta có: KM  KO  R   KO  R  KO  R    KO  OG  KO  OH   KG.KH  KD.KA  KM KD   KMD KA KM KAM (cmt ) (c.g.c)  KMD  KAM  DBA  CBD (Các góc tương ứng nhau) Vậy ta có tứ giác MBCD tứ giác nội tiếp c) Gọi E giao điểm thứ MD với (O), N trung điểm KE…… Ta có: MDE cát tuyến, MA tiếp tuyến nên MEA  DAM (góc nội tiếp góc tạo tiếp tuyến dây cung chắn cung AD)  Mà MAD  KMD (theo câu b)  MEA  EMK  MAD  Lại có hai góc vị trí so le  MK / / AE Tứ giác MAKE nội tiếp có I , N trung điểm cạnh bên AM EK  IN / / AE Do đó: FNI  KNI  FEA  IN / / AE   FAI Do tứ giác IANF nội tiếp (điều phải chứng minh) SỞ GIÁO DỤC VÀ ĐÀO TẠO THÁI BÌNH KỲ THI TUYỂN SINH LỚP 10 THPT CHUYÊN THÁI BÌNH NĂM HỌC 2018-2019 ĐÁP ÁN GỒM 04 TRANG HƯỚNG DẪN CHẤM VÀ BIỂU ĐIỂM MƠN: TỐN CHUNG (Dành cho tất thí sinh) Câu Điểm 2.5 Nội dung Cho biểu thức : x4   (với x  0, x  ; x  1; x  ) P  1 :  x  x   2x  x  a) Rút gọn biểu thức P b) Tìm x cho P  2019 c) Với x  , tìm giá trị nhỏ T  P  10 x Ýa 1.0  ( x  2)( x  2)  P  1 (2 x  1)( x  1) ( x  1)( x  2)   0.5 0.25 x 1 (2 x  1)( x  1) x 1 P  4x  P Ýb P  2019  x   2019 x  505 Ýc 10 10 10 x 18 x  4x    (  )  1 x x x 5 10 x 18 x 10 x 18 T (  ) 1    =21 ( Do x  côsi) x 5 x 5 Vậy T có giá trị nhỏ 21 x  T P 1 x m m (với m tham số, m  ) Gọi I ( x0 ; y0 ) tọa độ giao điểm hai đường thẳng d1 với d Tính T  x02  y02 Cho hai đường thẳng (d1 ) : y  mx  m (d ) : y   Hoành độ điểm I nghiệm phương trình 0.25 0.5 0.25 0.25 1.0 0.25 0.5 0.25 0.75 1  m2  x   mx  m  x  m m  m2  m2 2m  m 2m x   y   I ( ; )  m2  m2  m2  m2  m2 2m T ( ) ( ) 1 1 m  m2 Chú ý Ý học sinh dùng quỹ tích I đường tròn R=1 Gọi x1 , x2 hai nghiệm phương trình: x2  (2  m) x   m  (1) ( m tham số) a)Tìm m để x1  x2  2 1 b)Tìm m cho T  đạt giá trị nhỏ  ( x1  1)2 ( x2  1) Ýa 0.25 0.25 0.25 1.25 0.75   m2   m nên phương trình (1) ln có hai nghiệm phân biệt  x  x2  m  Theo viét   x1 x2  1  m x1  x2  2  ( x1  x2 )2   ( x1  x2 )2  x1.x2  0.25 (m  2)2  4(1  m)   m2   m  0.25 Ýb 0.25 0.5 ( x2  1)2  ( x1  1) 2  ( x1  x2 )  x1.x2  2( x1  x2 ) T  ( x1  1)2 ( x2  1)2 ( x1.x2  x1  x2  1) ( Phương trình có hai nghiệm phân biệt khác -1 với m) m2  T 1 T nhỏ m=0 0.25 x  8072  x  18162  3   x  y  3x  x  y   b) Giải hệ phương trình :  2   x  y  3x  1.5 a) Giải phương trình : Ýa 0.25 0.75 Đk x  2018 ta có 4( x  2018)  9( x  2018)  0.25 x  2018  x  2018   x  2018  0.25 x  2017 0.25 Ýb 0.75 x3  y3  3x2  x  y    [( x  1)3  y ]  3( x  1)  y  ( x   y)[( x  1)2  ( x  1) y  y  3]   y  x  0.25 0.25 x  Với y  x  vào x  y  3x  ta có x  x    x   0.25 Vậy hệ có hai nghiệm (0;1),( ; ) 2 Cho đường trịn tâm O bán kính a điểm J có JO  2a Các đường 3.5 thẳng JM , JN theo thứ tự tiếp tuyến M , N đường tròn ( O ).Gọi K trực tâm tam giác JMN , H giao điểm MN với JO a) Chứng minh : H trung điểm OK b) Chứng minh : K thuộc đường tròn tâm O bán kính a c) JO tiếp tuyến đường trịn tâm M bán kính r Tính r d) Tìm tập hợp điểm I cho từ điểm I kẻ hai tiếp tuyến với đường tròn ( O ) hai tiếp tuyến vng góc với Ýa Ýb Ýc Ýd 2 Do MK ON vng góc JN (1) 0.25 NK OM vng góc JM (2) 0.25  MK / /ON Nên từ (1) (2) có   Tứ giác OMKN hình bình hành(3), 0.25  NK / /OM suy H trung điểm OK Do OM = ON (4) Từ (3)&(4) có tứ giác OMKN hình thoi (5) Mặt khác OJ = 2OM = 2a suy MOJ  600 (6) Từ(5)và(6)  MOK  600  OMK  OK  OM  R  a  K thuộc đường tròn tâm O Do (M;r) nhận OJ tuyến tuyến mà MH  JO  H  r  MH a 1 r      Ta có MH OM JM 3a ( dùng hệ thức lượng tam giác vuông) Gọi IE,IF hai tiếp tuyến với (O) E,F IE  IF Suy tứ giác IEOF hình vng 0.25 0.25 0.25 0.25 0.25 0.25 0.5 0.25 Tính OI  a (Không đổi)(1) 0.25 Do O cố định (2) Từ (1) (2) tập hợp I nằm đường trịn tâm O bán kính a Cho x, y, z ba số thực không âm thỏa mãn :12 x  10 y  15z  60 Tìm giá 0.25 0.5 trị lớn T  x2  y  z  x  y  z Do x, y, z ba số thực không âm thỏa mãn :12 x  10 y  15z  60  x, y , z  x   Ta có  (*) y    z  0.25 Từ điều kiện ta có T  x2  y  z  x  y  z  x( x  5)  y ( y  6)  z ( z  4)  x  y  3z 12 x 60  x  y  3z   y  3z   12 5 x  x    Vậy GTLN T 12 đạt  y  or  y  z  z    0.25 HƯỚNG DẪN CHẤM CHUNG *Trên bước giải khung điểm bắt buộc cho bước ,yêu cầu học sinh phải lập luận ,biến đổi trình bày hợp lý cho điểm *Phải có hình vẽ ,khơng có hình vẽ không chấm điểm *Các làm theo cách khác với đáp án mà cho điểm tối đa theo biểu điểm *Điểm toàn tổng điểm phần khơng làm trịn ……………… Hết…………………… SỞ GIÁO DỤC VÀ ĐÀO TẠO LÂM ĐỒNG ĐỀ CHÍNH THỨC Câu 1: (2,0 điểm)  Tính giá trị biểu thức A   15  KỲ THI TUYỂN SINH VÀO LỚP 10 THPT CHUN NĂM HỌC 2018-2019 Mơn thi: TỐN Thời gian làm : 150 phút 10    15 Câu 2: (2,0 điểm) Trên hệ trục tọa độ Oxy (cách chọn đơn vị hai trục tọa độ nhau), cho đường thẳng  d  có hệ số góc  đường thẳng  d  qua A(3; 4) Tính khoảng cách từ điểm O đến đường thẳng d  Câu 3: (1,5 điểm) Cho  góc nhọn Chứng minh sin   sin   cos2   cos4   cos2   sin   Câu 4: (2,0 điểm) Cho đường trịn (O) đường kính AB Kẻ đường thẳng d tiếp tuyến đường tròn B Qua A kẻ hai đường thẳng cắt đường thẳng d E, F (điểm B nằm E F), AE cắt dường tròn (O) điểm thứ hai C, FA cắt đường tròn (O) điểm thứ hai D Chứng minh CDFE tứ giác nội tiếp Câu 5: (2,0 điểm) Cho phương trình x2  mx  2m   ( x ẩn số, m tham số) Tìm m để phương trình có hai nghiệm trái dấu mà nghiệm âm có giá trị tuyệt đối lớn nghiệm dương Câu 6: (1,5 điểm) Cho tam giác ABC cân A Kẻ đường cao BH ( H thuộc đường thẳng AC) Chứng minh BC  2CH AC Câu 7: (1,5 điểm) Cho ab  bc  ac  Chứng minh:  a  1 b2  1 c  1   a  b  b  c  c  a  Câu 8: (1,5 điểm) Cho hình chữ nhật ABCD Trên cạnh AB lấy điểm E , cạnh DC lấy điểm F cho AE  CF , M điểm nằm A, D Gọi G H giao điểm FE với MB MC Chứng minh: S AEGM  SMHFD  SGBCH Câu 9: (1,5 điểm) Với n số tự nhiên chẵn, chứng minh rằng:  20n  16n  3n  1 323 Câu 10: (1,5 điểm)  x  y  x  y  18 Giải hệ phương trình:   xy ( x  1)( y  1)  72 Câu 11: (1,5 điểm) Cho a, b, c  a  2b  3c  20, tìm giá trị nhỏ biểu thức : S  abc   a 2b c Câu 12: (1,5 điểm) Từ điểm A nằm ngồi đường trịn  O  , vẽ tiếp tuyến AB AC đường tròn ( B, C tiếp điểm) cát tuyến ADE (D nằm A E) Đường thẳng qua D song song với AB cắt BC BE theo thứ tự H K Chứng minh DH = HK ĐÁP ÁN Câu 1:   10    15  15    16  15   .      A   15  15  8 5   5    5  4   15  15    Vậy A = Câu Giả sử đường thẳng cần tìm có dạng y  ax  b Theo đề : đường thẳng  d  có hệ số góc  4 4  phương trình đường thẳng  d  có dạng y  xb 3 Theo đề ta có: đường thẳng  d  qua A  3;  a 4  b  b    d  : y   x  3 Gọi A, B giao điểm  d  với Ox Oy  A  xA ;0 ; B  0; yB    0   xA   x  6  A  6;0    A   B  0;8   y     yB  B  Xét tam giác vng OAB có đường cao AH khoảng cách từ gốc tọa độ tới đường thẳng  d  Suy độ dài  OA  xA  6; OB  yB  Áp dụng hệ thức lượng vào tam giác vng OAB có: 1 1 25 24    2   AH  2 AH OA OB 576 Vậy khoảng cách từ điểm O đến đường thẳng  d  24 Câu 3: Ta có: VT  sin   sin   cos   cos   cos   sin    sin  1  sin    cos    cos  1  cos    sin    sin  cos   cos    cos  sin   sin   cos  1  sin    sin  1  cos    cos   sin   sin   cos   cos  cos   sin  sin  (    900  sin  , cos   0)  cos   sin   (dpcm) 2 2 Vậy sin   sin   cos   cos   cos   sin  1 Câu d E C A B O D F Xét AEB vuông B có: CAB  CEB  90 (1) Mà có BC  AE ACB góc nội tiếp chắn nửa đường tròn  CBA  CAB  900 (2) Từ (1) (2)  CBA  CEB(3) Xét đường tròn (O) có CBA, CDA hai góc nơi tiếp chắn cung AC  CBA  CDA(4) Từ (3) (4)  CEB  CDA  CDFE tứ giác nội tiếp (góc đỉnh góc ngồi đỉnh đối diện) (đpcm) Câu Phương trình có hai nghiệm trái dấu  ac   2m    m   x1  x2  m  x1 x2  2m  Áp dụng định lý Vi ét ta có:  Phương trình có hai nghiệm trái dấu nghiệm âm có giá trị tuyệt đối lớn  x1  x2   m  Vậy m  thỏa mãn yêu cầu toán Vậy phương trình có hai nghiệm trái dấu  m  Câu 6: A H B C M Gọi M trung điểm BC  AM  BC (do tam giác ABC cân A) Xét ACM BCH có: ACB chung ; BHC  AMC  900  ACM  BC BCH ( g  g )  AC CM   BC.CM  AC.CH BC CH BC  AC.CH  BC  AC.CH (dpcm) Câu a   a  ab  bc  ca  a  a  b   c  a  b    a  c  a  b  b   b  ab  bc  ac   b  c  a  b  c   c  ab  bc  ac   a  c  b  c    a  1 b  1 c  1   a  b   b  c   c  a    a  b  b  c  c  a   2 Vậy đẳng thức chứng minh Câu E A B G M D H F C 2 Xét tam giác MBC có SMBC  AB AC  S ABCD  SMBC  SMBD  SMAB  SMGH  S BGHC  S EBG  S AEGM  S MDFH  S HFC (1) Xét hai hình thang AEFD EBCF có: AD  AE  DF  S EBCF  AD  EB  FC  Mà có: EB  DF , FC  AE  S AEFD  SEBCF SEBG  SHFC  SBGHC  S AEGM  SMDFH  SMGH (2) S AEFD  Trừ (1) cho (2) ta được: SMHG   SEBG  SHCF   SHFC  SEBG  SMHG    SMHG   SEBG  SHCF   Mà có: SEBG  SHFC  SBGHC  S AEGM  SMDFH  SMGH  S AEGM  SMDFH  SBGHC (đpcm) Câu +) Chứng minh đẳng thức  a  b   a n1  a n2b  a n3b2   a 2b n3  ab n2  b n1   a n  a n1b  a n2b   a 3b n3  a 2b n2  ab n1   a n1b  a n2b   a 2b n2  ab n1  b n  +)  a n  bn Vì n số chẵn, đặt n  2k , k  ta có: 20n  16n  3n   202k  162k  32k   400k  256k  9k  Để chứng minh  20n  16n  3n  1 323 , ta cần chứng minh  20n  16n  3n  1 chia hết cho 19 17 Ta có: 400k  1k   400  1  400k 1  400 k 2.1  400 k 3.12   400.1k 2  1k 1   399  400k 1  400 k 2.1  400 k 3.12   400.1k 2  1k 1   19.21  400k 1  400k 2.1  400 k 3.12   400.1k 2  1k 1  19 256k  9k   256    256k 1  256 k 2.9   9k 2.256  9k 1   247  256k 1  256k 2.9   9k 2.256  9k 1   13.19  256k 1  256k 2.9   9k 2.256  9k 1  19   400k  1k  256k  9k 19  Tương tự ta có: 400k  9k   400    400 k 1  400 k 2.9   k 2.400  k 1   17.23  400k 1  400k 2.9   9k 2.400  k 1  17 256k    256  1  256k 1  256 k 2.1   256.1k 2  1k 1   17.15  256k 1  256k 2.1   256.1k 2  1k 1  17  400k  9k  256k  17 Như ta có: 20n  16n  3n    400k  256k  9k  1 19    20n  16n  3n  1 19.17   n n n k k k 20  16     400  256   1 17   20n  16n  3n  1 323 Như ta có điều cần chứng minh Câu 10   x  y  x  y  18  x  y   xy  ( x  y )  18     xy ( x  1)( y  1)  72  xy ( xy  x  y  1)  72 Đặt x  y  a, xy  b ta có hệ cho trở thành:  a  a  18 (1) b  a  a  2b  18    b(a  b  1)  72  a  a  18  a  a  a  18  1  72(2)    2   (2)   a  a  18  a  3a  16   288   a  2a  17    a  1   a  2a  17    a  1   288   a  2a  17    a  1  288 2  a  4a  4a  68a  34a  289  a  2a   288  a  4a  31a  70a  288  288  a  a  4a  31a  70    a  a  5a  9a  45a  14a  70    a  a    a  9a  14    a   b  9 a   b   a  a   a   a       a  2  b  8   a  7  b  12 a  X  ta có hai số x, y nghiệm phương trình X  X     b  9  X  3 +) Với  Vậy ta hai nghiệm hệ phương trình  x; y    3; 3 ;  3;3 a  X  ta có hai số x, y nghiệm phương trình X  X     b  X  +)Với  Vậy ta hai nghiệm hệ phương trình  3;2  ;  2;3 a  2 X  ta có hai số x, y nghiệm phương trình X  X     b  8  X  4 +)Với  Vậy ta hai nghiệm hệ phương trình  4;2  ;  2; 4  a  7  X  3 ta có hai số x, y nghiệm phương trình X  X  12    b  12 X  +)Với  Vậy ta hai nghiệm hệ phương trình  x; y    3; 4  ;  4; 3 Vậy ta hai nghiệm hệ phương trình  x; y    3; 4 ;  4; 3 ;  4;2 ;  2; 4  ; 3;2 ;  2;3; 3; 3;  3;3 Câu 11 S  abc  a b 3c  3a   b   c              a 2b c 4  a   2b   c  3a b c  a  2b  3c               a   2b   c  Áp dụng bất đẳng thức Cô – si cho hai số khơng âm ta có: ) 3a 3a  2 3 a a b b )  2 3 2b 2b c c )    c c  3a   b   c   S   a  2b  3c               20     13  a   2b   c  a  2b  3c  20  3a   a   a   b  (a, b, c  Dấu “ = “ xảy :  b   2b c    c   c Vậy MinS  13 a  2; b  3; c  Câu 12 E B K H D A C Xét đường trịn  O  có: SBE góc tạo tiếp tuyến dây cung chắn cung BE BCE góc nội tiếp chắn cung BE  SBE  BCE (định lý) Mà có: SBE  BKH (hai góc so le trong, KD // AB)   BKH  BCE  SBE  Xét BKH BCE có: BKH  BCE (cmt ); EBC chung KH BH CE   KH  BH (1) CE BE BE BH HD CD Chứng minh tương tự ta có: BHD BDC ( g.g )    HD  BH (2) BD CD BD Xét BAD EBA có: BAE chung; DBA  BEA (góc nội tiếp góc tạo tiếp tuyến dây cung chắn cung  BKH BCE ( g.g )  BD)  BAD EAB( g.g )  BE AB (hai cặp cạnh tương ứng tỉ lệ)  BD AD AB AC (do AB, AC tiếp tuyến qua A đường tròn  O  nên AB =  AD AD BE CE CE CD AC)     (3) BD CD BE BD KH BH CE BD Từ (1) (2) (3) ta có:    AH  HK (đpcm) HD BH BE CD Mà có: SỞ GIÁO DỤC – ĐÀO TẠO TỈNH ĐỒNG NAI KỲ THI TUYỂN SIN LỚP 10 NĂM HỌC 2018-2019 Mơn thi: TỐN CHUN Thời gian: 150 phút ĐỀ THI CHÍNH THỨC Câu 1) Giải phương trình: x4  22 x2  25   a a a  4a 2) Cho biểu thức P      a 2 a3 a 2 a a) Rút gọn biểu thức P b) Tìm số thực dương a cho P đạt giá trị lớn  x  xy  Câu Giải hệ phương trình:  2  3x  xy  y  30  x, y   Câu Tìm tham số thực m để phương trình x2   m  1 x  2m  có hai nghiệm phân biệt x1; x2 thỏa mãn P  x1  x2   x1  x2   3x1 x2  đạt giá trị nhỏ Câu 1) Tìm cặp số nguyên  x; y  thỏa mãn điều kiện x2  y  xy  3x   2) Cho số thực a, b, c Chứng minh a  b3 b3  c c3  a3 1      2 2 2 ab  a  b  bc  b  c  ac  c  a  a b c Câu Trên mặt phẳng tọa độ Oxy cho hai điểm M  50;100  N 100;0  Tìm số điểm nguyên nằm bên tam giác OMN (Một điểm gọi điểm nguyên hoành độ tung độ điểm số nguyên) Câu Cho đường tròn (O) đường kính AB cố định Biết điểm C thuộc đường trịn (O) , với C khác A B Vẽ đường kính CD đường trịn (O) Tiếp tuyến B đường tròn (O) cắt hai đường thẳng AC AD hai điểm E F 1) Chứng minh tứ giác ECDF nội tiếp đường tròn 2) Gọi H trung điểm đoạn thẳng BF Chứng ,minh OE vng góc với AH 3) Gọi K giao điểm hai đường thẳng OE AH Chứng minh điểm K thuộc đường tròn ngoại tiếp tứ giác ECDF 4) Gọi I tâm đường tròn ngoại tiếp tứ giác ECDF Chứng minh I thuộc đường thẳng cố định đường trịn (I) ln qua điểm cố định C di động (O) thỏa mãn điều kiện ĐÁP ÁN Câu 1: 1) Giải phương trình Đặt x  t  t   phương trình trở thành: t  22t  25  Ta có:  '  112  25  96   '  t  11  6(tm)  Phương trình có hai nghiệm phân biệt :  t2  11  6(tm)    x2  2   x  11     2  x  11   x  2    2 x  2   x   2   x  2    x   2     Vậy phương trình cho có tập nghiệm S   2  ;  2;2  3;2  2) Cho biểu thức… a) Rút gọn biểu thức P Điều kiện a   a a a  4a P   a  a  a    a  a a    a a     a 2 a 1 a   a    a a  4a    a  a    a            a a a 2 a 2 a  a 2 a    a 1  a  a  a   a  a  a  b) Tìm số thực dương……… Điều kiện a  Ta có: 1 9  P  a  a     a     2 4  1 Dấu “=” xảy  a    a   a  (tm) 2  Vậy Max P  a  Câu 2:  x  xy  (1)  Giải hệ phương trình:  2  3x  xy  y  30 (2) Xét x  không nghiệm hệ cho Xét x  ta có phương trình (1) tương đương với : x  xy   x  y  6  x  y x x Thay vào phương trình (2) ta được: 6  6  3x  x  x     x    30 x  x  108  x  x  12  x  36   30  x  x  x  108    x   x     x   (Vi x   0) x   y    x  3  y  1 Vậy hệ cho có nghiệm  3;1 ;  3; 1 Câu Để phương trình có nghiệm phân biệt    m  1  8m  m2  6m    m   10   m   10  x1  x2  m   x1 x2  2m Áp dụng định lý Vi et ta có:  Theo đề ta có: P x1  x2   x1  x2   3x1 x2    m  1   m m  2  m  1  3.2m  m  4m   m   m 8m  m2  4m   m      0 Xét biểu thức : P   2  m  2 8  m  2  m  2  P 1 0P 8 Dấu “=” xảy  m    m  2(tm) Vậy Min P  1 m  2 Câu 1) Tìm cặp số nguyên… x  y  xy  3x     x  xy  x    xy  y  y    x  y  1   x  x  y  1  y  x  y  1   x  y  1    x  y  1 x  y  1  (*) Do x, y  ,2 x  y  1lẻ nên ta có trường hợp sau đây:  x  (tm)   2 x  y   1  2 x  y  y         x  y   4  x  y  3      x   *    2 x  y   2 x  y       (ktm)  4   x  y     x  y   y   Vậy nghiệm nguyên phương trình cho 1; 1 2) Cho số thực dương… Ta có:  a  b   a  ab  b  a  b a  b3 1     ab(a  b ) 2a 2b 2ab 2ab  a  b    a  ab  b   b    a  b   Điều đúng, dấu xảy  a  b  c  b3 1    2  cb  c  b  2c 2b Chứng minh hồn tồn tương tự ta có:  3 1  c a   2  ca  c  a  2c 2a  Cộng vế theo vế ta có: a  b3 b3  c c3  a 1 1 1 1            ab  a  b  bc  b  c  ac  c  a  2a 2b 2c 2c 2a 2b a b c Dấu “=” xảy a  b  c Vậy ta có điều phải chứng minh Câu Gọi phương trình đường thẳng OM là: y  ax  b b  a    OM : y  x 50a  b  100 b  Ta có:  Tương tự ta có: Phương trình đường thẳng ON là: y  Phương trình đường thẳng MN là: y  2 x  200 Những điểm nằm tam giác OMN phải thỏa mãn điều kiện: y  2 x   x     y  2x  y  2 x  200 2 x  200   x  100  Do tọa độ nguyên nên điểm thỏa mãn đề : x  1;2;3 ;98;99 Lại có: x  2 x  200  x  50;2x  2x  200  x  50 Từ đó: Nếu x  ta có: y  x  y   có điểm nguyên Nếu x  ta có y  x  y   có điểm nguyên ……………… Nếu x  50 ta có y  x  y  100  có 99 điểm nguyên Nếu x  51  y  2 x  100  y  98  có 97 điểm nguyên ………………… Nếu x  99 ta có: y  2 x  200  y   có điểm nguyên Vậy tổng số điểm thỏa mãn : 1     97   99  49.(2.1  48.2)  99  4901 điểm Câu E P C I M O B A Q K H D F 1) Chứng minh tứ giác ECDF nội tiếp 1  E  sdAB  sdBC  sdAC   2 Ta có:   ADC  sdAC   (Vì góc ADC góc nội tiếp (O) chắn cung AC) 2) Gọi H trung điểm… Gọi K giao điểm EO AH EAF góc nội tiếp chắn nửa đường trịn nên góc vng Tam giác ABF ABE vuông E nên: N AB AO  AE FA HF AO BAF  AEB (do phụ với EAB ) nên  FA FA  AFH  EAO  Mặt khác  HF AO  AFH AEO(c.g.c)    FA FA sin BAF  BF HF ;  AF FA sin AEB   FAH  EAO  FAH  EAK  900  AEO  EAK  AEK  900  OE  AH (dpcm) 3) Gọi K giao điểm… Ta có: OBD tam giác cân O nên ODB  OBD EDB góc nội tiếp chắn nửa đường trịn nên góc vng, DH  BH (tính chất đường trung tuyến tam giác vng) Do BHD cân H nên BDH  DBH Vậy ODH  ODB  BDH  OBD  DBH  OBH  900  OKH Do tứ giác OKDH nội tiếp   KDO  KHO   CEK  KDO  CDK  CEK CEK  KHO   Nên tứ giác ECKD nội tiếp Vậy K thuộc đường tròn ngoại tiếp tam giác ECD 4) Gọi I tâm đường tròn ngoại tiếp tâm giác ECDF…… Gọi N giao điểm CB KH Vì góc ECN , EKN vng nên: EN đường kính (I) , I trung điểm EN Gọi P hình chiếu I lên EF Do NF vng với EF (vì EFN góc nội tiếp chắn nửa đường tròn) nên IP//NF IP đường trung bình tam giác ENF  IP  FN Tứ giác AFNB có FN / / AB; FA / / NB nên hình bình hành, FN  AB Do đó: IP  AB  OB Mà OB cố định nên I di động đường thẳng song song với EF, cách EF khoảng không đổi OB AB cắt (I) điểm Gọi điểm M Q, R bán kính đường trịn tâm O  MOD  COQ  ODM OQC ( g  g )  MDO  CQO  OD OM    OD.OC  R  OM OQ OQ OC CAM  QAE  ACM   ACM  AQE  AQE ( g g ) AC AM   AC AE  AQ AM AQ AE AC AE  AB  R  AQ AM  R   AO  OQ   AO  OM   R   R  OQ   R  OM   R  R  R.OM  R.OQ  OQ.OM  R  R  R  OQ  OM   R  R  OQ  OM  R Do ta ln tính OQ, OM theo R Mà O, R cố định nên Q, M cố định Vậy đường trịn (I) ln qua điểm cố định M, Q C di động đường tròn (O) SỞ GIÁO DỤC VÀ ĐÀO TẠO QUẢNG NAM Đ CH NH TH C (Đề có trang) KỲ THI TUYỂN SINH LỚP 10 THPT CHUYÊN NĂM HỌC 2018-2019 Môn thi : TỐN (Chun Tin) Thời gian : 150 phút (khơng kể thời gian giao đề) Ngày thi : 09/6/2018 Câu (1,5 điểm) Cho biểu thức A  x , với x  x  Rút gọn biểu thức A tìm x  x2 x x4 để A  Câu (1,0 điểm) Tìm hai số nguyên tố p q, biết p  q p  4q số phương Câu (2,0 điểm) a) Giải phương trình x2  3x   ( x  1)  x 2  x  x  y  y  b) Giải h phương trình  2  2 x  y  x  y   Câu (1,0 điểm) Cho đường th ng (d ) : y  x  m ( m tham số) parabol ( P) : y  x Tìm m để (d ) cắt ( P) hai điểm phân bi t có hồnh độ x1, x2 cho x12  x22  10 Câu (3,5 điểm) Cho tam giác nhọn ABC (AB < AC) nội tiếp đường trịn (O) , D điểm cung nhỏ BC đường tròn (O) , H chân đường cao vẽ từ A tam giác ABC Hai điểm K, L hình chiếu vng góc H lên AB AC a) Chứng minh AL.CB = AB.KL b) Lấy điểm E đoạn th ng AD cho DB = DE Chứng minh E tâm đường tròn nội tiếp tam giác ABC c) Đường th ng KL cắt đường tròn (O) hai điểm M, N (K nằm M, L) Chứng minh AM = AN = AH Câu (1,0 điểm) Cho hai số th c ương a b thỏa m n ab thức A  a  b  ab a  b  ab Tìm giá tr nhỏ biểu - HẾT - Trang 1/5 SỞ GIÁO DỤC VÀ ĐÀO TẠO QUẢNG NAM KỲ THI TUYỂN SINH LỚP 10 THPT CHUYÊN NĂM HỌC 2018-2019 HDC CH NH TH C HƯỚNG DẪN CHẤM MƠN: TỐN (Chun Tin) (Bản hướng dẫn gồm 05 trang) Nội dung Câu Cho biểu thức A  Điểm x , với x  x  Rút gọn biểu thức A tìm x để  x2 x x4 A A Câu (1,5) Câu x   x2 x x4 x (mỗi ý 0,25đ)  x ( x  2) ( x  2)( x  2) x 24    x  ( x  2)( x  2) ( x  2)( x  2) 2 A    x x 2 x  (mỗi ý 0,25đ) x 2 Nội dung Tìm số nguyên tố p q, biết p  q p  4q số phương  p  q  a2  Theo đề ta có  p  4q  b , suy b2  a  3q   b  a  b  a   3q  * a; b  N Từ q số nguyên tố a  b  nên ta có trường hợp sau: b  a  + TH 1:  suy b  a  2a   3q , suy q lẻ b  a  3q * Câu Ta viết q  2k  ( k  N ) (1,0) Khi 2a  3q 1  6k  hay a  3k  p  a – q  9k  4k  k  9k   Do p nguyên tố nên k  p  13, q  b  a  + TH 2:  , suy b  a  q  2a  b  a  q Lại có p  a  q  a  2a –   a  1 a – 3 Do p nguyên tố nên a  p  5, q  11 b  a  q + TH 3:  b  a  b  a  Suy b  a  q  số nguyên tố Trang 2/5 1,5 0,5 0,5 0,25 0,25 Điểm 1,0 0,25 0,25 0,25 0,25 Kết luận: (p;q) = (5;11), (13;3) Trình bày cách khác:  p  q  a2  Theo đề ta có  p  4q  b  * a; b  N (0,25) Suy b2  a  3q   b  a  b  a   3q Vì p, q số nguyên tố nên a  2, b  Do ta có trường hợp sau: b  a  + TH 1:  Khi b  a  2a   3q Suy q lẻ b  a  3q Ta viết q  2k  ( k  N * ) (0,25) Khi 2a  3q 1  6k  hay a  3k  p  a – q  9k  4k  k  9k   Do p nguyên tố nên k  Suy p  13, q  b  a  + TH 2:  Khi b  a  q  2a  b  a  q Lại có p  a  q  a  2a –   a  1 a – 3 Do p nguyên tố nên a  Suy p  5, q  11 Vậy p  13, q  p  5, q  11 (0,25) Nội dung Câu a) Giải phương trình x  3x   ( x  1)  x Điều ki n: x   (0,25)  x2  3x   ( x  1)  x  ( x  1) x    x  x   x    x   x   x  (thỏa điều ki n ≤ x ≤ 4) x2 4 x     4  x  ( x  2) Vậy phương trình đ cho có hai nghi m: x  1, x  2  Câu x  x  y  y  (2,0) b) Giải h phương trình  2 x  y  x  y     x2  x  y  y   ( x  y)( x  y  1)   x  y x  y   + Với x  y thay vào pt thứ hai ta được: x2  x    x  x  3 Suy được: ( x ; y)  (1;1) ( x ; y)  (3; 3) Điểm 1,0 0,25 0,25 0,25 0,25 1,0 0,25 0,5 + Với x  y    y   x thay vào pt thứ hai ta được: 0,25 x2  x    x  x  3 Suy được: ( x ; y)  (1;0) ( x ; y)  (3;4) Vậy h phương trình đ cho có nghi m: (1;1),(3; 3),(1;0),(3;4) * Lưu ý: Học sinh giải trường hợp: với x  y , với x  y   cho 0,5đ Trang 3/5 Câu Nội dung Điểm Cho đường th ng (d ) : y  x  m ( m tham số) parabol ( P) : y  x Tìm m để (d ) cắt ( P) hai điểm phân bi t có hồnh độ x1, x2 cho x12  x22  10 Phương trình hồnh độ giao điểm (d ) ( P) : x2  x  m  (1) +  ' 1  m Câu + (d ) cắt ( P) hai điểm phân bi t  '  hay m   (1,0) + x1, x2 hai hoành độ hai giao điểm (d ) ( P) nên x1, x2 nghi m pt (1) 1,0 0,25 0,25 0,25  x1  x2  Theo đ nh lý Viet:  (thí sinh khơng viết định lý mà thể dòng  x1.x2  m 0,25 được) x12  x22  10  ( x1  x2 )2  x1x2  10   2m  10  m  (thỏa m  1 ) Vậy m  giá tr cần tìm Lưu ý : Nếu thí sinh khơng lập ∆’ riêng mà ghi chung phần lập luận nghiệm phân biệt 0,5đ Trang 4/5 Câu Nội dung Điểm Cho tam giác nhọn ABC (AB < AC) nội tiếp đường tròn (O) , D điểm cung nhỏ BC đường tròn (O) , H chân đường cao vẽ từ A tam giác ABC Hai điểm K, L hình chiếu vng góc H lên AB AC A O L N E 0,5 K M B / H C D Hình vẽ phục câu a: 0,25 đ Hình vẽ phục hai câu b, c: 0,25 đ a) Chứng minh AL.CB = AB.KL - Xét hai tam giác AKL ACB, có: + A chung ; AK AL + AK.AB = AH2 = AL.AC  = AC AB Câu Suy hai tam giác AKL ACB đồng ạng (3,5) AL KL Suy =  AL.CB = AB.KL AB CB Lưu ý: HS chứng minh ∆AKL ~ ∆ACB theo cách khác 0,75đ 1,0 0,25 0,25 0,25 0,25 b) Lấy điểm E đoạn th ng AD cho DB = DE Chứng minh E tâm đường tròn nội tiếp tam giác ABC 1,0 + AE đường phân giác góc A tam giác ABC (*) + Tam giác DBE cân D nên: BED = EBD (1) 0,25 BED = BAD + ABE = BCD + ABE = DBC + ABE (2); EBD = DBC + EBC (3) 0,5 Từ (1), (2) (3) suy ABE = EBC hay BE phân giác góc B tam giác 0,25 ABC (**) Từ (*) (**) suy E tâm đường tròn nội tiếp tam giác ABC c) Đường th ng KL cắt đường tròn (O) hai điểm M, N (K nằm M, L) 1,0 Chứng minh AM = AN = AH + Hai tam giác AKL ACB đồng ạng 1 0,25 Suy ALK = ABC  sdAM + sd NC = sdAC 2 1  sdAM + sd NC = sdAN + sd NC  sdAM = sdAN  AM = AN (4) 0,25 2 + Chứng minh hai tam giác ALN ANC đồng ạng 0,25       AL AN =  AN2 = AL.AC Mà AL.AC = AH2  AN = AH AN AC Từ (5) (6) suy AM = AN = AH Suy Trang 5/5 (5) 0,25 Câu Câu (1,0) Nội dung Điểm Cho hai số th c ương a b thỏa m n a  b  ab Tìm giá tr nhỏ biểu 1,0 ab thức A  a  b  ab  a, b    a, b   Ta có:   1   a  b  ab  a  b 1 0,25   x, y  1 1 Đặt A     x,  y ta có  a b x y x  y2 x  y  A x2  y  x2 y x2  y  A   xy xy xy  3( x  y ) x2 y  x2  y x2 y  3.2 xy x2  y   x2  y x2 y 4x2 y x2  y 2 0,25  x y  x y 1 x  y  xy     xy  xy     2       2 x  y Suy A  10 Dấu ảy   x  y  hay a  b  x  y  Vậy giá tr nhỏ A 10 a  b  0,5 Cách khác: Câu Nội dung Điểm Cho hai số th c ương a b thỏa m n a  b  ab Tìm giá tr nhỏ biểu 1,0 ab thức A  a  b  ab Ta có: a b  a  b  ab  ab    a b Dấu đ ng thức ảy    a  b  ab    Câu (1,0)  a  b  a  b ab ab ab ab A  ab      ab   10 ab 4 ab 2 Suy ra: A  10 a  b   Đ ng thức ảy  a  b ab  a  b    ab  (0,25) (0,5) (0,25) Vậy giá tr nhỏ A 10 a  b  - HẾT * Lưu ý: Nếu thí sinh làm không theo cách nêu đáp án cho đủ số điểm phần hướng dẫn quy định Trang 6/5 SỞ GIÁO DỤC – ĐÀO TẠO TỈNH HẢI DƯƠNG TRƯỜNG THPT CHUYÊN NGUYỄN TRÃI ĐỀ THI CHÍNH THỨC KỲ THI TUYỂN SINH LỚP 10 THPT NĂM HỌC 2018-2019 Mơn thi: TỐN CHUN Thời gian: 150 phút Câu 1) Cho x  a    a  a2  a  1 ,  a  0 ; P  x  x  x 1 1 x2  x  Rút gọn P theo a 2) Cho x, y, z thỏa mãn x  y  z  xyz  Chứng minh x   y   z   y   x   z   z   x   y   xyz  Câu x a) Giải phương trình: 2( x  1) x   x  3x  xy  x  y   b) Giải hệ phương trình    x  x  1  y  y  1  Câu 1) Đặt N  a1  a2   a2018 , M  a15  a25   a2018  a1; a2 ; .a2018    Chứng mỉnh N chia hết cho 30 M chia hết cho 30 2) Tìm tất số tự nhiên n, k để n8  42k 1 số nguyên tố Câu Cho nửa đường trịn (O; R) đường kính BC Gọi A điểm di động nửa đường tròn (A khác B, C) Kẻ AD  BC  D  BC  cho đường trịn đường kính AD cắt AB, AC nửa đường tròn (O) E, F, G (khác A), AG cắt BC H AD theo R chứng minh H, E, F thẳng hàng BE.CF 2) Chứng minh FG.CH  GH CF  CG.HF 1) Tính 3) Trên BC lấy M cố định (M khác B, C) Gọi N, P tâm đường tròn ngoại tiếp tam giác MAB MAC Xác định vị trí A để diện tích tam giác MNP nhỏ Câu Cho a, b, c dương thỏa mãn ab  bc  ca  Tìm giá trị lớn biểu thức P 2a 1 a  b 1 b  c 1 c  a  28b2  28c ĐÁP ÁN Câu 1) Rút gọn P Điều kiện a  0; x  x  a 1 1 a   a  1 a  2a   a  2a  2a (a  1)  a 1 a  2a  a  2a   2a  a 1  a 1 a2  a  1 a  a  1  a  1 2 a2  a  a2  a    a 1  a 1  a 1 a 1   a2  a   0 a 1  a  2a   a  a  a    1a  a 1 a 1   x  a  P  x x  x  x 1 1 x2  2x  x 1 1 x 1    x  x 1 1  x  1 2  a  1 x 1 x 1    2a  a 1 x a   a 1 a 1 Vậy P  2a  2) Chứng minh… Ta có: x  y  z  xyz    x  y  z   xyz  16 Mặt khác: x   y   z   x 16   y  z   yz   x  4( x  y  z )  xyz   y  z   yz      x x  xyz  yz  x x  yz     x   y   z   x x  yz  x  xyz Chứng minh hoàn toàn tương tự ta có:  y   x   z   y  xyz     z   x   y   z  xyz Do x   y   z   y   x   z   z   x   y   xyz  x  y  z  xyz  xyz    x  y  z  xyz  Vậy x(4  y)(4  z)  y   x   z   z 4  x 4  y   xyz  Câu x Giải phương trình: 2( x  1) x   x  a) Điều kiện xác định: x  Ta có phương trình cho tương đương với:  x  1 x  3  1  x   1   x   x  (*) x x x  x Đặt t  x  x (t  0)  t  x   1   *     t  t  x  x t  2t   t   x x  t (2  t )  (t  2)  x 2   t  2   t   x  t   t  (tm)     t  t  x x  x )t   x    x2  4x   x x    x  1 x  3    (tm) x  )t    x   x3  3x   x x x  ( x  1)( x  x  4)  x 1   x  (tm)    x  2 (ktm)  x  4x   Vậy tập nghiệm phương trình cho S  1;3 3x  xy  x  y  b) Giải hệ phương trình:   x( x  1)  y ( y  1)  3x  xy  x  y  Ta có hệ phương trình   x( x  1)  y ( y  1)  3x  xy  x  y   x  x  y  y  (1)    2  x  x  y  y  2 x  y  xy  x  y  2 (*)  *  x  x  y     y  y    ** Coi phương trình bậc hai ẩn x y tham số:    y  5  4.2  y  y    y  18 y   9( y  1)  0y   y   y  1  y y    x  4 Phương trình (**) có hai nghiệm:   y   y  1  y    2 y  x  4 y 1 y 1  y 1 ) x   1    y2  y    2    y  y   y   y  y  16   y  y  13  y 1 x 1   y  13  x  4 5  ) x   y  1    y    y  y  y    y  y2   y  y2  y    y2  y    y 1  x  1  13 4  Vậy nghiệm hệ cho 1;1 ;  ;   5  Câu 1) Đặt… Ta có : N  a1  a2   a2018 ; M  a15  a25   a2018  a1; a2 a2018    Xét hiệu M  N  a15  a25   a2018  a1  a2   a2018   a15  a1    a25  a2     a2018  a2018  Ta có a5  a  a  a  1  a  a  1 a  1  a  a  1  a  1 a  1 30  2.3.5 với 2,3,5 số nguyên tố Ta có: a  a  1  a  1 a  1 có tích số tự nhiên liên tiếp a  1; a; a  1nên a(a  1)(a  1) chia hết cho Nếu a chia cho dư 0,1 a, a  1, a  1sẽ chia hết cho Nếu a chi dư a  chia hết cho Vậy a  a  1  a  1 a  1 chia hết cho 2;3;5 nên chia hết cho 30 Do M  N chia hết cho 30 M chia hết cho 30 Ta có diều phải chứng minh 2) Tìm tất cả… Ta có: n8  42 k 1  n8   22 k 1    n2   2.2k 1 n2   22 k 1    2k 1.n  2   n  22 k 1    2k 1.n  2   n  22 k 1  2k 1.n  n  22 k 1  2k 1.n  Do n, k số tự nhiên n8  42 k 1 số nguyên tố nên n8  42 k 1   n  22 k 1  2k 1.n  n  22 k 1  2k 1.n   n  22 k 1  2k 1.n   n  2.2k n   2k     n  2k    2k   2  n  2k  k    n8  42 k 1      k n   2   k n      k (VN )  2    n  2k  1  k (VN )  2  Vậy n  1, k  giá trị cần tìm Câu P A F G H E B I N D O C M 1) Tính… H,E,F thẳng hàng Ta có: BAC  DAE  AFD  900 (các góc nội tiếp chắn nửa đường tròn ) Áp dụng hệ thức lượng ABC, ADB, ADE vng D ta có: AB AC  AD.BC BD.CD  AD2 BE.AB  BD2 CF AC  CD2  BE.CF AB AC   BD.CD   AD AD  AB AC  BE.CF AD3 AB AC BC    AD  BC  R BE.CF AD AD Gọi I trung điểm AD E, I , F thẳng hàng (do AEDF hình chữ nhật) Xét đường trịn tâm I, đường kính AD ta có: IG  IA  AD Xét đường tròn tâm O, đường kính BC ta có: OG  OA  BC  OI đường trung trực AG (tính chất đường trung trực)  OI  AH Lại có: AI  OH  I trực tâm AOH  IH  OA  H , E, F thẳng hàng (đpcm) 2) Chứng minh FG.FH+……… Ta có: AGD  900 (góc nội tiếp chắn nửa đường tròn) Xét AGD ADH ta có: A chung; AGD  ADH  900  AGD ADH ( g.g )  ADG  AHD (các góc tương ứng) Lại có: AFG  ADG (các góc nội tiếp chắn cung AG)   AFG  AHD  AHC  ADG   GFCH tứ giác nội tiếp (góc đỉnh góc đỉnh đối diện) Gọi J điểm GC cho GFJ  HFC Khi ta có: GJ FG   FG.HC  GJ FH HC FH GH FH GFH JFC ( g g )    GH FC  FH JC JC FC  FG.CH  GH CF  FH  GJ  JC   FH CG GFJ HFC ( g g )  Đây định lý Ptoleme Vậy FG.FH  GH CF  CG.HF 3) Trên BC lấy M cố định… MNP ABC ( g.g ) ANP ABC ( g.g ) Ta có: MNP  ANP(c.c.c)   Lại có: AM đường cao ANP Từ ta suy diện tích tam giác MNP nhỏ NP nhỏ Mặt khác NP AL AM   BC AD AD AM nhỏ D trùng M hay A giao điểm đường thẳng qua M vng AD góc với BC với nửa đường tròn (O; R) Câu Theo đề ta có: ab  bc  ca  Áp dụng BĐT Cô-si vào biểu thức tốn ta có: 2a 1 a  b b2  c c2    2a ab  bc  ca  a  b  b  ab  ac  bc c c  ab  ac  bc  2a  a  b  a  c  b  b  c  b  a  c  a  c  c  b   a a  ab ac  b b  4(b  c) a  b  c c  4(b  c) a  c Cộng vế theo vế ta được: 2a  b  c  a a b b c c      a  b a  c 4(b  c) a  b 4(b  c) a  c  a2 b2  c2  ab ac bc     11  a  b a  c 4(b  c) 4 Mặt khác: a 49b a 49b  2  ab 2 a 49c a 49c  2  ac 2 b  c   7bc   a  28b  28c   ab  ac  bc   P 19 7  4  15 a  a  7b  7c  15 Dấu "  " xảy    15 ab  bc  ca   b  c  15  15 a  19  15 Vậy MaxP    b  c  15  15 ĐẠI HỌC QUỐC GIA TP HCM TRƯỜNG PHỔ THÔNG NĂNG KHIẾU HỘI ĐỒNG TUYỂN SINH LỚP 10 ĐỀ THI TUYỂN SINH LỚP 10 Năm học : 2018-2019 Mơn thi: TỐN (khơng chun) Thời gian làm bài: 120 phút Bài (1 điểm) Biết  x  y         x  y y    x  y)  x y  x 2      x    Bài 2: (2 điểm) a) Giải phương trình  y x y   y     Tính x y x y   x  2x2   x   x( x  7) 3 x  x  3 x  1   y   x  3 b) Giải hệ phương trình  2  x  1 y  y    y   Bài 3: (2 điểm ) Cho phương trình x2  x  3m  11  0(1) a) Với giá trị m phương trình (1) có nghiệm kép ? Tìm nghiệm b) Tìm m để phương trình (1) có hai nghiệm phân biệt x1; x2 cho 2017 x1  2018x2  2019 Bài 4: (2 điểm) a) Đầu tháng năm 2018, vào vụ thu hoạch, giá dưa hấu bất ngờ giảm mạnh Nơng dân A cho biết sợ dưa hỏng nên phải bán 30% số dưa hấu thu hoạch với giá 1500 đồng kilogam ( 1500d / kg ), sau nhờ phong trào “giải cứu dưa hấu” nên may mắn bán hết số dưa lại với giá 3500đ/1 kg; trừ tiền đầu tư lãi triệu đồng (khơng kể cơng chăm sóc tháng nhà) Cũng theo ông A, sào đầu tư (hạt giống, phân bón….) hết triệu đồng thu hoạch dưa hấu Hỏi ông A trồng sào dưa hấu b) Một khu đất hình chữ nhật ABCD ( AB  CD) có chu vi 240 mét chia thành hai phần khu đất hình chữ nhật ABMN làm chuồng trại phần lại làm vườn thả để nuôi gà (M, N thuộc cạnh AD, BC) Theo quy hoạch trang trại nuôi 2400 gà, bình quân gà cần mét vng diện tích vườn thả diện tích vườn thả gấp lần diện tích chuồng trại Tính chu vi khu đất làm vườn thả Bài 5: (3 điểm) Tứ giác ABCD nội tiếp đường tròn (T) tâm O, bán kính R, CAD  450 , AC vng góc với BD cắt BD I, AD  BC Dựng CK vng góc với AD  K  AD  , CK cắt BD H cắt (T) E  E  C  a) Tính số đo góc COD Chứng minh điểm C, I, K, D thuộc đường tròn AC  BD b) Chứng minh A tâm đường trịn ngoại tiếp tam giác BHE Tính IK theo R c) IK cắt AB F Chứng minh O trực tâm tam giác AIK CK.CB  CF.CD ĐÁP ÁN Bài        x y  x y       x  y   x  y    x    x y   y x y  x  y  xy  x  y  xy y yx x   x  y  2x  y xy x  y  2( x  y )  3( x  y )   x  y  xy xy   x  y  xy   xy  x  y     x y 1  x  y  xy  xy  x  y  xy    x y  0 x y x  1 y x Vậy  y Bài a) Giải phương trình 2x2   x  Điều kiện: x  2x2   x  3 x  x  x  7  3 x  x( x  7) 2x2   x  3 x  x 7  x    x  (tm)   2x   x 7  x   1    x  ( ktm)  3 x   2x    (2)   x y   5 x y   x  x   x      x  (ktm)  2  2x    x   x   x      x  1 (tm) Vậy tập nghiệm phương trình S  0; 1 b)  x  3 x  1   y   x  3 (1)  2  x  1 y  y    y   (2) x    x  3   x 1  y   x  y 1 1   x  3  x  1   y      +) Với x  3 thay vào phương trình   ta có: 4 y  y    y   (vơ nghiệm VT  0;VP  0) +) Với x  y  thay vào phương trình (2) ta có:  y    y  (3) y2  y    y  2    y  y   y  (4) (3)  x  y       x; y   1;2   y  2 y   y    x      x; y    3;4   2  y  (tm)  y  5y   y  4y   4   Vậy nghiệm hệ phương trình :  x; y   1;2  ;  3;4  Câu 3: a) Phương trình (1) có nghiệm kép     4(3m  11)    12m  44   m  15 b  2a 15 Vậy với m  phương trình (1) có nghiệm kép, nghiệm kép x  15 b) Phương trình có hai nghiệm phân biệt     m  Gọi hai nghiệm phân biệt phương trình x1; x2 , theo định lý Vi-et ta có: (2)  x1  x2    x1 x2  3m  11(3) Theo giả thiết ta lại có 2017 x1  2018x2  2019, kết hợp: Khi phương trình (1) trở thành x  x   có nghiệm kép x1  x2     x1  x2   x   x2  x  1  x1   x2    2017 x1  2018 x2  2019   x2  2017 1  x2   2018 x2  2019  x2  Thay vào (3) ta có: 2  3m  11  m  (tm)  2   Thử lại : với m=3 ta có phương trình (1) ln có hai nghiệm phân biệt Vậy m  thỏa mãn yêu cầu toán Bài 4: a) Gọi số sào dưa nhà ông A x (sào) (ĐK: x  0) Số dưa thu hoạch 2x (tấn)  2000x (kg) Số dưa bán với giá 1500d / kg 30%.2000 x  600 x (kg ) Số dưa bán với giá 3500 đ/1kg 2000x  600x  1400x (kg) Do số tiền thu bán hết 2x dưa là: 600x.1500  1400x.3500  5800000x (đồng)  5,8x (triệu đồng) Số tiền đầu tư cho x sào dưa 4x (triệu đồng) Do trừ tiền đầu tư lãi triệu đồng nên ta có phương trình: 5,8x  x   1,8x   x  (tm) Vậy nhà ông A trồng sào dưa b) A M D B N C Để nuôi 2400 gà, cần 1m2 diện tích vườn thả diện tích vườn thả MNCD SMNCD  2400m2 SMNCD  800  m2   2400  800  3200 m2 Diện tích khu chuồng trại ABNM S ABNM  Diện tích khu đất ABCD S ABCD   Gọi chiều rộng AB chiều dài khu đất x(m) y(m) 0  x  y  Chu vi khu đất 240m nên ta có phương trình: 2( x  y)  240  y  120  x (1) Diện tích khu đất ABCD 3200m2 nên ta có phương trình xy  3200 Thay (1) vào (2) ta phương trình: x 120  x   3200  x  120 x  3200    x  40  x  80   (tm)  x  40  y  80  (ktm)  x  80  y  40  AB  CD  40m S 2400  MD  MNCD   60(m) CD 40 Chu vi khu vườn thả hình chữ nhật MNCD  MD  CD    60  40  200 (m) Vậy chu vi khu đất làm vườn thả 200m Bài F B C I O H K D E A a) Ta có COD  2.CAD (góc tâm góc nội tiếp chắn cung CD)  COD  2.450  900 ( gt ) nên góc CID  900 Vì AC  BD Vì CK  AD ( gt ) nên góc CKD  900  CID  CKD  900 (2)  Tứ giác CIKD có đỉnh I K nhìn cạnh CD góc 900 nên tứ giác nội tiếp đường trịn đường kính CD Vì AC  BD ( gt ) nên AID vuông cân I  IA  ID (1) Ta có góc CBD =góc CAD  450 (hai góc nội tiếp chắn cung CD) Tam giác CIB vuông I có CBI  CBD  450 nên tam giác CIB vuông cân I  IB  IC (2) Từ (1) (2)  IA  IC  IB  ID  AC  BD (Vì I thuộc đoạn AC I thuộc đoạn BD) b) Chứng minh A tâm…… ACK vuông K  ICH  ACK  900  CAK  900  450  450 Tam giác CIH vng I có ICH  450 (cmt) nên vng cân I  IC  IH (3) Từ (2) (3)  IB  IH  I trung điểm BH , mà AI  BH  AC  BD   AI trung trực BH (4) CIH vuông cân I  DHE  IHC (đối đỉnh)= 450 Mặt khác HED  CAD  450 (2 góc nội tiếp chắn cung CD)  DHE  HED  450  HDE vuông cân D Mà DK đường cao hạ từ đỉnh D HDE  DK trung trực HE  AK đường trung trực HE (5) Từ (4) (5)  A giao điểm trung trực BH trung trực HE  A tâm đường trịn ngoại tiếp BHE +) Tính IK theo R Ta có: IK đường trung bình BHE nên IK  BE Ta có BCH  BCI  ICH  450  450  900 (do BCI CHI vuông cân)  BOE  2.BCE  2.900  1800 (góc tâm góc nội tiếp chắn cung BE (T))  B, O, E thẳng hàng BE đường kính T   BE  2R  IK  BE R c) Chứng minh O trực tâm AIK Vì IA  ID, OA  OD  R nên OI trung trực AD  OI  AD  OI  AK (6) Tam giác CAK vng K có CAK  450 nên CAK vuông cân K  KC  KA Mặt khác OC  OA  R  OK trung trực AC  OK  KA (7) Từ (6) (7)  O giao điểm đường cao hạ từ I K AIK  O trực tâm AIK +) Chứng minh CK.CB=CF.CD Ta có: BAC  BEC (hai góc nội tiếp chắn cung BC (T)) Vì IK // BE (tính chất đường trung bình)  BEC  FKC (đồng vị)  BAC  FKC Tứ giác AFCK có hai đỉnh A K nhìn FC góc nên AFCK tứ giác nội tiếp  CFB  1800  CKA  900 (8) Vì ABCD tứ giác nội tiếp nên FBC  CDK (cùng bù với góc ABC) (9) Từ (8) (9)  FBC KDC ( g.g )  CF CB   CK CB  CF CD (đpcm) CK CD SỞ GIÁO DỤC – ĐÀO TẠO TỈNH TUYÊN QUANG ĐỀ THI CHÍNH THỨC KỲ THI TUYỂN SINH LỚP 10 THPT NĂM HỌC 2018-2019 Mơn thi: TỐN CHUN Thời gian: 150 phút Ngày thi: 03/06/2018 Câu a) Giải phương trình : x   x2  x    8x3   x2  x  y   b) Tìm nghiệm nguyên hệ phương trình :  2  16 x  xy  y  y   Câu Tìm tham số m để phương trình x2  2(m  1) x  m2  a) Có hai nghiệm phân biệt dương b) Có hai nghiệm x1  x2 thỏa mãn :  x1  m   x2  3m Câu a) Tìm nghiệm nguyên phương trình: 3x2  xy  y  5x   b) Cho a, b,c số nguyên Chứng minh a2016  b2017  c 2018 chia hết cho a2018  b2019  c2020 chia hết cho Câu Cho đường trịn (O) đường kính AB Từ điểm C thuộc đường trịn kẻ CH vng góc với AB (C khác A B) Đường trịn bán kính CH cắt (O) D E (D thuộc cung AC) Gọi N giao điểm DE CH Giao điểm DE với CA CB I K Chứng minh rằng: a) Hai tam giác CAD CDI đồng dạng b) N trung điểm CH Câu Cho số thực dương a, b, c thỏa mãn a  b  c  Tìm giá trị nhỏ biểu thức: M a  4a  b  4b  c  4c    a2  a b2  b c2  c ĐÁP ÁN Câu 1: a) Giải phương trình Ta có: 2 x    Điều kiện xác định: 4 x  x    x   8 x3    a; b   a  x   Đặt   ab   x  1  x  x  1  x3  b  x  x      Phương trình cho trở thành: a  3b   ab  (a  3)(1  b)   x  2 x   a  x        x  (tm) b   x  x    x(2 x  1)   x   Vậy tập nghiệm phương trình cho : S  4; ;0   b) Tìm nghiệm nguyên… Ta có:  x  x  y    x  x    y   4 16 x  xy  y  y   16 x  xy  y   y  x  x   16 x  xy  y   x  1   x  y    y 2  y2 1 x    x 1  4x  y    2 y 1   x 1  y  4x  x  y2 1 TH1: x  2 y  y   y  1    2y    y  y   y   18 y  27   y  y  18 y  20    y    y  y  y  10    y   x 1 TH2: x  y2  y  y  2( y  1)    2y   25  y  y   10 y  10  50 y  75   y  y  50 y  84    y    y  y  y  42    y   x 1 Vậy nghiệm nguyên hệ cho 1;2  Câu a) Có nghiệm dương phân biệt Ta có: Để phương trình cho có hai nghiệm dương thì: 4(m  1)  4m2  8m         m    2m      S   2(m  1)  P   m  m    m  b) Có hai nghiệm phân biệt……  x1  x2   m  1 (2)  x1 x2  m Áp dụng định lý Vi-et ta có:  Mặt khác :  x1  m   x2  3m  x12  2mx1  m  x2  3m  x12  2(m  1) x1  x1  m  x2  3m  x12   x1  x2  x1  x1  m2  x2  3m (1)   x1 x2  x1  x2  x1 x2  3m  x1  x2  3m (3)  x1  x2  2(m  1) x  m   2 x1  x2  3m  x2  m  Từ (1) (3) ta có:  Thay vào (2) ta có:  m  2 m  4  m2  2m    m  (tm) Vậy m  Câu a) Tìm nghiệm ngun… Ta có: Phương trình cho tương đương với 3x  xy  y  x    3x  x   xy  y  3x  x   y (2 x  1) 3x  x  y ( Do x   x   0) 2x 1 Do x, y nguyên nên:  x  x    x  1  3  x  1  x  1    x  x     x  1   x  1      20 x    4 x  1   x  1   8 x    x  1    x    x  1   x  1   x  1 2 x   x   y     x   1  x   y  2 Vậy nghiệm nguyên  x; y  phương trình cho 1;0 ;  0; 2 b) Cho a, b,c số nguyên… Ta có: a 2018  b 2019  c 2020   a 2016  b 2017  c 2018   a 2016  a  1  b 2017  b  1  c 2018  c  1  a 2015 a(a  1)(a  1)  b 2016 (b  1)(b  1)  c 2017 c(c  1)(c  1) Ta có tích số tự nhiên liên tiếp chia hết cho 6, có số chẵn số chia hêt cho Do vậy: a(a  1)(a  1); b(b  1)(b  1); c(c  1)(c  1) chia hết  a 2018  b2019  c 2020   a 2016  b2017  c2018    Vậy ta có điều phải chứng minh Câu C K E N D A I H B a) Hai tam giác CAD CDI đồng dạng Ta có: Do đường trịn đường kính CH cắt (O) D E nên CD  CE Do C điểm cung DE Từ đó:  CDE  CAD (hai góc nội tiếp chắn hai cung nhau)  CDE  CAD Do vậy:    DCA chung  CAD b) N trung điểm CH Ta có: CAD CDI CDI ( g.g ) (cmt ) CA CD   CD  CA.CI  CH CD CI CA CH CA.CI  CH   CH CI   CIH CHA  c.g c   CIH  CHA  900  IH  CA Chứng minh hoàn toàn tương tự ta có HK vng với CB  Tứ giác CIHK hình chữ nhật (tứ giác có góc vng) Suy hai đường chéo CH IK cắt trung điểm đường Vậy N trung điểm CH IK Câu Ta có: Áp dụng BĐT Co si BĐT quen thuộc : 1    x y z x yz a  4a  b  4b  c  4c  M   a2  a b2  b c2  c a  1  4a  b  1  4b  c  1  4c     a  a  1 b  b  1 c(c  1) 2a  4a 2b  4b 2c  4c 1  6.9 54     6    9  a(a  1) b(b  1) c(c  1)  a 1 b 1 c 1 a  b  c  Vậy giá trị nhỏ M 9, dấu xảy a  b  c  Co  si  SỞ GIÁO DỤC – ĐÀO TẠO TỈNH THÁI NGUYÊN KỲ THI TUYỂN SINH LỚP 10 THPT NĂM HỌC 2018 – 2019 Mơn thi: TỐN CHUN ĐỀ THI CHÍNH THỨC Thời gian: 150 phút 3 5 Câu Rút gọn biểu thức A    52 1   x2  y   x Câu Giải hệ phương trình :  3   x  12 x  y  x  Câu Tìm x, y nguyên dương thỏa mãn: 16  x3  y3   15xy  371 Câu Giải phương trình x    x  3x2 12 x  14 Câu Cho x, y, z số thực dương Chứng minh x2 x  y  14 xy 2  y2 y  3z  14 yz 2  z2 z  3x  14 xz 2  x yz Câu Cho tam giác ABC cân có BAC  1000 , D thuộc nửa mặt phẳng không chứa A bờ BC, CBD  150 , BCD  350 Tính ADB Câu Cho tam giác ABC nội tiếp (O), AB < AC , đường cao BD, CE cắt H Gọi M trung điểm BC, MH cắt (O) N a) Chứng minh A, E, D, H, N thuộc hình trịn b) Lấy P đoạn BC cho BHP  CHM , Q hình chiếu vng góc A lên HP Chứng minh DENQ hình thang cân c) Chứng minh (MPQ) tiếp xúc (O) ĐÁP ÁN Câu Rút gọn biểu thức A 3 5   52 1   1 1 1  52 1   2 1   5 2 3  2 1 1  2 3 3     3 7  95 3  21    4  Câu 2  (1) x  y   4x 3   x  12 x  y  x  a) Giải hệ phương trình:  (2) Ta có phương trình (2) tương đương với: x3  x  12 x   y    x    y  Hệ cho trở thành: 2  x  2  y   x  y   4x    3 3  x  12 x  y  x     x    y   y  z    y  z 2  yz  Đặt x   z ta có:  3 2  y  z    y  z   y  yz  z    y  z 1  yz   a  y  z ta có hệ phương trình: b  yz  Đặt   a2 1 b    a2  a2   a  2b   b  b     2  a 1  b   a 1  a    2a  a   2  a  2a          a2  a  y  z 1 b     b   yz  a    y    x   z   x    y, z hai nghiệm phương trình : x  x     y  x      z   x  Vậy hệ phương trình có tập nghiệm  3;0 ; 2;1  Câu 3: Vì x, y nguyên dương nên VP  VT  nên x  y x  y 1 Ta có: 15xy  16  x3  y3   371 số lẻ nên x, y lẻ, :  Xét x  y  thay vào phương trình thỏa mãn Xét x  Ta có: x   y  16  x3  y   16  x3   x     16  x  12 x     15 xy  371  15 x  x    371  15 x  30 x  371 16  x  12 x    15 x  30 x  371  81x  162 x  243  81 x  x   Ta có: x2  x   0, x   16  x3  y3   15xy  371 Vậy trường hợp vô nghiệm Vậy phương trình có cặp nghiệm ngun dương  x; y    3;1 Câu Giải phương trình x    x  3x2 12 x  14 Điều kiện xác định:  x  2 x    x  3x  12 x  14  x    x  1   x    x   3x  12 x  12 2x   x2  2x   2x   x  x  9    3 x  2 2x   x 1  2x   x   x  2 2x   x 1   x  2  2x   x  3 x  2  x      1   0  2x   x   2x   x 1 x2 (do3    0,   x  ) 2 2x   x   2x   x Vậy phương trình có nghiệm x  2 Câu Áp dụng BĐT Co –si ta có: x2  y  xy  x  y  14 xy  x  y  12 xy  xy  x  y  12 xy  x  y  x  y  14 xy  x  12 xy  y   3x  y   3x  y 2   y  3z  14 yz  y  z Chứng minh hồn tồn tương tự ta có:   z  3x  14 xz  3z  x  a b2 c2  a  b  c  Áp dụng BĐT Cauchy Schawz: ta được:    x y z x yz  x  y  z x2 y2 z2 x yz  VT      3x  y y  z 3z  x 3x  y  y  z  z  x Vậy ta có điều phải chứng minh Dấu "  " xảy x  y  z Câu 6: E A B C D Gọi E điểm đối xứng với B qua A  EA  AB Vì ABC cân A  AB  AC  AB  AC  AE  A tâm đường tròn ngoại tiếp tam giác BEC  BCE  900 1800  BAC 1800  1000 Ta có: ABC  ACB    400 2 Lại có: DBE  DCE  150  400  350  900  1800  BECD tứ giác nội tiếp (dấu hiệu nhận biết)  AD  AB  AC  AE  ABD cân A  ABD  ADB  15o  400  550 Câu A N Q D O E H B P C M K a) Chứng minh A,E,D,H,N thuộc đường trịn Kẻ đường kính AK  ACK  900 (góc nội tiếp chắn nửa đường trịn) hay AC  KC ( gt ) Lại có: BH  AC  D  KC / / BH (từ vng góc đến song song) Chứng mnh tương tự ta có: BK / / HC   AB   BHCK hình bình hành (dấu hiệu nhận biết) Lại có: M trung điểm BC, HK  BC  M   M trung điểm HK Ta có: ADH  AEH  900  900  1800  ADEH tứ giác nội tiếp  A, D, E, H thuộc đường trịn đường kính AH Lại có: ANK  900 (góc nội tiếp chắn nửa đường trịn)  ANK nhìn đoạn AH góc 900  N thuộc đường trịn đường kính AH  A, N , D, E, H thuộc đường trịn đường kính AH b) Lấy P đoạn BC… Ta có: AQH  90 nên Q nhìn AH góc vuông  Q A, E, D, H, N thuộc đường trịn đường kính AH Mặt khác : BHP  QHD (hai góc đối đỉnh) CHM  EHN (hai góc đối đỉnh) Mặt khác, xét đường trịn đường kính AH ta có: QHD góc nội tiếp chắn cung QD, EHN góc nội tiếp chắn xung NE 1  sdNE  sdQD  NE  QD (tính chất dây căng cung) 2 Mà tứ giác DENQ tứ giác nội tiếp có cạnh NE  QD  DENQ hình thang cân c) Chứng minh (MPQ) tiếp xúc (O) Ta chứng minh tứ giác MNPQ nội tiếp Ta có: BEC  BDC  90  BEDC tứ giác nội tiếp  MCH  HDE  sdEB (các góc nội tiếp chắn cung EB) 2 Lại có: HDE  sdEH (trong đường trịn đường kính AH) Ta có: HMP  CHM  MCH (tính chất góc ngồi tam giác) 1 1  HMP  EHN  sdEH  sdNE  sdEB  sdNH  NQH 2 2  MNPQ tứ giác nội tiếp  BHP  CHK  NKB  BHP  NBP BN HB KC   BP KN KN NKC (c  g  c) NKP( g.g )   BNP  KNC (Các góc tương ứng) Kẻ tiếp tuyến Nx (O) Ta có ngay: PNx  BNx  BNP  NCM  KNC  NCP Do Nx tiếp tuyến (MPQ) Vậy (MPQ) tiếp xúc với (O) Nx (ta có điều phải chứng minh) SỞ GIÁO DỤC – ĐÀO TẠO TỈNH BẮC NINH KỲ THI TUYỂN SINH LỚP 10 THPT NĂM HỌC 2018-2019 Môn thi: TỐN CHUN Thời gian: 150 phút ĐỀ THI CHÍNH THỨC Câu  a  a  b2 a  a  b2   a  a  b2 a  a  b2  a) Rút gọn biểu thức : P    a  a 2b , a  b 0 :  b  b) Cho phương trình: x2  ax  b  với x ẩn, a, b tham số Tìm a, b cho  x1  x2  phương trình có nghiệm thỏa mãn  3  x1  x2  35 Câu a) Giải phương trình: x   3x   x  b) Cho số thực a, b, c thỏa mãn  a, b, c  2, a  b  c  Tìm GTLN GTNN P  a  b2  c ab  bc  ca Câu a) Tìm cặp số nguyên x, y thỏa mãn x2  y  b) Chứng minh hiệu lập phương số nguyên liên tiếp bình phương số tự nhiên n n tổng số phương liên tiếp Câu 1) Từ A (O) vẽ tiếp tuyến AB, AC (B, C tiếp điểm) AO cắt BC H Đường trịn đường kính CH cắt (O) điểm thứ hai D Gọi T trung điểm BD a) Chứng minh ABHD tứ giác nội tiếp b) Gọi E giao điểm thứ đường trịn đường kính AB với AC, S giao điểm AO với BE Chứng minh TS // HD 2) Cho (O ),  O2  cắt hai điểm A, B Gọi MN tiếp tuyến chung đường tròn với M, N thuộc (O ),  O2  Qua A kẻ đường thẳng d song song với MN cắt (O ),  O2  ,BM, BN C, D, F,G Gọi E giao điểm CM DN Chứng minh EF = EG Câu Cho 20 số tự nhiên, số có ước ngun tố khơng vượt q Chứng minh chọn số cho tích chúng số phương ĐÁP ÁN Câu a) Rút gọn biểu thức  a  a  b2 a  a  b2 P   a  a  b2 a  a  b2       a  b  a  a  a  b2 a  2  a  a  b2 a  b2    a  a 2b , :  b   a  b 0 b2 a  a  b2  a  a  b  a a  b  a  a  b  2a a  b a   a  b2   b2 a a  b 4a a  b b2 a a  b2   b2 a a  b a a  b2  a  b2 a    a  b2   a  b2 a0  2  a b b) Cho phương trình……… Để phương trình cho có hai nghiệm x1; x2    a2  4b   x1  x2  a (1) (2)  x1 x2  b Áp dụng định lý Vi-et ta có:  Theo đề ta có:  x1  x2   x1  x2    3   x  x  35   x1  x2   x1  x2   x1 x2   35  x1  x2  (3)  x1  x2       5  x1  x2   x1 x2   35  x1  x2   x1 x2  (4) Thế (1) (2) vào (4) ta được:  a   b   a  b   b  a  (*) Bình phương hai vế (3) ta được:  x1  x2   52   x1  x2   x1 x2  25  a  4b  25  a   b  6  a  4a  28  25  a     a  1  b  6 Vạy  a; b   1; 6  ;  1; 6  Câu a) Giải phương trình x   3x   x  Ta có điều kiện xác định: x  1   a  x  a  x  Đặt  Khi ta có hệ phương trình sau đây:  a, b     b  x   b  x     a  b  a b  a  a   2 2 3a  b  3a   a  a   (*)  *  3a  a  2a  a   a  2a  2a    a (a  2)   a   a      a    a  2a      a   a    a  2a     a  2(tm)   a  2(ktm)  b  a2  a     x   a2  x      x 1  x  x   b     (TM ) Vậy phương trình có nghiệm x  b) Cho số thực a,b, c… a  b  2ab  Áp dụng BĐT Co si ta có: b  c  2bc c  a  2ca  1  a  b  c   a  b  a  c  b  c    2ab  2ac  2bc  2 2  a  b  c  ab  ac  bc a  b2  c2 1 ab  bc  ca a  b  c Dấu “=” xảy    a  b  c 1 a  b  c  P Vậy MinP  1khi a  b  c  Theo đề ta có:  a, b, c    a   b   c     abc   ab  ac  bc    a  b  c     abc   ab  ac  bc   12     ab  ac  bc    abc   ab  bc  ca  P P a  b  c  2ab  2ac  2bc 2 ab  ac  bc a  b  c ab  ac  bc 2 2 2  a    b  c   b   abc     a  c  Dấu "  " xảy   a  b  c    c   a  b   0  a, b, c  Vậy MaxP  abc  0, a  b  c  3,0  a, b, c  2 Câu a) Tìm cặp số ngun tố… Ta có số phương chia cho nhận số dư nên ta có: (3k )  9k  2 (3k  1)  9k  6k   1 mod 3  2  3k    9k  12k   1 mod 3 Nếu x, y  x,y khơng chia hết cho số dư Vế trái cho  2.1  1 chia dư vô lý x  y   hai số x, y phải có số  x    y   y   y  2( y  0)  2  y   x  2.9   x  19  x  Vậy cặp số nguyên  x; y    3;2  b) Chứng minh hiệu lập phương… Gọi số tự nhiên liên tiếp a, a  1 a   , theo đề ta có:  a  1  a3  n2  a3  3a  3a   a3  n2  3a  3a   n2  a   n   02  12  a  (tm) +)Xét TH: 1  a  ta có:  2  a  1  n     a  1 (tm) a  +)Xét TH:    2a   3a  3a    2a  1  a  1 2 Vậy ta có n tổng hai số phương liên tiếp Câu (*) B J T A H S O D E C Bài a) Chứng minh ABHD nội tiếp Gọi I, J tâm đường trịn đường kính CH, AB Xét (J) ta có: ADB góc nội tiếp chắn nửa đường trịn  ADB  900 Ta có: AB, AC hai tiếp tuyến đường tròn (O) tiếp điểm B, C cắt A Và AO  BC  H  AO  BC H hay AHB  900 (tính chất hai tiếp tuyến cắt nhau) Xét tứ giác ABHD ta có: ADB  AHB  900 (cmt )  ABHD tứ giác nội tiếp b) Gọi E giao điểm thứ đường trịn… Vì tứ giác ABHD tứ giác nội tiếp (cmt)  DBH  DAH (hai góc nội tiếp chắn cung DH) Xét đường trịn (I) ta có: HDC góc nội tiếp chắn nửa đường tròn  HDC  900  BDA  HCD  900   ADH  ADB  BDH  90  BDH  Lại có:  BDC  BDH  HDC  900  BDH  ADH  BDC Xét ADH BDC ta có: HAD  DAC (cmt ); ADH  BDC (cmt )  ADH BDC ( g.g ) AD AH (các cặp cạnh tương ứng)  BD BC AD BD 2.TD TH (T trung điểm BD)     AH BC 2.HC HC AD TD Xét TAD CAH ta có:  (cmt ); TDA  CHA  900 AH CH   TAD CAH (c.g.c)  TAD  HAC (hai góc tương ứng)  TAD  TAS  HAD Mà    HAC  HAD  DAE  TAS  DAE Mặt khác : DAE  DBE (Hai góc nội tiếp chắn cung DE)   TAS  SBT  EAD   ABTS tứ giác nội tiếp  STD  BAS (góc ngồi đỉnh góc đỉnh đối diện) Mà BAS  BDH (hai góc nội tiếp chắn cung BH đường tròn (J))   STD  TDH  BAH  Lại có hai góc vị trí so le  ST / / HD(dpcm) Bài G C H A I F D B M K N E Gọi MO1  d  H ; NO2  d  I , AB  MN  K O1M  CD  H  O2 M  CD  I  Ta có : MN//CD    O1M , O2 N trung trực CA DA (đường kính dây cung)  CH  HA, MHA  90    IA  ID, NID  90    MNIH hình chữ nhật M  H  I  900  HI  MN  CD  MN / / CD  Xét CED ta có:  (cmt ) MN  CD    MN đường trung bình CED  M , N trung điểm EC, ED  MC  ME, ND  NE Xét CAE ta có: M , H trung điểm CA, CE (cmt)  AM đường trung bình CAE  MN / / AE Mà MH  CD (cmt )  AE  CD (từ vng góc đến song song) Xét MKA BKM ta có: MAK  KMB (góc nội tiếp góc tạo tiếp tuyến dây cung chắn cung AB) MKA chung  MKA BKM ( g.g )  MK KA   KM  KA.BK BK KM (1) Xét NKA BKN ta có: NAK  KNB ( góc nội tiếp góc tạo tiếp tuyến dây cung chắn cung AB) NKA chung NK KA   KN  KA.BK (2) BK KN Từ (1) (2) suy KM  KN Do MN / / FG , áp dụng định lý Ta let ta có: KN MK KB    AG  AF AG AF AB (cmt )  EG  EF (tính chất đường trung trực) (dpcm) Mặt khác AE  FG  NKA BKN ( g.g )  Câu Ta có : số có ước ngun tố khơng vượt q có dạng 2x.3y.5z.7t Do x, y, z, t số có trường hợp chẵn, lẻ nên số có tổng cộng 2.2.2.2  16 trường hợp x, y, z, t  20  Theo nguyên lý Dirichle, tồn   1  số a, b saao cho  16  x y z t  a  1.3 1.5 1.7 số mũ tương ứng tính chẵn lẻ  x2 y2 z2 t2  b   x1  x2  2m  y  y  2n    a.b   2m.3n.5 p.7 q   z1  z2  p t1  t2  2q Đây số phương Vậy ta ln chọn số cho tích chúng số phương từ 20 số tự nhiên mà số có ước ngun tố khơng vượt SỞ GIÁO DỤC VÀ ĐÀO TẠO HẢI DƯƠNG ĐỀ CHÍNH THỨC ĐỀ THI TUYỂN SINH LỚP 10 THPT CHUYÊN NGUYỄN TRÃI NĂM HỌC 2016 - 2017 Môn thi: TỐN (Chun) Thời gian làm bài: 150 phút, khơng kể thời gian giao đề (Đề thi gồm có 01 trang) Câu (2,0 điểm) a  x2 a  x2 a) Rút gọn biểu thức: A  2 a   a với a  0, x  x x b) Tính giá trị biểu thức P  ( x  y)3  3( x  y)( xy  1) biết: x  3  2  3  2 , y  17  12  17  12 Câu (2,0 điểm) a) Giải phương trình: x2   x3  x      x  x2  2x   y  y    b) Giải hệ phương trình:   x  3xy  y  Câu (2,0 điểm) a) Tìm dạng tổng quát số nguyên dương n biết: M = n.4n + 3n chia hết cho b) Tìm cặp số (x; y) nguyên dương thoả mãn: (x2 + 4y2 + 28)2  17(x4 + y4) = 238y2 + 833 Câu (3,0 điểm) Cho đường trịn tâm O đường kính BC, A điểm di chuyển đường tròn (O) (A khác B C) Kẻ AH vng góc với BC H M điểm đối xứng điểm A qua điểm B a) Chứng minh điểm M nằm đường tròn cố định b) Đường thẳng MH cắt (O) E F (E nằm M F) Gọi I trung điểm HC, đường thẳng AI cắt (O) G (G khác A) Chứng minh: AF2 + FG2 + GE2 + EA2 = 2BC2 c) Gọi P hình chiếu vng góc H lên AB Tìm vị trí điểm A cho bán kính đường trịn ngoại tiếp tam giác BCP đạt giá trị lớn Câu (1,0 điểm) Cho a, b, c số thực dương thay đổi thỏa mãn: a + b + c = Tìm giá trị nhỏ biểu thức : Q  14(a  b2  c )  ab  bc  ca a 2b  b2c  c 2a Hết -Họ tên thí sinh: Số báo danh: Chữ kí giám thị 1: Chữ kí giám thị 2: SỞ GIÁO DỤC VÀ ĐÀO TẠO HẢI DƯƠNG ĐÁP ÁN VÀ HƯỚNG DẪN CHẤM MƠN TỐN ĐỀ THI TUYỂN SINH LỚP 10 THPT CHUYÊN NĂM HỌC 2016 - 2017 (Hướng dẫn chấm gồm: 04 trang) Nếu học sinh có cách làm khác cho điểm tối đa Câu Ý Nội dung a a  x2 a  x2 2 a   a với a  0, x  x x Rút gọn biểu thức: A  a  x2  2x a a  x2  2x a A  = x x  x a x a x x  a x  x  a Điểm 1,00 0,25 x 0,25 +) Với x  a x  a  x  a nên A =   x  a  x  a 2x  2 x x x 0,25 +) Với  x  a x  a   x  a  a  x nên A = 0,25 a xx a a  x x Tính giá trị biểu thức: P  ( x  y)3  3( x  y)( xy  1) biết: b x  3  2  3  2 , y  17  12  17  12 Ta có: x3   3 2  3 2      3 2 3 2 3 3 2 32 1,00 32  32   x3   3x  x3  3x  (1) Tương tự: y3  y  24 (2) Trừ vế với vế (1) (2) ta được: x3  y3  3( x  y)  20  (x - y)3 + 3(x - y)(xy + 1) = 20 Vậy P = 20 0,25 0,25 0,25 0,25 a Giải phương trình: x2   x3  x  (1) 1,00 +) ĐK: x  1 PT (1)  (x2 - 3x + 3) + 3(x + 1) = (x  1)(x  3x  3) (2) 0,25 Do x2 - 3x + > nên (2)   3(x  1) x 1 4 x  3x  x  3x  t  x 1 2 Đặt t  ; t  PT: + 3t = 4t  3t - 4t + =   (TM) t  x  3x   x 1 +) Với t = PT:   x  4x    x   2 x  3x  x 1 +) Với t = PT:   x  12x    x   42 x  3x  3     x  x  x   y  y   (1) b Giải hệ phương trình:   x  3xy  y  (2)   Ta có: (1)  x  x  2x   (Do y2   y    y2   y  0,25 0,25 0,25 1,00 y2   y  0,25 y2   y  với y)  x   (x  1)2    y  y2   x  y 1 (x  1)2  y (x  1)2   y  0   x 1 y  (x  y  1) 1  0 2   (x  1)   y    x  y    2  (x  1)   (x  1)  y   y  (3) Do a (x  1)2   x   x  1, x 0,25 y2   y   y, y nên (3) vô nghiệm x  Thay y = - x - vào (2) tìm nghiệm  x     1 Với x =  y = -2; x =   y  Vậy hệ có nghiệm (1;-2),   ;  3  3 n n Tìm dạng tổng quát số nguyên dương n biết: M = n.4 + chia hết cho 2k 2k k k k 0,25 0,25 1,00 k +) n = 2k (k nguyên dương): M = 2k.4 + = 2k.16 + Ta có: 16 dư 0,25 với 2k chia  M dư với (2k.2k + 2k) = 2k.(2k + 1) chia  (2k + 1) chia hết cho  k chia 0,25 dư 3, hay k = 7q +  n = 14q + (q  N ) +) n = 2k + (k nguyên dương): M = (2k + 1).42k + + 32k+1 = 4(2k+1).16k + 3.9k  M dư với (k + 4).2k + 3.2k = (k + 7).2k chia  k chia hết cho  k = 7p (p  N ) Vậy n = 14q + n = 14p + 1, với p q số tự nhiên Tìm cặp số (x; y) nguyên dương thoả mãn: b 0,25 0,25 1,00 (x2 + 4y2 + 28)2 - 17(x4 + y4) = 238y2 + 833 Ta có:  x  y  282  17( x  y )  238 y  833   x  4( y  7)   17  x  ( y  7)2  0,25  16 x4  8x ( y  7)  ( y  7)2    x  ( y  7)    x  y   0,25 Vì x, y  N * nên x  y  x  y x  y  Do từ (1) suy ra: 0,25  (2 x  y)(2 x  y)  (1) 2 x  y   x    x  y   y  0,25 KL: (x; y)=(2; 3) thoả mãn toán a Chứng minh điểm M nằm đường tròn cố định A 1,00 F S B C K H O I E G D M Lấy K điểm đối xứng O qua B, B O cố định nên K cố định Tứ giác OAKM hình bình hành nên KM = OA BC không đổi OA  BC  M nằm đường trịn tâm K, bán kính b Chứng minh tổng bình phương cạnh tứ giác AEGF không đổi 0,25 0,25 0,25 0,25 1,00 Xét  AHB  CHA có BHC = BHA =900, BAH = ACB (cùng phụ với ABC )   AHB đồng dạng  CHA Gọi S trung điểm AH, I trung điểm HC nên 0,25  ABS đồng dạng  CAI  ABS = CAI Ta lại có BS đường trung bình  AMH  BS//MH  ABS = AMH  AMH = CAI 0,25 Mà CAI + MAI =900  AMH + MAI =900  AI  MF Xét tứ giác AEGF nội tiếp (O), có AG  EF Kẻ đường kính AD, GD  AG EF  AG nên EF // GD, tứ giác nội tiếp 0,25 EFGD hình thang cân  FG = ED  AE2 + FG2 = AE2 + ED2 = AD2 = BC2 Tương tự ta chứng minh được: AF2+ EG2 = BC2 0,25 Vậy AE2+ FG2 +AF2+ EG2 = 2BC2 Gọi P hình chiếu vng góc H lên AB Tìm vị trí điểm A cho bán kính c đường tròn ngoại tiếp tam giác BCP đạt giá trị lớn 1,00 Gọi Q hình chiếu H AC  Tứ giác APHQ hình chữ nhật (S tâm)  AQP  AHP  ABC nên tứ giác BPQC nội tiếp A Q S P B H O 0,25 C O' Đường trung trực đoạn thẳng PQ, BC, QC cắt O’ O’ tâm đường 0,25 tròn ngoại tiếp tam giác BCP Có: OO’ // AH vng góc với BC OA  PQ O'S  PQ  O’S//OA nên tứ giác ASO’O hình bình hành AH  OO’ = AS = AH Trong trường hợp A nằm cung BC ta có: OO’ = AS = 0,25 Tam giác OO’C vuông O nên O’C = OC2  AH Do OC không đổi nên O’C lớn 0,25 AH lớn  A cung BC Cho a, b, c số thực dương thay đổi thỏa mãn: a + b + c = 1,00 Tìm giá trị nhỏ biểu thức: P  14(a  b2  c )  ab  bc  ca a 2b  b2c  c 2a Ta có: a2 + b2 + c2 = (a + b + c)(a2 + b2 + c2) = a3 + b3 + c3 + a2b + b2c + c2a + ab2 + bc2 + ca2 Theo bất đẳng thức Cô si: 0,25 a + ab  2a b; b + bc  2b c; c + ca  2c a  a + b + c  3(a b + b c + c a) 2 Do đó: P  14(a  b2  c )  2 2 2 3(ab  bc  ca) a  b2  c Đặt t = a2 + b2 + c2 Ta ln có: 3(a2 + b2 + c2)  (a +b + c)2 = Do vậy: t  Khi đó: P  14t  3(1  t )  t  27t     27t   23 2t 2 2t 2 2t Vậy MinP = 23 a = b = c = 3 0,25 0,25 0,25 SỞ GIÁO DỤC VÀ ĐÀO TẠO KIÊN GIANG ĐỀ CHÍNH THỨC KỲ THI TUYỂN SINH VÀO LỚP 10 THPT CHUYÊN NĂM HỌC 2018-2019 Mơn thi: TỐN (chun) Thời gian: 150 phút Bài (2,0 điểm)  x x 2 x  1 x a) Rút gọn biểu thức : A   (với x  0, x  1, x  4)   :  x x 2 x2 x  2 x b) Giải phương trình : x2  x    x  5 x   Bài (1, điểm) Cho Parabol ( P) : y  x2 đường thẳng (d ) : y  2mx  4m (với m tham số) Tìm tất giá trị tham số m để (d ) cắt ( P) hai điểm phân biệt có hồnh độ x1; x2 thỏa mãn x1  x2  Bài 3: (1,0 điểm) Cho tam giác AMB cân M nội tiếp đường tròn (O; R) Kẻ MH  AB  H  AB  Biết AM  10 cm, AB  12 cm Tính độ dài MH bán kính R Bài (1,0 điểm) A B Một người muốn làm quạt có chu vi 80 cm (hình minh họa – phần quạt AOB) Tính số O đo góc cho diện tích quạt lớn Bài (2,0 điểm) Cho điểm A cố đinh, xAy  600 điểm B nằm xAy  B  Ax, B  Ay  Gọi M, N hình chiếu B Ax Ay Đường thẳng BN cắt Ax H đường thẳng BM cắt Ay K a) Chứng minh HK  2MN b) Gọi I, D trung điểm AB, HK Chứng minh tứ giác MIND nội tiếp c) Giả sử AB  cm, gọi O trung điểm MN Tính độ dài IO Bài (1,0 điểm) Cho ba số dương x, y, z thỏa mãn điều kiện x  y  z  Chứng minh x2 y2 z2   1 yz zx x y ĐÁP ÁN Bài a) ĐKXĐ: x  0, x  1;4             x   x 2 x 1 x x   x 1  x x   x  x x 2   x 2 x 1 x 2 x 1  x 1  x x 2       x  x  2x  x   x  2 x  1 x x     x 2  x 1   2   x 1  x 1  x 1  2 x 1  x x 2  1 x x A  : x  x  x  x   2 x 2 A x 1 Vậy Giải phương trình b) x  x    x  5 x   x  1 ĐKXĐ: Với điều kiện ta có: x2  x    x  5 x    x  x  x   x     x   x   x   12      x  x     x   x   x   x   x   12      x     x   x   x  1  x   x   12         x   x   x   x   12  Đặt (*) x   x 1  t ta có: t  t  4 *  t  t     t  3 t      t 3 +) Với , ta có:  x  x   x 1   x 1  x 1    x  1  x  x  x  x         x   x  3(tm)  x  2x   x 1   x  x  3   x   Với t=4, ta có:   x  6 x   x   4  x   x      x    x   x  6  x  6      x  12 x  36  x    x  11x  35  VN  x3 Vậy phương trình cho có nghiệm Bài 2: Hoành độ giao điểm (d) (P) nghiệm phương trình: x2  2mx  4m  x  2mx  4m  *  pt (*) (d) cắt (P) hai điểm phân biệt có hai nghiệm phân biệt m    '   m  4m    m   x1  x2  2m  Áp dụng định lý Vi-et ta có:  x1 x2  4m Theo đề ta có: x1  x2    x1  x2    x12  x22  x1 x2    x1  x2   x1 x2  x1 x2    2m   2.4m  4m   4m  8m  m  (1) )m   1  4m  8m  8m   m   (ktm)  m   )m   (1)  4m  8m  8m    2m   2m  1    m m    m Vậy với Bai x1  x2  x1; x2 (d) cắt (P) hải điểm phân biệt có hoành độ thỏa mãn M E O A B H Áp dụng định lý Pytago cho tam giác MAH vuông H ta có:  AH  MH  AM 2  AB   12   MH  AM  AH  AM     10        2 2 Kẻ OE vng góc với MA suy E trung điểm MA (tính chất đường kính-dây cung)  ME  EA  MA  5cm Xét tam giác MOE tam giác MAH có: MEO  MHA  900 Góc AMH chung;  MOE Bài 5: MAH ( g  g )  OM ME ME.MA 5.10   R  OM    6, 25  cm  MA MH MH Độ dài cung tròn tạo nên hình quạt chu vi C  2R  l  2R  Suy chu vi hình quạt là: l  R 1800  R 180  80 14400 360  R  lR  R 2 Squat   360 Diện tích hình quạt :  R  14400 360  2 S     40 R  R    R  20   400  S  400 360   R    R  20   R  20  360 R   R  14400    Dấu “=” xảy 14400 360 360 2     2(rad ) 20    AOB  2(rad )   Vậy Bài K y D N B O I A M x a) ANH Xét vuông N có: cos NAH  cos 600  AN  AH Xét tứ giác MNKH có: NKM  NHM  300 (do phụ với góc NAH) Mà hai góc nhìn đoạn NM Suy MNKH tứ giác nội tiếp đường tròn ANM  AHK Nên Xét ANM (góc góc ngồi đỉnh đối diện) AHK H ANM  AHK (cmt ) NAH Có: chung;  ANM AHK ( g  g )  AN NM    HK  NM (dpcm) AH HK b) Gọi I, D lần lượt… ANB  AMB  900  900  1800 Xét tứ giác MANB có suy MANB tứ giác nội tiếp ANB  AMB  90  MANB Vì tứ giác nội tiếp đường trịn tâm I đường kính AB  AB   I;  Xét đường trịn   ta có: NAM góc nội tiếp chắn cung MN NIM góc tâm chắn cung MN  MIN  NAM  2.600  1200 KNH  KMH  900  MNKH Ta có: KH tứ giác nội tiếp đường trịn tâm D, đường kính  HK   D;   ta có: Xét đường trịn  NKM góc nội tiếp chắn cung MN NDM góc tâm chắn cung MN  MDN  NKM  2.300  600 NDM  NIM  1200  600  1800 Xét tứ giác MIND có Suy tứ giác MNKH nội tiếp đường tròn (đpcm) c) Giả sử… Nhận thấy hai tam giác vuông ANB, AMB có IN, IM hai đường trung tuyến ứng với cạnh huyền Tương tự ta có:  IN  IM  ND  MD  KH AB  4cm IM  IN (cmt ); ND  MD(cmt ), ID chung Xét tam giác IDN tam giác IMD có:  IDN IMD(c.g.c)  NID  MID  ID NIM phân giác MIN Xét cân I có ID phân giác góc NIM nên ID đồng thời trung trực cạnh MN, suy ID cắt MN trung điểm MN Mà có O trung điểm MN suy ba điểm I, O, D thẳng hàng  NIO  NID  1200 NIM   600 2  IO  IN cos NIO  4.cos 600  2cm Bài Áp dụng bất đẳng thức Bunhiacopxki với ba số    A B C  a; b; c ; , ,   a b c  có:   A 2  B 2  C 2   A B C   a  b  c       b  c   A B  C    a        a   b   c    a b c A2 B C  A  B  C      a b c abc          2 Áp dụng bất đẳng thức ta có:  x  y  z x  y  z  x  y  z 1 x2 y2 z2     y  z z  x x  y  y  z    z  x    x  y  2( x  y  z ) 2 Vậy ta có điều cần chứng minh SỞ GIÁO DỤC VÀ ĐÀO TẠO THỪA THIÊN HUẾ KỲ THI TUYỂN SINH LỚP 10 THPT CHUYÊN QUỐC HỌC NĂM HỌC 2018-2019 Khóa ngày 02 tháng năm 2018 Mơn thi: TỐN ( CHUN TỐN) Thời gian làm bài: 150 phút ( Không kể thởi gian phát đề) ĐỀ CHÍNH THỨC Câu 1: ( 1,5 điểm) 5x 12 x 32 x x Q x x 16 số nguyên, đồng thời P x ước Q x a) Cho biểu thức P x Q x b) Cho t x2 x x Tính giá trị biểu thức A x2 x2 x4 Tìm số nguyên x cho P x theo t Câu 2: (2,0 điểm) 11 x đường thẳng d : y x Gọi A, B giao điểm P d Tìm tọa độ điểm C trục tung cho CA CB có giá trị nhỏ a) Cho parabol P : y b) Giải hệ phương trình 2x y2 xy x2 y2 x 5x y y 0 Câu 3: ( 1,5 điểm) a) Xác định giá trị m để phương trình x 1 biệt x 1, x thỏa mãn điều kiện x 2x b) Giải phương trình 3x x 3x 7x 2mx 6m 6x ( x ẩn số) có hai nghiệm phân 3 Câu 4: ( điểm) Cho tam giác nhọn ABC AB AC nội tiếp đường tròn tâm O, có ba đường cao AD, BE,CF trực tâm H Gọi M giao điểm AO với BC P,Q chân đường vng góc vẽ từ M đến AB, AC a) Chứng minh H tâm đường tròn nội tiếp tam giác DEF b) Chứng minh HE.MQ HF.MP MB DB c) Chứng minh MC DC AB AC Câu 5: ( điểm) 1 49 16x 4y z 16 Tìm giá trị x, y, z cho a) Cho x, y, z số thực dương có tổng Chứng minh b) Cho số tự nhiên z số nguyên x, y thỏa mãn x 2z 42 x y2 y xy x 2y số phương lớn HẾT Thí sinh khơng sử dụng tài liệu làm Giám thị không giải thích thêm Họ tên thí sinh:……………………… Số báo danh: …………………… LỜI GIẢI THAM KHẢO CHO HỌC SINH GV: ĐỖ CAO LONG TP HUẾ 5x 1a) Cho biểu thức P x Tìm số nguyên x cho P x 12 x 32 x x Q x x 16 Q x số nguyên, đồng thời P x ước Q x Giải: 5x Ta có P x x0 Suy P x nguyên x0 4 x0 x0 x0 x0 12 x0 64 Ta có P P ; Q Q Vậy x x x x x 16 12 x ước nguyên dương 12 P 64 ; Q 64 75 1b) Cho t x x x2 Lời giải 1: 1) Nếu x t x2 t2 Khi đó: A x Tính giá trị biểu thức A A x 2) Nếu x x2 x 12 x 32 x 16 t 1 x2 x x4 Giải: x2 x2 theo t 1t 1 x x t x x 1 t x 1 t2 1 t2 t 2t t2 Từ hai trường hợp suy A 2t Lời giải 2: x2 Ta có A x2 x4 x x x2 2x x2 x2 : x x x 1 x2 t : x2 x2 x x2 x2 x2 2x x x 1 x2 t2 2t 11 x đường thẳng d : y x Gọi A, B giao điểm P d Tìm tọa độ điểm C trục tung cho CA CB có giá trị nhỏ Giải: 11 Hoành độ A B nghiệm phương trình: x x Phương trình có hai nghiệm: x x Suy A 4; , B ; 16 2a) Cho parabol P : y 4; đối xứng với A qua Dễ thấy hai điểm A, B nằm phía so với trục tung Lấy điểm A ' trục tung Khi CA CB CA ' CB A ' B , nên CA CB đạt giá trị nhỏ A ',C , B thẳng hàng, tức C giao điểm đường thẳng A ' B với trục tung Phương trình đường thẳng d ' qua A ' B có dạng y Ta có hệ Vậy C 0; 4a b a 16 a b b y y y x b 2x 2b) Giải hệ phương trình Ta có: 2x Suy d ' : y ax xy x 5x x y 2 x 9x 2 3x y x 2x 18x y2 5x y y y 0 Giải: x 1y 2x 5x 2 x x2 y2 xy y 5x x x y 2 3x 2 3x 2 0 y 2x y x  Trường hợp y x2 2x x2 y 2x y x y 2x y x x x 1, thay vào phương trình thứ hai hệ ta được: 2x 2 2x x 5x x x 4x 13 Trường hợp hệ cho có hai nghiệm: x ; y 1;1 , x ; y ; 5  Trường hợp y x, thay vào phương trình thứ hai hệ ta được: x 2x Trường hợp hệ cho có nghiệm: x ; y Vậy hệ cho có hai nghiệm: x ; y Điều kiện để phương trình x m2 ' 6m Khi x 2mx 6m Trường hợp 1: x1 x1 2x 2x 3 3x 3x Đặt a 2m 2m 2m m m ( x ẩn số) có hai nghiệm phân m x m m x 2m x m m x 3, ta có: 3 3, x 2m x 3b) Giải phương trình 3 6m 2m , vơ nghiệm 3, ta có: 2m m Vậy m Ta có 3, x Trường hợp 2: x1 x1 m 2mx Giải: ( x ẩn số) có hai nghiệm phân biệt là: 6m 13 ; 5 1;1 , x ; y 2mx x 1;1 3a) Xác định giá trị m để phương trình x 1 biệt x 1, x thỏa mãn điều kiện x 2x x x x 3 3x x 3x x 3x 7x 6x 1,b 3 3x 6x b3 Mà a b3 c3 3ab a b 3cd a 3ab a b d3 c3 3x b 6x 3 3 3x 7x d a c d3 3ab c 7x a b b ab cd a 6x Giải: 7x 7x 3, c b 3x Phương trình cho trở thành: a a3 3 2 2, d b c d d (2) c d nên (2) trở thành: a b ab cd Trường hợp a Trường hợp ab 18x 9x b , ta có a b cd , ta có ab 5x 3x x cd 6x 3x 6x 3x 3x x 6x x x x Vậy phương trình cho có năm nghiệm: x 4) Cho tam giác nhọn ABC AB ;x 1; x 7x ;x 3x 13 x x 7x 13 AC nội tiếp đường tròn tâm O, có ba đường cao AD, BE,CF trực tâm H Gọi M giao điểm AO với BC P,Q chân đường vng góc vẽ từ M đến AB, AC 4a) Chứng minh H tâm đường tròn nội tiếp tam giác DEF Giải: Ta có BFC BEC AFC ADC AEB ADB Suy tứ giác BFEC , AFDC , AEDB nội tiếp Suy ra: ABE 90 ADE (cùng chắn cung AE đường tròn AEDB ) (1) ADF ACF (cùng chắn cung AF đường tròn AFDC ) (2) ABE ACF (cùng chắn cung FE đường tròn BFEC ) (3) Từ (1), (2), (3) suy ADF ADE nên AD đường phân giác góc EDF tam giác EDF (4) Tương tự, ta chứng minh được: BE,CF đường phân giác góc DEF , DFE tam giác DEF (5) Từ (4), (5) suy H tâm đường tròn nội tiếp tam giác DEF 4b) Chứng minh HE.MQ HF.MP Gọi N giao điểm tia AO với đường trịn O Ta có: NC AC nên NC || BH Tương tự, ta có NB || CH Suy BHCN hình bình hành Ta có: FEH BCH (cùng chắn cung FB ), FBH ECH (cùng chắn cung FE ) nên hai tam giác HFE, HBC đồng dạng Do đó, hai tam giác HFE, NCB đồng dạng HE NB (6) HF NC Mặt khác: MQ || NC (cùng vng góc với AC ), MP || NB (cùng vng góc với AB ), suy Suy MQ NC AM AN MP NB HE Từ (6), (7) suy HF NB NC MP MQ MB DB 4c) Chứng minh MC DC MP MQ (7) HE MQ AB AC HF MP MB AB MC Hai tam giác vuông ADC MQC đồng dạng nên ta có AC MB AB MP Từ (8), (9) suy ra: (10) MC AC MQ Hai tam giác vuông ADB MPB đồng dạng nên ta có Ta có AMQ DB QM AB AM ANC DB MP AD MQ AD AB.MP AD AC MQ MC AD MB (8) (9) ABD suy hai tam giác vuông ADB AQM đồng dạng nên ta có AB.MQ AM (11) Tương tự: AMP ANB ACD suy hai tam giác vuông ADC APM đồng dạng nên ta có DC AC AC MP DC (12) PM AM AM DB AB MQ Từ (11), (12) suy (13) DC AC MP MB DB Từ (10), (13) suy ra: MC DC AB AC 16x 5a) Cho x, y, z số thực dương có tổng Chứng minh 4y z 49 16 Giải: Ta có 16x 4y z 49 16 x y 16 z a Với hai số thực khơng âm a, b ta có Dấu "=" xảy a b Áp dụng kết trên, ta có: x a y Dấu "=" xảy Và 16 z 49z 49y 56 14 a b (2) y (3) 16 z 49z z 16 z 16 49 Dấu "=" xảy x x y z Vậy bất đẳng thức cho chứng minh 49 x ;y y 42 x y2 42 x Với: x2 x Ta có: 1 y x y xy y 98 y xy Tìm giá trị x, y, z cho x 2y số phương lớn z z ;z 5b) Cho số tự nhiên z số nguyên x, y thỏa mãn x 2z ab (1) Cộng (1), (2), (3) theo vế ta được: x y Dấu "=" xảy b b 1 49x 49x x x Dấu "=" xảy 49x x x 49y 28 Trương tư, ta có: y 49x 49 xy x2 x xy z 2 2 y x 1 Giải: 21 x x ,1 y2 x y2 y xy y2 y 2z 2 42 x y2 x 2y Do đó, 2z 42 x y2 z x 2y số phương mod Nếu z lẻ 2z 2z mod dư 1) Từ suy z số chẵn z y , Ta có y y Suy 2z Nghĩa tồn số tự nhiên n cho 2z x 21 21 x y x 21 số phương n mod mod Khi n 2 mod vơ lí (vì số phương chia cho Đặt z 2k, k Vì 21 1.21 Trường hợp 1: Trường hợp 2: Ta có n n 2k n k 21 n 2k n k Nếu x 0, y x y 2k 2z Nếu x 2, y 2z Vậy x 2, y 3, z k 2k n n 21 2k 21 y Không tổng quát, giả sử x x y 42 x 2 10, khơng có giá trị k thỏa mãn trường hợp 2k x 1 2k n2 2k nên ta có hai trường hợp sau: n Từ giả thiết, ta có Giải hệ ta 2k 3.7 n 22k 21 y2 42 x y2 x 2y x 2y 102 100 2500 - HẾT - 502 y , suy x 1 y SỞ GIÁO DỤC VÀ ĐÀO TẠO KHÁNH HÒA KỲ THI TUYỂN SINH VÀO LỚP 10 TRƯỜNG THPT CHUYÊN LÊ QUÝ ĐÔN  Năm học 2018 - 2019  Mơn thi: TỐN (CHUN) Ngày thi: 06/6/2018 (Thời gian: 150 phút - không kể thời gian phát đề) ĐỀ THI CHÍNH THỨC  Bài 1: (3,00 điểm) a) Giải phương trình x2  x   3x x  b) Có số tự nhiên có chữ số abc cho a, b, c độ dài ba cạnh tam giác cân Bài 2: (2,00 điểm) a) Chứng minh với số thực a, b, c ta có  a  b  c   a  b2  c2   ab  bc  ca  2 b) Cho ba số x, y, z khác đồng thời thỏa mãn x  y  z  , 1 1  2 2  x y z xyz 1    Tính giá trị biểu thức Q   y 2017  z 2017  z 2019  x 2019  x 2021  y 2021  x y z Bài 3: (3,00 điểm) Cho đường tròn (O) đường kính BC H điểm nằm đoạn thẳng BO (điểm H không trùng với hai điểm B O ) Qua H vẽ đường thẳng vng góc với BC , cắt đường tròn (O) A D Gọi M giao điểm AC BD , qua M vẽ đường thẳng vng góc với BC N a) Chứng minh MNBA tứ giác nội tiếp  BO  OH    AB  BH b) Tính giá trị P   c) Từ B vẽ tiếp tuyến với đường tròn (O) , cắt hai đường thẳng AC AN K E Chứng minh đường thẳng EC qua trung điểm I đoạn thẳng AH điểm H di động đoạn thẳng BO Bài 4: (1,00 điểm) Với a, b, c số thực dương thỏa mãn điều kiện a  b  c  abc Chứng minh  a2  b2    c2  a b Bài 5: (1,00 điểm) Để tiết kiệm chi phí vận hành đồng thời đưa du khách tham quan hết 18 danh lam thắng cảnh tỉnh K, Công ty Du lịch lữ hành KH thiết lập tuyến chiều sau: có tuyến từ A đến B từ B đến C khơng có tuyến từ A đến C Hỏi có cách thiết lập để hết 18 địa điểm ?  HẾT  - Đề thi có 01 trang; - Giám thị khơng giải thích thêm Họ tên thí sinh: SBD: /Phòng: Giám thị 1: Giám thị 2: HƯỚNG DẪN CHẤM TUYỂN SINH LỚP 10 THPT CHUYÊN LÊ QUÝ ĐÔN MÔN: TOÁN (CHUYÊN) NĂM HỌC 2018 – 2019 - Hướng dẫn chấm có 04 trang; - Các cách giải khác đúng, cho điểm tối đa phần tương ứng Đáp án Điểm a) Giải phương trình x2  x   3x x  2,00 Bài Điều kiện x  1, ta có x2  x   3x x   x2  3x x   2( x  1)  0,5 Đặt u  x v  x   , Phương trình cho trở thành u  3uv  2v2  u  v  u  v    u  v  u  2v     u  2v  u  2v 0,5 0,5 x  1 x 1  x   x 2 x 1  x x   x  22 TH2: u  2v ta có x   x   4( x  1)  x  TH1: u  v ta có 0,5 1 b) Có số tự nhiên có chữ số abc cho a, b, c độ dài ba cạnh tam giác cân TH1: lập thành tam giác a  b  c  , có số lập Kết hợp với điều kiện ta có nghiệm phương trình cho x   2, x  Bài (3,00đ) 1,00 0,25 TH2: Xét a  b  c Vì a+ b> c nên (2,2,c) có cách chọn c, lập số (3,3,c) có cách chọn c, lập số (4,4,c) có cách chọn c, lập số Bắt đầu từ a = b  trở a  b  10 dù chọn c số từ đến (khơng tính 0,25 trường hợp trùng với a, b) ta có a+b>c Chọn a, b có cách chọn, Chọn c có cách chọn Nên có 8.5 = 40 cách chọn, lập 40 số Vì vai trị a, b, c nên có ( 2+4+6+40)×3 = 156 số 0,25 Vậy có 9+156 = 165 số cần tìm 0,25 a) Cho a, b, c số thực Chứng minh 1,00  a  b  c   a2  b2  c2   ab  bc  ca  2 Ta có VT   a  b  c    a  b   c    a  b    a  b  c  c  a  b2  c2   ab  bc  ca   VP (Điều phải chứng minh) Bài (2,00đ) b) Cho ba số 1 1  2 2  4, x y z xyz x, y, z khác đồng 1    x y z thời Tính 0,5 0,5 thỏa mãn x yz  giá trị biểu , thức 1,00 Q   y 2017  z 2017  z 2019  x2019  x 2021  y 2021  Ta có: 0,25 Bài Đáp án  1 1 1 1 2 x  y  z  1 1 1 4     2 2 2     2    x y z xyz x y z xyz x y z  xy yz zx  Điểm 1 1      ( theo câu a) x y z 1 1 1 Mà    , suy    (1) x y z x y z 1  (2) Mặt khác x  y  z   x yz Từ (1) (2) suy 1 1     ( xy  yz  zx)( x  y  z )  xyz   x  y  y  z  z  x   x y z x yz x   y   y   z x2021   y 2021, y 2017   z 2017 , z 2019   x 2019 Vậy Q   z   x Cho đường tròn (O) đường kính BC H điểm nằm đoạn thẳng BO (điểm H không trùng với hai điểm B O ) Qua H vẽ đường thẳng vng góc với BC , cắt đường tròn (O) A D Gọi M giao điểm AC BD , qua M vẽ đường thẳng vng góc với BC N a) Chứng minh MNBA tứ giác nội tiếp Bài (3,00đ) 0,25 0,25 0,25 1,00 Ta có BAC  900 ( góc nội tiếp chắn nửa đường tròn)  BAM  900 0,25 Mặt khác MNB  900 0,25 Suy Tứ giác MNBA có BAM  MNB  900  900  1800 Suy tứ giác MNBA tứ giác nội tiếp đường trịn đường kính MB 0,25 0,25  BO  OH b) Tính giá trị P      AB  BH AB AB  Ta có ABC vng A, nên: BH  BC BO AB 2 BO  AB OH  BO  BH  BO   BC BO OH BO  AB  BO    2  1 BH AB  AB  Vậy P = c) Từ B vẽ tiếp tuyến với đường tròn (O) , cắt hai đường thẳng AC AN lần 1,00 0,25 0,25 lượt K E Chứng minh đường thẳng EC qua trung điểm I đoạn thẳng AH điểm H di động đoạn thẳng BO 0,25 0,25 1,00 Đáp án Bài Điểm Ta thấy MBN  DBC (đối đỉnh) DBC  DAC ( Tứ giác DBAC nội tiếp) Suy MBN  DAC  NMB  BCA (1) Tứ giác MNBA nội tiếp nên ta có NMB  NAB (2) 0,25 Tam giác OAC cân O  BCA  OAC (3) Từ (1), (2), (3) suy NAB  OAC  OAC  BAO  NAB  BAO  BAC  NAO Mà BAC  900  NAO  900 Suy NA tiếp tuyến (O) Theo tính chất tiếp thuyến ta có EA=EB EAB  EBA Trong tam giác vng KAB, ta có EAB  EBA  BKA  EAK (Phụ với góc nhau)  KAE cân E  AE  KE  EB  KE Mặt khác AH//BK ( vng góc với AB) AI//KE  HI//EB  CI AI (Định lý Thales)  CE KE 0,25 0,25 CI HI AI HI (Định lý Thales)    CE BE KE BE Mà KE  EB , suy AI  HI nên I trung điểm đoạn thẳng AH Với a, b, c số thực dương thỏa mãn điều kiện a  b  c  abc Chứng minh 0,25  a2  b2    c2  a b 1 1 1 Ta có a  b  c  abc     Đặt  x ,  y,  z bc ca ab a b c 1,00 0,25 Khi x, y, z  xy  yz  zx  Vì  a2  b2 1 z2    c2    x2   y  1 a b z    Bài (1,00đ)   1   1   x2 1  y 1   x2  y   x2 1 y2     Bài    x2 1  1 y2 1  1  xy    x  y   x 1 1 x 1 y  x2   1  1   z  1  xy   z  1  0 z xy  z  1   0, x, y, z  z 2 1 y2   x  y  1 z2 0,25 1 z2  z  x  y  1 z2 0 z  z   xz  yz   z 1 y 1  0 z 0,25  z  z  x2  y  (4) z Ta có:  x  y   x  y  x y   4  1 z2 0 z 0,25 Ta có điều phải chứng minh Để tiết kiệm chi phí vận hành đồng thời đưa du khách tham quan hết 18 danh 1,00 Bài (1,00đ) Đáp án lam thắng cảnh tỉnh K, Công ty Du lịch lữ hành KH thiết lập tuyến chiều sau: có tuyến từ A đến B từ B đến C khơng có tuyến từ A đến C Hỏi có cách thiết lập để hết 18 địa điểm trên? Gọi A địa điểm có nhiều tuyến đường (gồm đường xuất phát từ A đường đến A) Ta chia địa điểm lại thành loại: Loại 1: tuyến đường xuất phát từ A có n(1) = m tuyến đường Loại 2: tuyến đường đến A có n(2) = n tuyến đường Loại 3: khơng có tuyến đến từ A có n(3) = p tuyến đường Khi đó: m+n+p = 17 Số tuyến đường liên quan đến A có m + n tuyến Số tuyến đường không liên quan đến A không vượt p  m  n  Điểm 0,25 0,25 Số tuyến đường liên quan đến loại không vượt m.n Vì a  b  c  3 ab  bc  ca   a  b2  c  ab  bc  ca   a  b2  c2    ab  bc  ca    a  b    b  c    c  a   0, với số thực a, b, c Do a  b  c ab  bc  ca  2 (5) Gọi S số cách thiết lập hết 18 địa điểm, áp dụng (5) Có S  m  n  p  m  n   m.n  m.n   p  1 m  n  p  1  0,25  m  n  p  1 m  18 Dấu đẳng thức xảy m  n  p      n    p   108 Vậy thiết lập tối đa 108 tuyến đường chiều để hết 18 địa điểm cho - HẾT - 0,25 SỞ GIÁO DỤC ĐÀO TẠO VĨNH LONG ĐỀ CHÍNH THỨC KỲ THI TUYỂN SINH LỚP 10 THPT CHUN NĂM HỌC 2018–2019 Mơn: TỐN (CHUN) Thời gian làm bài: 150 phút (không kể thời gian giao đề) Bài (2.0 điểm)  x3 x 2  a) Cho biểu thức A   với x  x  Tìm giá trị A  : x   x  x x 8 x  14  b) Tính giá trị biểu thức A  12  80  32  12  80  32 Bài (1.0 điểm) Cho phương trình x2   2m  3 x  m2   (1) ( x ẩn số, m tham số) a) Chứng tỏ phương trình (1) có hai nghiệm phân biệt với giá trị m b) Giả sử x1 , x2 hai nghiệm phương trình (1) Tìm m để phương trình có hai nghiệm phân biệt x1  x2 thỏa mãn x1  x2  Bài (1.5 điểm) a) Giải phương trình  x    12 x    x  y   36  x  b) Giải hệ phương trình  y  xy     Bài (1.5 điểm) a) Tìm số t nhi n x thỏa mãn biểu thức P   x4  x2  14 x  49 số ngu n tố b) Tìm nghiệm ngu n phương trình x  xy  y  x  3y  Bài (1.0 điểm) Cho tam giác ABC vng A có AB  6cm , AC  8cm Các đường phân giác phân giác ngồi góc B cắt đường thẳng AC M N Tính diện tích tam giác BMN Bài (2.0 điểm) Cho tam giác ABC vuông A  AB  AC  đường cao AH Vẽ đường tròn  O  đường kính BC Trên cung nhỏ AC lấ điểm E ( E  A , E  C ) cho hai tia AE BC cắt I ; AC cắt BE N Kéo dài AH cắt đường tròn  O  điểm thứ hai D , DE cắt BC M a) Chứng minh MN song song AD b) Chứng minh hai tam giác OME OEI đồng dạng Bài (1.0 điểm) Cho a, b, c số dương Chứng minh rằng: a3 b a a) 2 a b a b3 c3 a bc    b) 2 2 a  ab  b b  bc  c c  ca  a HẾT Thí sinh khơng sử dụng tài liệu Giám thị khơng giải thích th m Họ t n thí sinh: SBD: SỞ GIÁO DỤC VÀ ĐÀO TẠO KỲ THI TUYỂN SINH LỚP 10 THPT CHUYÊN VĨNH LONG NĂM HỌC 2018 – 2019 Môn thi: TOÁN CHUYÊN HƯỚNG DẪN CHẤM Điểm 2.0 Bài  x3 x 2  a) Cho biểu thức A   với x  x  Tìm giá trị  : x   x  x x 8 A x  14  Với x  0; x  , ta có:  x3 x 2  A    : x   x  x x 8  x3 x 2 x2 x 4    x 2 x2 x 4 x 2 x2 x 4         x 2 x 2 x2 x 4   x 0.5 x x2 x 4  Ta có x  14    2.3    Khi đó, ta có: A     x   3   14        x 3       3 3   24  8    0.25 0.25 b) Tính giá trị biểu thức A  12  80  32  12  80  32  Ta có A2  24       A  32   0.5 0.25 Do A  nên A   0.25 Cho phương trình x2   2m  3 x  m2   (1) ( x ẩn số, m tham số) 1.0 a) Chứng tỏ phương trình (1) có hai nghiệm phân biệt với giá trị m Ta có ac   m2  1  nên phương trình (1) có hai nghiệm phân biệt trái dấu với giá trị m 0.25 b) Giả sử x1 , x2 hai nghiệm phương trình (1) Tìm m để phương trình có hai nghiệm phân biệt x1  x2 thỏa mãn x1  x2  Do phương trình (1) có hai nghiệm phân biệt trái dấu x1  x2 Suy x1  , x2  0.25  x1   x1 , x2  x2 0.25  x1  x2     x1  x2    2m    m  0.25 1.5 a) Giải phương trình  x    12 x  Biến đổi tương đương phương trình ta x    12 x   x  18 x  81  12 x  0.25  x  18 x  81  36 x  12 x    x     x  1 2  x2   6x   x2  6x   x     x =  x   6 x   x  x  10  0.5 Vậ phương trình cho có tập nghiệm S  2; 4   x  y   36  x  b) Giải hệ phương trình  y  xy      x  y   36  x  (1)  Điều kiện x  y   , ta có  y  xy   (2)   0.25 Phương trình (2)   y  x   x  36 2 x  y   2x  y    y  x     2y  x  x  x  thỏa điều kiện Vậ hệ phương trình có nghiệm   y  y  Suy (1)  0.25 0.25 1.5 a) Tìm số t nhi n x thỏa mãn biểu thức P   x  x  14 x  49 số ngu n tố P    x  x   x  x  0.25 Ta có  x  x2  x  Vì P số ngu n tố n n  x  x2   x  x      x  2 (L) Vậy x   P  19 (thỏa mãn) b) Tìm nghiệm ngu n phương trình x2  xy  y  x  y  0.5 x2  xy  y  x  y   x   y  2 x  y  y   1    y     y  y  2  3 y  y  Để phương tình 1 có nghiệm    3 y  y    y  y    2  y   0.5 Vì y nguyên nên y  2 y  1 Với y  2 , 1  x   x  x  Với y  1 , 1  x  x    x  Vậ nghiệm phương trình:  0; 2  ,  0; 1 , 1; 1 Cho tam giác ABC vuông A có AB  6cm , AC  8cm Các đường phân giác phân giác góc B cắt đường thẳng AC M N Tính diện tích tam giác BMN 0.25 1.0 0.25 ABC vuông A  BC  AB2  AC  BC  10(cm) MA MC  BM đường phân giác ABC  BA BC Áp dụng tính chất dã tỉ số ta có: MA MC MA  MC AC       MA  3(cm) BA BC BA  BC BA  BC  10 NA NC  BN đường phân giác ABC  BA BC Áp dụng tính chất dã tỉ số ta có: NA NC NC  NA AC       NA  12(cm) BA BC BC  BA BC  BA 10  NM  NA  MA  15(cm) SBMN  BA.NM  45 (cm2 ) Cho tam giác ABC vuông A  AB  AC  đường cao AH Vẽ đường tròn  O  đường kính BC Tr n cung nhỏ AC lấ điểm E ( E  A , E  C ) cho hai tia AE BC cắt I ; AC cắt BE N Kéo dài AH cắt đường tròn  O  0.25 0.25 0.25 2.0 điểm thứ hai D , DE cắt BC M a) Chứng minh MN song song AD Chứng minh tứ giác MNEC nội tiếp NEC  90  NMC  90 Ta có MN  BC, AD  BC suy MN / / AD 0 b) Chứng minh OME ∽ OEI Gọi F giao điểm OE với đường tròn (O) ( F khác E ) Ta có BOF  EOC  EC  BF 0.5 0.25 0.25 0.25 DEF  AIB   1 sñFD  sñBD  sñBF 2   0.25  sñ AB  sđEC (tính chất góc có đỉnh b n ngồi đường tròn) Suy AIB  DEF Xét hai tam giác OME OEI 0.25 0.25 EOI chung EIO  MEO  OME ∽ OEI Cho a, b, c số dương Chứng minh rằng: a3 b a 2 a b a b3 c3 a bc    b) 2 2 a  ab  b b  bc  c c  ca  a a3 a(a  b2 )  ab2 ab ab b a) Ta có :   a   a   a 2 2 a b a b 2ab a b b3 c c3 a c b) Tương t theo câu a), ta có: 2  b  , 2 b c c a Cộng vế theo vế ba bất đẳng thức ta có: a3 b3 c3 abc  2  2 a b b c c a 3 a a a   Ta có: 2 2 a b a  ab  b a  b2 a2   b2 3 b b b3   Và b2  c2 b  bc  c b2  c2 2 b  c c3 c3 c3   c2  a2 c  ac  a c2  a2 2 c  a Cộng vế theo vế ba bất đẳng thức tr n ta có: a3 b3 c3  a3 b3 c3  a  b  c        a  ab  b2 b2  bc  c c  ca  a  a  b b  c c  a  a) HẾT./ 1.0 0.25 0.25 0.5 SỞ GIÁO DỤC – ĐÀO TẠO THÀNH PHỐ HỒ CHÍ MINH KỲ THI TUYỂN SINH LỚP 10 THPT NĂM HỌC 2018-2019 Mơn thi: TỐN CHUN ĐỀ THI CHÍNH THỨC Thời gian: 150 phút Ngày thi: 03/06/2018 Câu 1(1 điểm): Cho a, b, c ba số thực thỏa mãn điều kiện a  b  c  a   a  c  1 a  b  1 Tính giá trị biểu thức A  a  b2  c2 Câu (2 điểm) a) Giải phương trình : x    x  x x  y  b) Giải hệ phương trình:  3  x  y  x  y Câu (2 điểm): Cho tam giác ABC  AB  AC  vng tai A có đường cao AH Gọi E, F hình chiếu H lên AB, AC a) Chứng minh rằng: BE CH  CF BH  AH BC b) Gọi D điểm đối xứng B qua H gọi O trung điểm BC Đường thẳng qua D vuông góc với BC cắt AC K Chứng minh BK  AO Câu (1,5 điểm): a) Chứng minh x  x   với số thực x b) Cho x, y số thực thỏa mãn điều kiện x2  xy  y  Tìm giá trị lớn nhỏ biểu thức P  x2  y Câu (1,5 điểm) Cho tam giác ABC vuông A Gọi M trung điểm BC O tâm đường tròn ngoại tiếp tam giác AMB Đường thẳng AC cắt (O) điểm thứ hai K Đường thẳng BK cắt đường tròn ngoại tiếp tam giác ABC L.Các đường thẳng CL KM cắt E Chứng minh E nằm đường tròn ngoại tiếp tam giác ACM Câu (2 điểm) Các số nguyên dương từ đến 2018 tô màu theo quy tắc sau: Các số mà chia cho 24 dư 17 tô màu xanh; số mà chia cho 40 dư tơ màu đỏ Các số cịn lại tơ màu vàng a) Chứng tỏ khơng có số màu tơ hai màu xanh đỏ Hỏi có số tơ màu vàng b) Có cặp số  a; b  cho a tô màu xanh; b tô màu đỏ a b  ĐÁP ÁN Câu Ta có: a  b  c   b  a  c  a   a  c  1 a  b  1  a   a  c  1 a  a  c  1  a   a  c  1 c  1  a   a  c  1 c  1   a  2a  c  1   c  1    a  c  1   c  1  2 a  c   a     b  a  c  c   c      A  a  b  c  02  12   1  2 Vậy A  Câu Bài a Giải phương trình : x    x  x    x  3  x  x  Điều kiện :  x  4x x   x  4x2  x    4x   4x2  4x x   x      2x  x   1 2 x  x    x   2x 1    x  x   1  x   x  x   x     57 (tm)  x     x   2 x     57    (ktm)  x  2    4 x  x      x    x  x     x   1   x     x      2   x    x  x    3  41  4 x  3x   (tm)  x    3  41 (ktm)  x        57 3  41   ;      Đối chiếu với điều kiện ta có hệ phương trình tập nghiệm S    x  y  1(1)  b Giải hệ phương trình:    x  y  x  y (2) Lấy phương trình (2) trừ phương trình (1) ta được: y  y  x3  y   y  y  1   y  1  x   y  1  y  1  x3  1  y 1  y   x3 (*) Mà từ (1)  x2   y3  x   y  x   y Kết hợp với (1) (*) ta được:  1  1  y    y3  1  y  1  y   1  y  1  y  y  2  1  y  1  y  1  y    y  y      1  y  1  y  y  y   y  y    1  y   2 y  y    1  y  y  2  y    x    y  y 1  x    y      y0  y  2     y  2   x  3 Vậy hệ phương trình có tập nghiệm S   2; 3 ; 0;1 ; 1;0  Câu A F K E B H O D C Bài a Dễ thấy AEHF hình chữ nhật (tứ giác có góc vng)  AF  EH ; AE  FH Ta có BE CH  CF BH  AH BC   BE CH  CF BH    AH BC   BE CH  CF BH  2.BE.CF CH BH  AH BC (*) Áp dụng hệ thức lượng cho tam giác ABC vng A có đường cao AH ta có AH  BH CH (1) Xét EBH FHC ta có: BEH  HFC  900 ; EBH  FHC (đồng vị)  EBH FHC ( g.g )  BE.FC  FH EH  AE AF BE EH BH       EH HC  FC.BH FH FC HC  BE.HC  FH BH   BE CH  BE.FH BH  BE AE.HB  HF HB   CF BH  CF EH HC  CF AF HC  HF HC  2 BE.CF CH BH  AE AF AH  (*)  HE HB  HF HC  AE AF AH  AH BC   AH  HE  HB   AH  AF  HC  AE AF AH  AH BC ( Pitago)  AH  HB  HC    AE HB  AE AF AH  AF HC   AH BC  AH BC   AE HB  AE AF AH  AF HC   AH BC  AE HB  AE AF AH  AF HC   AE HB  AF HC  AE AF AH Ta có: BEH HEA( g  g )  EH BH   AE.BH  EH HA EA HA AH AF   AF HC  AH HF HC HF  AE BH  AF HC  AE.EH HA  AF AH HF AHF HCF ( g  g )   AE AF HA  AF AH AE  AE AF AH (dpcm) Vậy BE CH  CF BH  AH BC Câu b Gọi M giao điểm BK AO Xét tứ giác ABDK ta có: BAK  BDK  900  900  1800  ABDK tứ giác nội tiếp (dấu hiệu nhận biết)  DBK  DAK (hai góc nội tiếp chắn cung DK)  AH  BD  ABD tam giác cân A  BH  HD Xét BAD ta có:   BAH  HAD ( AH phân giác BAD)   DBK  DAK  BAD  BAC  2.BAH  BAC  2.BCA BCA  BAH  Theo tính chất góc ngồi tam giác AOC cân O nên ta có: BOA  OCA  OAC  2.BCA  DBK  BAC  2.BCA  900  BOA  DBK  BOA  900 Xét BOM ta có: BOM  MBO  900 (cmt )  BOM vuông M Hay  BK  AO(dpcm) Câu Bài a 1  1  1 Ta có x  x   x  x   x  x    x     x    4  2  2    x   x  Dấu xảy   (vô lý), dấu khơng xảy  x  x     2 Vậy x  x   với số thực x Bài b Ta có: x2  xy  y   P  xy   xy  P  x2  y  xy Áp dụng BĐT Cô si cho hai số x ; y ta có: x  y  x y  xy  2 2 2 x2  y x2  y P P  xy     xy  2 2 P P    P3 2 3P  P    P  3    2 P6 P P P    3    Dấu xảy  x2  y  x  y  x  y 1  x2  x2  x2     2  x  y  x  x  x  Vậy Pmax   x  y  3; Pmin   x  y  Câu B M O A C K L E Xét đường trịn (O) có BAK  900  BAK nội tiếp chắn nửa đường tròn (O)  BK đường kính đường trịn (O) Ta có BMK góc nội tiếp chắn nửa đường trịn (O)  BMK  900 hay KM  BC  EM đường cao tam giác EBC Xét đường tròn ngoại tiếp tam giác ABC có BAC  900  BAC nội tiếp chắn nửa đường tròn ngoại tiếp tam giác ABC  Đường trịn ngoại tiếp tam giác ABC có đường kính BC Ta có: BLC góc nội tiếp chắn nửa đường tròn ngoại tiếp tam giác ABC  BLC  900 hay BL  LC  BL  EC  BL đường cao EBC Xét EBC ta có: EM BL hai đường cao tam giác cắt K  K trực tâm EBC  KC  BE Mà KC  BA( gt )  B; A; E thẳng hàng  EAC  900 Xét tứ giác AECM ta có: EAC  EMC  900 Mà hai góc nhìn đoạn EC  AECM tứ giác nội tiếp (dấu hiệu nhận biết) Hay E thuộc đường tròn ngoại tiếp tam giác ACM (đpcm) Câu Bài a Giả sử có số a tổ màu xanh màu đỏ  a  ,1  a  2018  a chia cho 24 dư 17 hay a  24k  17  k  *, k  83  a chia cho 40 dư hay a  40l   l  *, l  50   24k  17  40l   40l  24k  10  20l  12k   4(5l  3k )  Vô lý 5l  3k  không chia hết cho Do giả sử sai Vậy khơng có số tô hai màu xanh đỏ Câu 6b a tô màu xanh  a  24d1  17  d1  *; d1   2018  17   d1  83  24  2018    b tô màu đỏ  b  40d   d  *; d   d  50  40   Ta có a  b   40d2   24d1  17   40d2  24d1  10   40d1  24d  10   40d  24d1  12 10d  6d1      40d1  24d  10  2  40d  24d1  5d  3d1  TH1: 10d2  6d1    5d2  3d1   Vơ lý  5d2  3d1  số chẵn Mà số lẻ TH2: 5d  3d1   d1  Có d1  83  5d  5d   83  d  50 Vì d1  nên 5d2  1(mod3) TH1: d2  1 mod3  5d2  5(mod3)  2(mod3) (loại) TH2: d2  2(mod3)  5d2  10(mod3)  1(mod3)(tm)  d2  3k   k  *   3k   50    k  16  k  *  có 16 giá trị k thỏa mãn Với giá trị k ta cho giá trị d , từ cho giá trị d1 hay nói cách khác, với giá trị k cho cặp số  d1; d  , tức cho cặp số  a; b  thỏa mãn u cầu tốn Vậy có 16 cặp số  a; b  thỏa mãn yêu cầu toán SỞ GIÁO DỤC VÀ ĐÀO TẠO BẮC GIANG ĐỀ CHÍNH THỨC ĐỀ THI TUYỂN SINH LỚP 10 THPT CHUYÊN BẮC GIANG NĂM HỌC 2018 - 2019 MÔN THI: TOÁN Ngày thi: 07/6/2018 Thời gian làm bài: 150 phút, không kể thời gian giao đề Câu I (5,0 điểm)  x4 x 4 x x   1  Cho biểu thức A      :   (với x  0; x  1)  x x  x  x  1  x     a) Rút gọn biểu thức A  2018 b) Có giá trị nguyên x để A  2018 Cho phương trình x   m  1 x   (1), với x ẩn, m tham số Gọi x1 , x2 hai 3x12  3x22  x1  x2  nghiệm phương trình (1) Đặt B  Tìm m B đạt giá trị lớn x12  x22  Câu II ( 5,0 điểm) Giải phương trình x   x2  x    x  xy  x  y   Giải hệ phương trình    x   16  y  x  x  y  Câu III (3,0 điểm) Chứng minh không tồn số tự nhiên n để 2018  n2 số phương Mười đội bóng chuyền tham gia giải bóng chuyền VTV cup 2018 Cứ hai đội giải đấu thi đấu với trận Đội thứ thắng x1 trận thua y1 trận, đội thứ hai thắng x2 trận thua y2 trận,…, đội thứ mười thắng x10 trận thua y10 trận Biết trận đấu bóng chuyền khơng có trận hịa Chứng minh rằng: x12  x22   x102  y12  y22   y102 Câu IV (6,0 điểm) Cho tam giác ABC nhọn nội tiếp đường tròn  O  với AB  AC Gọi M điểm thuộc cạnh BC ( M không trùng với B C ), đường thẳng AM cắt đường tròn  O  điểm D khác A Đường tròn ngoại tiếp tam giác MCD cắt đường thẳng AC điểm E khác C Đường tròn ngoại tiếp tam giác MBD cắt đường thẳng AB điểm F khác B a) Chứng minh tứ giác BECF nội tiếp đường tròn b) Chứng minh hai tam giác ECD , FBD đồng dạng ba điểm E, M , F thẳng hàng c) Chứng minh đường thẳng OA vng góc với đường thẳng EF Cho tam giác ABC vuông A Các cạnh tam giác ABC thỏa mãn điều kiện BC  2BC.AC  AC Tính số đo góc ABC Câu V (1,0 điểm) Cho x, y, z số thực thỏa mãn x2  y  z  Tìm giá trị lớn biểu thức M  x3  y  y  z  z  x3 Hết -Cán coi thi khơng giải thích thêm Họ tên thí sinh: Số báo danh: Giám thị (Họ tên ký): Giám thị (Họ tên ký): SỞ GIÁO DỤC VÀ ĐÀO TẠO BẮC GIANG HƯỚNG DẪN CHẤM BÀI THI TUYỂN SINH LỚP 10 THPT CHUYÊN BẮC GIANG NĂM HỌC 2018-2019 NGÀY THI: 07/6/2018 MƠN THI: TỐN (Bản hướng dẫn chấm có 05 trang) HDC CHÍNH THỨC Hướng dẫn giải Câu Câu I + Biến đổi x4 x 4 x x   1 x x x 2 + Ta có A  + Vậy A  1   x 1 1 x : x 1  x   x 1  x 2  x 1  x 2   x 1   x 1 x 1  x 1  x   x 2 x   x 1 x 1 x = Phần 1.a + Biến đổi (2,0 điểm)    Điểm (5.0 đ) x 1  x 1  x 1 x 1 0.5  x 1  x 1 x 1 , với điều kiện x  0, x  x 0,5  2018 1 1  1  1   2018 x 2018 x 2018 x  2018   x  2018 Vì x  0, x  x nguyên nên x 2;3;4; ;2018 Suy có 2017 giá trị ngun x thỏa mãn tốn Phương trình x   m  1 x   (1) + Nhận xét    m  1  12  0, m  0.5 x A Phần 1.b (1,0 điểm) 0.5 0.5 0.25 0.25 Suy (1) ln có hai nghiệm phân biệt 0.25 x1 , x2  x1  x2  m  + Theo hệ thức Viet ta có:   x1 x2  3 2 Phần 3x12  3x22  x1  x2   x1  x2    x1  x2   B   Ta có (2,0 điểm) x2  x2  x2  x2  2  x1  x2   x1 x2    x1  x2    m  1  6   m  1        2  x1  x2   x1 x2   m  1    3m2  10m  20 m  2m  0.5   B  3 m2   B  5 m  3B  20  (*) 11 + Nếu B  (*) phương trình bậc hai ẩn m Phương trình (*) có nghiệm m  '  hay  B  5   B  3 3B  20    B  19 B  35    B  Vậy giá trị lớn B m   + Nếu B  m   0.5 0.25 0.5 (5.0 đ) Câu II + Điều kiện x    x  3  x     x  x  5   x 1   x  1 x  5  x3 2 + Phương trình cho tương đương   x 1      x  1    x  5       x  5   x3 2   x   Phần (2.5 điểm) 0,25 0.5 0.75 +) x 1   x  +)   x  5  vơ nghiệm x3 2   x  5  0, x  3 x3 2 + So sánh điều kiện ta tập nghiệm phương trình 1 0.75 0.25 16 + ) Điều kiện x  , y  x  +) x2  xy  x  y     x  3 x  y      y  x  +) Với x  thay vào phương trình Phần (2.5 điểm) 0.75 5x   16  y  x2  x  y  , ta 0.75  y  13 y   13  133 16  y  y     y  y  5 5x   16  y  x2  x  y  , ta +) y  x  thay vào phương trình 5x   10  3x  x  x     x  2 5x    3 x  2 10  3x     5x      x   x  3      x  2    2x  3  10  3x   5x     10  3x   x  x  0.75 x      2x    x   10  x  +) Với x   y  (thỏa mãn) +) Vì 10 5  x   5x       3 5x   2 5x   10 x   2x  10  3x  Do phương trình   x   vô nghiệm 5x   10  3x    13  133     Vậy hệ phương trình có tập nghiệm  2;  ;  3;        Câu III Giả sử 2018  n số phương 2018  n  m 2 0.25 m  Suy 2018  m2  n2  2018   m  n  m  n  Phần (1.5 điểm) (3.0đ) * Như hai số m  n m  n phải có số chẵn (1) Mà  m  n    m  n   2m nên suy hai số m  n m  n tính chẵn lẻ (2) Từ (1) (2) suy hai số m  n m  n hai số chẵn   m  n  m  n  chia hết cho Mà 2018 không chia hết điều giả sử sai Vậy không tồn số tự nhiên n để 2018  n2 số phương Có 10 đội bóng, đội thi đấu trận với đội cịn lại Do số trận thua đội từ đội thứ đến đội thứ 10 : y1   x1 , y2   x2 , , y10   x10 10.9  45 trận Vì khơng có trận hịa nên tổng số trận thắng 10 đội là: x1  x2   x10       45 0.5 0.5 0.5 0.5 Có tất số trận đấu : Phần (1.5 điểm) 0.5 Ta có : 2 y12  y22   y102    x1     x2      x10   y12  y22   y102  10.92  18  x1  x2   x10    x12  x22   x102   x12  x22   x102  y12  y22   y102 ( đpcm) 0.5 (6.0 đ) 4,0 điểm Câu IV Phần A x B O E M C D Phần a (1.0 điểm) F Tứ giác CDME nội tiếp  AM AD  AE.AC Tứ giác BMDF nội tiếp  AM AD  AB.AF 0,5 Suy AB AF  AE AC Do tứ giác BECF nội tiếp 0.5 Tứ giác CDME nội tiếp  DEC  DMC (1) Tứ giác BMDF nội tiếp  DMC  DFB (2) ( bù với góc DMB ) 0.5 Từ (1) (2)  DEC  DFB (3) Tứ giác ABDC nội tiếp  DBF  ACD (4) ( bù với góc ABD ) Phần b Từ (3) (4) suy tam giác ECD FBD đồng dạng (2.0 điểm) Theo chứng minh trên, ta có tam giác ECD FBD đồng dạng  EDC  BDF 0.5 0.5 Tứ giác ECDM nội tiếp  EDC  EMC Tứ giác BMDF nội tiếp  BDF  BMF 0.5 Suy EMC  BMF ( vị trí đối đỉnh) Vậy ba điểm E, M , F thẳng hàng Kẻ tiếp tuyến Ax đường tròn  O   ACB  BAx Do tứ giác CEBF nội tiếp  ECB  EFB hay ACB  EFA Phần c (1.0 điểm) Suy BAx  EFA ( vị trí so le trong)  Ax // EF mà Ax  AO  EF  AO 0.25 0.25 0.5 (2.0 điểm) Phần B D E C A Gọi D trung điểm cạnh BC Theo giả thiết ta có CD  CD  AC (1) AC Kẻ phân giác AE tam giác ACD Theo tính chất đường phân giác, EC AC AC EC AC EC AC ta có (2)       ED AD DC ED  EC AD  AC CD AC  CD EC CD AC Từ (1) (2) suy   AC CD  AC CD Suy tam giác ACE đồng dạng với tam giác DCA nên tam giác ACE cân A 1 Lại có EAC  CAD  ACB 2 Do : ACB  ACB  ACB  1800  ACB  720  ABC  180  2CD   4CD AC  AC  CD2  CD AC  AC  Câu V Áp dụng tính chất a  b  a  b Đẳng thức xảy ab   Khi M  x  y  z 3   x3  2 x x 2  x2      2 Mặt khác x  y  z    y    y  2   y  2 y z2    (1.0 điểm)   z  2  z  2 z  Vậy M  x  y  z 3   x  0.75 0.5 (1.0 đ) 0.25 0.25  y  z  = 32  0.75 0.25   Đẳng thức xảy  x; y; z   2;0;0  x; y; z   2 2;0;0 hốn vị Vậy giá trị lớn M 32 0.25 Tổng Điểm toàn 20 đ Lưu ý chấm bài: - Trên sơ lược bước giải Lời giải học sinh cần lập luận chặt chẽ hợp logic Nếu học sinh làm cách khác mà giải cho điểm tối đa - Đối với câu IV, học sinh khơng vẽ hình khơng chấm - Điểm tồn khơng làm trịn SỞ GIÁO DỤC VÀ ĐÀO TẠO QUẢNG NAM KỲ THI TUYỂN SINH LỚP 10 THPT CHUYÊN NĂM HỌC 2018-2019 Đ CH NH TH C Mơn thi : TỐN (chun) Thời gian : 150 phút (không kể thời gian giao đề) Ngày thi : (Đề thi có 01 trang) Câu (2,0 điểm)  a 1   2a b  ab  ab  a a) Cho biểu thức A      :    ab  1  ab  ab     với a  0; b  ab  Rút gọn biểu thức A tìm giá trị lớn A a + b = ab b) Tìm tất cặp số nguyên ( x ; y) thỏa mãn đẳng thức x y2  x  y2  xy Câu (2,0 điểm) x  x   x  3x   3x  x   x  x   27 8 x  y  18  b) iải h phương trình   4x  6x   y y2 a) Giải phương trình Câu (1,0 điểm) Cho hai hàm số y  x y  mx Tìm m để hai đồ thị hai hàm số cho cắt ba điểm phân bi t ba đỉnh tam giác Câu (2,0 điểm) Cho hình vng ABCD có cạnh a Trên cạnh AD lấy điểm M cho AM  3MD Kẻ tia Bx cắt cạnh CD I cho ABM  MBI Kẻ tia phân giác CBI , tia cắt cạnh CD N a) So sánh MN với AM + NC b) Tính di n tích tam giác BMN theo a Câu (2,0 điểm) Cho đường tròn tâm O, dây cung AB không qua O Điểm M nằm cung lớn AB Các đường cao AE, BF tam giác ABM cắt H a) Chứng minh OM vng góc với EF b) Đường trịn tâm H bán kính HM cắt MA, MB C D Chứng minh M di động cung lớn AB đường thẳng kẻ từ H vng góc với CD ln qua điểm cố định Câu (1,0 điểm) Cho ba số thực dương a, b, c Chứng minh a  b2 b2  c c  a 3(a  b  c )    ab bc ca abc - HẾT Họ tên thí sinh: Số báo danh: SỞ GIÁO DỤC VÀ ĐÀO TẠO QUẢNG NAM KỲ THI TUYỂN SINH LỚP 10 THPT CHUYÊN NĂM HỌC 2018-2019 H C CH NH TH C HƯỚNG DẪN CHẤM MƠN TỐN CHUN (Bản hướng dẫn gồm 04 trang) Câu Nội dung Câu  a 1   2a b  ab  ab  a   1 :   , a) Cho biểu thức A    ab ab  1  ab     (2,0) với a  0; b  ab  Rút gọn biểu thức A tìm giá trị lớn A Điểm 1,0 a + b = ab  a 2a b  ab :  ab  ab 2(1  a )   ab (1  a ) ab A Khi a  0; b  , a + b = ab  0,25 0,25 a+ b 1 1 1  1  1 ab a b a b 0,25 1  1 (1  )     Dấu “ = “ xảy  b  4; a  Vậy giá Do A   b  b b 0,25 trị lớn A a  b  4 1,0 b) Tìm tất cặp số nguyên ( x ; y) thỏa mãn đẳng thức x y2  x  y2  xy - Với y  , ta có x  - Với y  , ta có: x y2  x  y2  xy  x y2  5y2  ( x  y)2  x   ( x  y)2 y2  a2  a    x  x  a 5 x2  a2     x 3   x  3   x  a 1  y  1 Khi x =  y  y     y  y 1 Khi x = -3  y  y      y  3 Vậy  x; y    0;0  ;  3; 1 ;  3;3 ;  3;1 ;  3; 3 Câu Nội dung Câu a) Giải phương trình x2  x   x2  3x   3x2  x   x  x  Đặt a  x  x  3; b  x  3x  2; c  3x  x  2; d  x  x  0,25 0,25 0,25 0,25 Điểm 1,0 0,25 (2,0) Ta có a  b  c  d  3 3 a  b  c  d (1) (2) (2)   a  b   3ab  a  b    c  d   3cd  c  d  3 a  b  ab  a  b   cd  a  b     ab  cd -Với a  b , ta có 0,25   69 x  x  x   x  3x   x  x       69 x   0,25 -Với ab  cd , ta có    11  11    x  3 x  3x     3x  x   x  x  3  x  0;1; ;  0,25 2      11  11  69  69 ;x ;x  ;x  Vậy pt có nghi m x  0; x  1; x  2 2  27 8 x  y  18  b) iải h phương trình  1,0 x x   1  y y   27 3 x   18 (2 x )     18  y3   y   0,25 x x   3  2 x x   1    y  y  y y  x Đặt a  x ; b =  3 a  b  Ta có a  b  18    y ab  ab(a  b)    3 3  3 3  Giải tìm  a  ;b  ;b    a   2  2     3  3  ; ; Tìm nghi m  x; y  h  ;          0,25 0,25 0,25 Câu Nội dung Điểm Câu Cho hai hàm số y  x y  mx Tìm m để hai đồ thị hai hàm số cho cắt ba điểm phân bi t ba đỉnh tam giác 1,0 (1,0) Phương trình hồnh độ giao điểm 2 x  mx  x  m x   x  0; x  m m ;x   2 0,25  m m2   m m2  Gọi ba giao điểm O(0;0); A  ; H giao điểm AB trục ;B  ;  2   2      0,25 m tung, suy AB  m ; OH  m2 0,25 Tam giác OAB  OH  AB   m 2 0,25 Giải tìm m  0; m  3; m   , loại m  Vậy m  3; m   Câu Nội dung Câu a) So sánh MN với AM + NC (2,0) Hình vẽ phục vụ câu a) A B Trên cạnh BI lấy điểm H cho BH = BA = a  ΔABM  ΔHBM ΔHBN  ΔCBN Điểm Suy BHM  BAM  900 ; BHN  BCN  900 Suy M; H; N thẳng hàng, MN = MH + HN = AM + NC 0,25 M D H I N C 1,0 0,25 0,25 0,25 Ghi chú: khơng có hình khơng chấm b) Tính di n tích tam giác BMN theo a 1,0 Đặt NC  x  MN  AM  NC  a  x ; DN  a  x 0,25 a2 3   a  x Theo định lí Pitago MN  MD  DN   a  x   16 4  a Giải tìm x  1 3 a  25 Di n tích tam giác BMN BH MN  a  a    a 2 4  56 Câu Nội dung a) Chứng minh OM vuông góc với EF Câu (2,0) Hình vẽ phục vụ câu a (khơng tính điểm hình vẽ câu b, khơng có hình khơng chấm) 0,25 0,25 0,25 Điểm 1,0 0,25 x x M M E E F F O H H A B A O I C D B K N Tứ giác ABEF có AEB  AFB  900 nên nội tiếp đường tròn  MEF  FAB Từ M kẻ tia tiếp tuyến Mx với đường tròn tâm O (như hình vẽ), ta có xMB  MAB  FAB Suy xMB  MEF  Mx / / EF Theo tính chất tiếp tuyến đường trịn, ta có MO  Mx  MO  EF b) Chứng minh M di động cung lớn AB đường thẳng kẻ từ H vng góc với CD qua điểm cố định Kẻ đường kính MN đường trịn tâm O Tứ giác AHBN có AH song song với NB (cùng vng góc với MB), có BH song song với NA (cùng vng góc với MA) nên hình bình hành Suy HN cắt AB trung điểm I đoạn Do MH / /OI ; MH  2.OI Gọi K điểm đối xứng O qua I, suy OK = 2OI điểm K cố định Tứ giác MHKO có MH, OK song song ( gấp đơi OI) nên hình bình hành Suy HK / / MO Xét đường tròn tâm H bán kính HM, theo tính chất đường kính vng góc với dây cung, suy E trung điểm MD F trung điểm MC Do EF / /CD  MO  CD  HK  CD Vậy M động cung lớn AB đường thẳng kẻ từ H vng góc với CD qua điểm cố định K Câu Cho ba số thực dương a, b, c Chứng minh rằng; a  b2 b2  c c  a 3(a  b  c )    (1,0) ab bc ca abc 2 a  b b  c c  a 3(a  b  c )    Với ba số thực dương a, b, c ta có (1) ab bc ca abc  a  b2 b2  c2 c2  a  2  a  b  c     3(a  b  c )  ab bc ca     c a  b2 ab c     a b c a  b2  c2 bc a   b c  a2 ca   a b2  c2 a  b 2  0,25 0,25 0,25 1,0 0,25 0,25 0,25 0,25 1,0 0,25  b2  c2  b c2  a2 0 ab bc ca ac  c  a  bc  c  b  ab  b  a  bc  b  c  ab  a  b  ac  a  c        0 ab ab ca ca bc bc 0,25 ac  c  a  bc  c  b  ab  b  a  0,25     (2)  a  b  b  c   a  b  a  c   a  c b  c  Với ba số thực dương a, b, c ta có (2) ln Vậy (1) ln (đpcm) 0,25 * Lưu ý: Nếu thí sinh làm không theo cách nêu đáp án giám khảo cho đủ số điểm phần hướng dẫn quy định 2 SỞ GIÁO DỤC VÀ ĐÀO TẠO BÌNH DƯƠNG ĐỀ THI CHÍNH THỨC ĐỀ THI TUYỂN SINH VÀO LỚP 10 THPT CHUYÊN Năm học: 2018 – 2019 Mơn thi: TỐN CHUN Thời gian làm bài: 150 phút (Không kể thời gian phát đề) Câu (3,0 điểm)  a) Giải phương trình:  x  x   x   7x b) Cho số thực x, y thỏa mãn x  2018  x  y   2018  y  2018 Tính giá trị biểu thức: Q  x 2019  y 2019  2018  x  y   2020 Câu (1,5 điểm) Gọi x1 , x2 hai nghiệm phương trình x   m  1 x  2m   Tìm tất 2 x  x  giá trị m nguyên dương để A       có giá trị nguyên  x2   x1  Câu (2,0 điểm) a) Tính giá trị biểu thức: 1 1 P     1   2025 2024  2024 2025 b) Tìm tất cặp số nguyên dương x, y thỏa mãn: x  y  3 x  y  Câu (3,5 điểm) Cho đường tròn (O) bán kính R điểm M nằm ngồi đường tròn (O) Kẻ tiếp tuyến MA, MB tới đường tròn (O) (A, B tiếp điểm) Trên đoạn thẳng AB lấy điểm C (C khác A C khác B) Gọi I, K trung điểm MA, MC Đường thẳng KA cắt đường tròn (O) điểm thứ hai D a) Chứng minh rằng: KO2 – KM2 = R2 b) Chứng minh tứ giác BCDM tứ giác nội tiếp c) Gọi E giao điểm thứ hai đường thẳng MD với đường tròn (O) N trung điểm KE Đường thẳng KE cắt đường tròn (O) điểm thứ hai F Chứng minh bốn điểm I, A, N, F thuộc đường tròn HẾT -Họ tên thí sinh: Chữ kí giám thị 1: Số báo danh: Chữ kí giám thị 2: HƯỚNG DẪN GIẢI VÀ BIỂU ĐIỂM DỰ KIẾN: Câu Phần  72 x  x  2 x Nội dung  Điểm 7x (1) ĐK:  x  (1)   x  x   x  x  x  7  x  x  x  x   x   7x  a)           7x  x 2 7x  x   7x  x 0  7x 2 0 7x  x 0 1.5 7x 20 7x  x 7x 2 7  x  x  7  x   x  3,5 (TM)   x  (TM) Vậy tập nghiệm phương trình S  3,5;3 Câu (3,0đ) x  2018  x  y   2018  y  2018 Thực phép nhân liên hợp, ta có:   (1)  (1)   x  2018  x  y  2018  y  2018 x  2018  x     2018 y  2018  y  2018 x  2018  x    y  2018  y  2018  x  x  2018  x  2018  y  x  y b)  (1)  x  2018  x   y  (2)  2018  y   2018 y  2018  y    2018 x  2018  x  2018 y  2018  y    x  2018  x  2018  y  y  2018  x  2018  y   x  y Từ (2) (3)  x  y   x  y  2x  2 y  x   y Thay x   y vào biểu thức Q, ta được: (3) 1.5 Q  ( y ) 2019  y 2019  2018   y  y   2020   y 2019  y 2019  2018.0  2020  2020  '  (m  1)2  (2m  6)  m2  2m   2m   (m2  4m  4)   (m  2)   m  Phương trình ln có hai nghiệm phân biệt x1 , x2  x  x  2m  Áp dụng hệ thức Vi-ét, ta có:   x1 x2  2m  Do đó: 2 2  x1   x2   x1 x2  x1 x2  x12  x22  A       2    2 x2 x1  x1 x2   x2   x1   x2 x1  2  ( x1  x2 )  x1 x2   (2m  2)  2(2m  6)    2   2 x1 x2 2m      Câu (1,5đ) 2  4m  8m   4m  12   4m  12m  16        2 2m  2m      1.5   2m(2m  6)  16       2m   2  2m  m 3    Q Với m  N * , m  2m  m3  A có giá trị nguyên 8  2m  Z  Z m3 m3  m   1; 2;4;8  m   m   3 2  m  4;2;5;1;7;11 Vậy m  4;2;5;1;7;11 giá trị cần tìm 1 1     1   2025 2024  2024 2025 * Với n  N , ta có: 1   n  1 n  n n  n  n n   n P  Câu (2,0đ) a)  n 1  n n 1  n 1    n  n  n   n  n  n n n 1 Áp dụng kết trên, ta được: 1 1 1 1 P         2 3 2024 2025 1 44    1  45 45 2025  1.0  x  y   x  xy  y   x  xy  y  x  xy  y  x  xy  y   x2  y    x  y  (1) Theo đề bài: x2  y  3 x  y    x2  y    x  y  b) (2) Từ (1) (2)   x  y  6 x  y 1.0 (3)  x  y  (do x, y  N *  x  y  0) 2 Vì x , y số phương nên chia cho dư Mà x  y  3 x  y   x2 y Từ (3) (4)  x  y  (thỏa mãn đề bài) Vậy x  y  (4) A I D M P K H O 0.25 C B Câu (3,5đ) a) Gọi H, P giao điểm OM với AB, IK Ta có: OA = OB = R MA = MB (tính chất hai tiếp tuyến cắt nhau)  OM đường trung trực AB  OM  AB H  MAC có IM = IA KM = KC  IK đường trung bình  MAC  IK // AC hay IP // AH  MAH có IM = IA IP // AH  PM = PH Vì IK // AC OM  AC  OM  IK P  Các tam giác KPO, KPM vuông P Áp dụng định lí Py-ta-go, ta có: KO2  KP  PO2 KM  KP  PM  KO2  KM  PO2  PM  (PO  PM)(PO  PM)  OM.(PH  OH  PM)  OM.OH (do PM  PH)  OAM vuông A (vì MA tiếp tuyến A (O)) Áp dụng hệ thức cạnh đường cao tam giác vng, ta có: OM.OH  OA2  R 1.25 Mà KO2  KM2  OM.OH  KO2  KM2  R (đpcm) A I Q D M O K C B b) Vẽ tiếp tuyến KQ (O) (Q A nằm phía với MC)   KQO vng Q  KO2  KQ2  OQ2  KQ2  R (định lí Py-ta-go) Mà KO2  KM2  R  KO2  KM2  R  KQ2  KM2  KQ  KM  KC    KQD  KAQ có: QKA chung; KQD  KAQ   sđDQ      KQD  KAQ (g.g) KQ KD KC KD     (vì KQ  KC) KA KQ KA KC   KCD  KAC (c.g.c)  C1  A1 1.0    C1  B1  A1  B1  sđBD     Tứ giác BCDM tứ giác nội tiếp (đpcm) A I D c) M N F K C B O E 1.0 Xét đường tròn ngoại tiếp tứ giác BCDM có  DMC  B2 (2 góc nội tiếp chắn cung CD)   Mà B2  E1   sđAD     DMC  E1 Nhưng hai góc vị trí so le  MK // AE  AEKM hình thang Hình thang AEKM (AE // MK) có IA = IM NE = NK  IN đường trung bình hình thang AEKM  INF  AEF (2 góc đồng vị)   Mặt khác: IAF  AEF   sđAF      IAF  INF  AEF   AIFN tứ giác nội tiếp  điểm A, I, F, N thuộc đường tròn (đpcm) Thầy Nguyễn Mạnh Tuấn Trường THCS Cẩm Hoàng – Cẩm Giàng – Hải Dương SỞ GIÁO DỤC – ĐÀO TẠO TỈNH BÀ RỊA – VŨNG TÀU KỲ THI TUYỂN SINH LỚP 10 THPT NĂM HỌC 2018-2019 Mơn thi: TỐN CHUN Thời gian: 150 phút Ngày thi: 03/06/2018 ĐỀ THI CHÍNH THỨC   a3  a  a 1 Câu a) Rút gọn biểu thức: P   1 a   a 1  a   b) Giải phương trình: x  x  1 x  3x   24  (a  1)   x  y  xy  x c) Giải hệ phương trình:   x  y  xy  Câu a) Cho đa thức f ( x)  x  ax  b thỏa mãn f (1)  f    Chứng minh phương trình f ( x)  x có hai nghiệm phân biệt Tìm số nghiệm f  f ( x)   x b) Cho A  m2n2  4m  2n với m, n số nguyên dương Khi n  tìm m để A số phương Khi n  chứng minh A số phương Câu Cho a; b; c  dương thỏa mãn abc  a  b  c   Chứng minh rằng: ab  ac  bc  Tìm GTNN biểu thức: P  a  b2  b  c2  c  c2 Câu Cho đường trịn (O) đường kính AB, M thuộc (O) khác A B Các tiếp tuyến A M cắt C Đường tròn  I  qua M tiếp xúc với AC C Các đường CO CB cắt  I  E F Vẽ đường kính CD (I), giao điểm DE AB K a) CMR; tam giác OCD cân OEFK tứ giác nội tiếp b) Chứng tam giác OEF CED đồng dạng c) Đường thẳng qua điểm  O   I  cắt AC H Chứng minh đường AF , CK , OH đồng quy Câu Cho tam giác ABC có cạnh Điểm M di động bên tam giác thỏa mãn: BMC  1200 Đường thẳng BM cắt AC Q CM cắt AB P Chứng minh AP  AQ khơng đổi Tìm GTLN diện tích tứ giác APMQ ĐÁP ÁN Câu a) Rút gọn biểu thức… Ta có:   a3  a  a 1 P 1 a   a  1  a 1  a        1 a 1 a  a   P   a    a 1 a 1      1 a  a  a 1 P   a  ( Do a  a   P P  a 1 a a   a   a   0) 1 a  a  a 1 a  a a 1 a 1 a a a 1 1 a 1 a b) Giải phương trình: x  x  1  x  3x    24 Phương trình tương đương với: x  x  1 x  1 x    24   x  x  x  x    24   x  x    x  x   24    x  x   x  x     x2  x    Do x2  x   0   x   x  3  x    x  3 Vậy tập nghiệm phương trình S  2; 3  x  y  xy  x c) Giải hệ phương trình:   x  y  xy  ĐKXĐ: x  y  Từ phương trình thứ ta được: x  y  xy  x  x  x  2  y   x   x    x   x  y     x  y 2  y   y  2 4 y  y    x    y  2y      y2 y 1  y   y   y  x  y  2y  y     x y2 y  y  y     y  y  y      Vậy hệ cho có nghiệm  2;2  Câu a) Cho đa thức……… Ta có: f 1    a  b   a  b f ( x)  x  x  ax  b  x  x  bx  b  x   x  bx  b  x   x  x  1  b  x  1    x  1 x  b   x   x  b f  0   b   b  Do phương trình cho ln có nghiệm phân biệt Mặt khác f  f  x    x  f ( x)  af ( x)  b  x  f ( x)  bf  x   b  x   f ( x)  x  bf ( x)  bx  x  bx  b  x    f ( x)  x   f  x   x   b  f  x   x   f  x   x    f  x   x   f  x   x  b  1   f ( x)  x   f ( x)  x  b    f ( x)  x   x  bx  b  x  b   (1)  f ( x)  x   x  (b  1) x   (2) Phương trình (1) có hai nghiệm phân biệt x  1, x  b Xét phương trình (2):    b  1   b2  2b    b  1 b  3  (do (cmt ) b  3)  Phương trình (2) có hai nghiệm phân biệt Ta có:   b  1    b   x  nghiệm (2) b2   b  1 b   b     x  b khơng nghiệm phương trình (2) Vậy phương trình cần tìm có nghiệm phân biệt b) Cho A  m2n2  4m  2n với m, n số nguyên dương Khi n  tìm m để A số phương Khi n  chứng minh A số phương Khi n  ta có: A  4m2  4m    2m  1   4k 2   2m  2k  1 2m  2k  1   2m  2k   m  TH 1:   (tm)  2m  2k   k  2m  2k   1 m  1 TH :   (ktm) 2m  2k   5 k  1  2m  2k   m  TH :   (tm) m  k   k     2m  2k   5 m  1 TH :   (ktm) 2m  2k   1 k  Vậy m  Với n  5, m   A  n2  2n    n  1    n  1 2 A  n2  2n    n    2n    n   ( Do n  5) 2   n    A   n  1 Do A khơng thể số phương Khi m  ta có: A  m n  4m  2n A   mn  1  2mn  4m  2n  A   mn  1   n   m  1   A   mn  1  2(n  2)  5(do m   m   1)  A   mn  1 ( Do n    n     1) Lại có: A  m2n2  4m  2n   mn    mn  1  A   mn  Do A số phương 2 Câu Áp dụng BĐT Cơ si ta có:  ab  ac  bc    ab.ac  bc.ac  ab.bc   3abc  a  b  c    ab  bc  ca  Tiếp tục áp dụng BĐT Cơ si ta có: a  b2  a   b   b  2a    2b   b   2a 2a   2b   b  b 2 a  b  c 2a 2b 2c 2a 2 2b 2 2c P        b  c  a 9a  ab 9b  bc 9c  ca  a  b  c   ab  ac  bc Đặt t  a  b  c  3 ab  ac  bc     t  3 8t  3   8t  27t    a  b  c   27  a  b  c    ab  ac  bc  a  b  c  1  a  b  c    ab  ac  bc  P Vậy GTNN P Đạt a  b  c  Câu a) CMR: Tam giác OCD cân…… Ta có: CMD  900 (góc nội tiếp chắn nửa đường tròn  I  ) CMO  900 ( gt )  D, M , O thẳng hàng CD  AC  AB  AC Mặt khác  ( gt )  AB / / CD ( gt ) Do OCD  COA  COD (slt )  OCD cân D Mặt khác OCD  KEF (góc ngồi góc đỉnh đối diện tứ giác nội tiếp)  KOF  KEF  tứ giác OFEK nội tiếp b) Chứng minh tam giác OEF…… Ta có: OME  ECD (góc ngồi góc đỉnh đối diện tứ giác nội tiếp) Mà ECD  OBE (slt )  OME  OBE  tứ giác OMBE (tứ giác có hai đỉnh nhìn cạnh góc nhau) Do đó: OEB  OMB (hai góc nội tiếp chắn cung OB) Mà OM  OB  OMB cân O  OMB  OBM  AOM  KOF  KEF  OEB  KEF Mà CED  900 (góc nội tiếp chắn nửa đường trịn (I))  KEB  900  OEB  OEK  900  KEF  OEK  900  OEF  900  OE  OF  OEF  CED  90 Do đó:   CED  OFE  CDE OEF ( g.g ) c) Đường thẳng qua điểm (O) (I)…… Ta có tam giác OCD cân D (cmt) có DF  CO (góc nội tiếp chắn nửa đường tròn)  FC  FO (đường cao đồng thời trung tuyến) (1) Tứ giác OEFK nội tiếp (cmt)  OKF  OEF  1800  OKF  1800  OEF  1800  900  900  OK  KF  AB  KF ( gt )  FK / / AC (2) Mà AB  AC Từ (1) (2)  KA  KO (định lý đường trung bình tam giác) Áp dụng phương trình tích đường trịn ta có: HC  HN HM  HA2  HC  HA Do AF , CK , OH ba đường trung tuyến tam giác ACO nên đồng quy Ta có điều phải chứng minh Câu A P M Q K B N C Kẻ MN phân giác BMC  NMP  1200 Xét tứ giác BNMP có: NBP  NMP  600  1200  1800  Tứ giác BNMP tứ giác nội tiếp (tứ giác có tổng hai góc đối 180 )  BPN  BMN  600 (hai góc nội tiếp chắn cung BN)  BPN Tương tự ta chứng minh CNQ  NP / / AQ   Tứ giác NPAQ hình bình hành (dhnb)  AQ  PN  BP  NQ / / AP  AQ  AP  BP  AP  AB  = số Đặt PB  AQ  x  AP  CQ   x Hai tam giác CMQ CAP đồng dạng nên : SCMQ SCAP   CM CQ CM  x  CA.CP CP SCMQ SCAP  SCMQ S APMQ SCAP    CP 1  x  CM CP  1  x  CM CP  1  x  CM SCAP AP AC  x   S ABC AB AC  SCAP  1  x  S ABC  S AMPQ  1  x  CP  1  x  CM  S ABC CP  CM  1  x  1  1  x   CP  Gọi K giao điểm NQ PC, Áp dụng định lý Ta let ta có:  PM BP  MK  KQ KQ  x PM x PM x PM         2 1 x MK 1  x  PM  MK x  1  x  PK  KQ  CQ  AP CA PK AQ x PK x PM x2      PK  xCP   CK CQ  x CP  PK  x CP x  x  CP  CM x2  CP x  x 1 CM 1 x   CP x  x  x  x2  34   S APMQ     1    1  x  x   x  x     12  Vậy GTLN diện tích tứ giác APMQ M tâm tam giác ABC xảy P trung điểm AB hay 12 SỞ GIÁO DỤC VÀ ĐÀO TẠO NAM ĐỊNH ĐỀ THI VÀO 10 CHUN NĂM HỌC 2018-2019 Mơn: TỐN CHUYÊN Câu (2 điểm) a) Rút gọn biểu thức P  x2 y2 x2 y    x  y 1  y   x  y 1  x  1  x 1  y  b) Chứng minh  1 1 1          2018 2 2 2017 20182 Câu (2 điểm) a) Giải phương trình : 1  x  x  x   x   x     x  y   y  x  y  1  x   b) Giải hệ phương trình:  4y  x  14 y  3  x  y 1 1  Câu (3 điểm) Cho đoạn thẳng AB C điểm nằm A B Trên nửa mặt phẳng bờ đường thẳng AB, vẽ nửa đường trịn đường kính AB nửa đường trịn đường kính BC Lấy điểm M thuộc nửa đường trịn đường kính BC (M khác B, C) Kẻ MH vng góc với BC (H thuộc BC); đường thẳng MH cắt nửa đường trịn đường kính AB K Hai đường thẳng AK, CM cắt E a) Chứng minh BE  BC.AB b) Từ C kẻ CN vng góc với AB (N thuộc nửa đường trịn đường kính AB) Gọi P giao điểm NK CE Chứng minh tâm đường tròn nội tiếp tam giác BNE PNE nằm đường BP c) Cho BC  2R Gọi O1; O2 tâm đường tròn nội tiếp tam giác MCH MBH Xác định vị trí điểm M để chu vi tam giác O1HO2 lớn Câu (1,5 điểm) a) Tìm tất cặp số nguyên  x; y  thỏa mãn x2  y  41  xy b) Có số tự nhiên n khơng vượt q 2019 thỏa mãn n3  2019 chia hết cho Câu (1,5 điểm) a) Cho số thực dương thỏa mãn a  b  Chứng minh rằng:  a  b    a  b   4ab  2  a  3b b  3a  b) Cho 100 điểm mặt phẳng cho điểm có điểm thẳng hàng Chứng minh ta bỏ điểm 100 điểm để 99 điểm cịn lại thuộc đường thẳng ĐÁP ÁN Câu 1: a) Điều kiện : x   y, x  1, y  x2 y2 x2 y P    x  y 1  y   x  y 1  x  1  x 1  y    x2 y2  x2 y     x  y   y  x  1  x 1  y   x  x3  y  y x2 y  x  y 1  y 1  x  1  x 1  y  2  x  y  x  y    x  y   x  xy  y  x2 y   x y 1  y 1  x  1  x 1  y  2  x  y   x  y  x  xy  y  x2 y   x y 1  y 1  x  1  x 1  y  x  y  x  xy  y x2 y2   1  y 1  x  1  x 1  y   x  y  x  xy  y  x y (1  y )(1  x) x  y   x ( x  y )  y (1  x )   1  y 1  x  x  y  x  1  y 1  x 1  x    1  y 1  x   x  1  x  y  y  xy  x 1  y   y (1  y )   1 y 1  y 1  x  1  y  x  xy  y   x  xy  y  1 y b) Chứng minh rằng…… 1 1  1 2a  b  c  1 1  Ta có:         2.        a b c abc a b c  ab bc ac  a b c 1  1 1  a  b  c  ta có:        a b c a b c 2 1 1 1      1      Ta có:   (2)  nên    2  2  Tương tự ta có: 1 1 1 1   1   1  2  3 1 1 1 1   1   1  2 (4) 1 1 1 1   1   1  2 2 2017 2018 2017  2018  2017 2018 1 1 1  VT            2018   2018 (dpcm) 2 2017 2018 2018 Câu  x  1  a) Điều kiện: x  x      x  1  pt  1  x  x  x   x  x   2 1  x  x  x    x  x   1  x   1  x  x  x   x  x   x  x     x  x2  2x     x  1 1  x  x  x   x   x2  2x   2x   1  x  x  x    x   x  x    x   x      4 x  x  x    3 x  x    x2  x    x2  2x      x    VN  x  1  x  1   (tm)  x   (ktm) Vậy phương trình có nghiệm x   x   y ( x  y  1)  x      y  1 b) Điều kiện: 8  x   y 1     y  x  y  1  x   x  y   y ( x  y  1)  x  (1)  4y  3  x  y    x  14 y  (2)  Ta có: 1  y  x  y  1  x    x  y    xy  y  y  x    x  y    x  y  y  1  x  y  (*) Đặt *  t ( x  y)  k ( y  1)  x  y   tx   k  t  y  k  x  y   t  t    k  t  3   k  2 k  2   (*)   x  y  y  1   x  y    y  1   x  y    y  1   x  y  y  1  (**) +)TH1: y  1  **  x  1 Khi       1  1     1  14.(1)   3.3   (vô lý) +)TH2: Chia vế phương trình (**) cho y  ta được:    x y x y 0 **    2  y 1  y 1    Khi ta có: x y 1 y 1 (tm) x y  2 (ktm) y 1  x  y  y 1  y  x 1 x 1  x  14 8  2   x  x 1 1 1 2  x  1  8 x   x2  x   x 1 1 2  x  1 Đặt f ( x)   x   x2  x  x 1 1 ; f (8)  11   f (1) f (8)  66  36  Ta có: f (1)  x 1   3 có nghiệm đoạn  1;8 Lại có: f (7)   x  nghiệm (3)  y  Vậy hệ phương trình có nghiệm  x; y    7;3  Câu 3: 1 3 E N K P M A O1 O2F G C H B a) Ta có BME  BKE  900 (góc nội tiếp chắn nửa đường tròn)  Hai điểm M K nhìn BE góc 900  Tứ giác BMKE tứ giác nội tiếp  BEC  BKH (hai góc nội tiếp chắn cung BM) Mà BKH  BAE (cùng phụ với ABK )  BEC  BAE Xét tam giác BEC tam giác BAE có: ABE chung; BEC  BAE (cmt ) BE BC   BE  BA.BC (1) BA BE b) Ta có: ANB  900  ANB vng N  BC.BA  BN (2)  BEC BAE (g.g)  Từ (1) (2)  BN  BE  BNE cân B  BNE  BEN trực NE Ta có: BNP  BAK (Hai góc nội tiếp chắn cung BK) Mà BAK  BAE  BEC  BEC BAE   BNP  BEP (4) (3) B thuộc trung Từ (3) (4)  PNE  PEN  PNE cân P  P thuộc đường trung trực NE  BP trung trực NE Do tam giác BNE cân B tam giác PNE cân P nên trung trực BP đồng thời đường phân giác Do tâm đường tròn nội tiếp tam giác BNE PNE nằm BP c) Gọi G, F giao điểm O1O2 với MC MB 1 2 O H MH  O1HM O2 HB( g.g )   O2 H BH MH MC O H CM OH O H Ta có: MHC BHM ( g.g )       BH BM O2 H BM CM BM Vì O1H ; O2 H phân giác hai góc kề bù Ta có: O1HM  O2 HB  450 O1MH  HMC  MBH  O2 BH  O1H  O2 H  O1HO2  900  BMC Xét tam giác O1HO2 CMB có O1HO2  BMC  900 ;  O1HO2 CMB O1H O2 H  (cmt ) CM BM (c.g.c)  O1O2 H  CBM (hai góc tương ứng) Mà O1O2 H  HO2 F  1800  CBM  HO2 F  1800  Tứ giác HO2 FB tứ giác nội tiếp (tứ giác có tổng hai góc đối 1800 )  MFG  O2 HB  450 (cùng bù với O2 FB)  MEG vuông cân M Xét tam giác MO2 H tam giác MO2 F có: HMO2  FMO2 ( gt ); MHO2  MFG  450 (cmt )  MO2 H  MO2 F MO2 chung  MO2 H  MO2 F ( g.c.g )  MH  MF ; HO2  FO2 Chứng minh tương tự ta có MH  MG HO1  GO1  Chu vi tam giác O1O2 H là: C  HO1  HO2  O1O2  GO1  FO1  O1O2  GF  Để chu vi tam giác O1HO2 lớn  GFmax Xét tam giác vng cân MEF có GF  MG  MF  2MG  2MH  EF  MH  GFmax  MH max  M điểm cung BC Câu a) x  y  41  xy  x  xy  y  x  xy  y  41   x  y    x  y   16  25   4    5   x  y     x  y  5   x  y  4    x  y  5   x  y    x  y  5    x  y  4  x  y   2  x  y    x  y    x  y  5    x  y  4   x  y    x  y  4    x  y  5  x  y     13 14   x    x    (ktm) (ktm)  x    y      3     x   13   x   14     3 ( ktm ) (ktm)      y   y       3     x  (tm)   x  (tm)  y   y      x  1   x  2   y  3 (tm)   y  3 (tm)    Vậy cặp ( x; y ) nguyên thỏa mãn x2  y  41  xy  x; y  1;3 ;  1; 3 ;  2;3 ;  2; 3 b) Có số tự nhiên n… n3  2019   n3  n    n  2019  Ta có : n3  n  n  n  1 n  1 tích số tự nhiên liên tiếp nên  n3  n   Để n3  2019 chia hết cho n  2019 phải chia hết cho Ta có: 2019  3 mod6  n  3(mod6) Đặt n  6k  3 k   ta có:  n  2019   6k   2019    k  336   k  336 Vậy có 337 số tự nhiên n thỏa mãn n3  2019 chia hết cho Câu a) Ta có:  a  3b  b  3a  Co  si  a  3b  b  3a  ab Từ giả thiết ta có: a  b   a  b  ab   ab   a  b  4ab  1  a  b  2  a  3b b  3a    a  b    a  b   1  a  b   a  b 2  3a  6ab  3b  2a  2b  a  b   2a  2b  2ab  3(a  b)2  (a  b)  4ab   4a  4b   8ab  4a  4b   1  2a  2b   (luon dung ) Vậy đẳng thức chứng minh b) Xét tam giác ABC với A, B, C điểm 100 điểm cho Lấy D điểm thứ tư  D AB, D AC, D BC Không tính tổng quát ta giả sử D BC Lấy E điểm thứ Nếu E AB điểm A, D, C, E khơng có điểm thẳng hàng Nếu E AD điểm A, B, C,E khơng có điểm thẳng hàng Nếu E AC điểm A, D, B, E khơng có điểm thẳng hàng Do E BC Tương tự ta chứng minh 95 điểm lại thuộc BC Cho nên bỏ điểm A 99 điểm cịn lại thuộc BC A B D E C SỞ GIÁO DỤC VÀ ĐÀO TẠO QUẢNG NGÃI KỲ THI TUYỂN SINH VÀO LỚP 10 THPT NĂM HỌC 2018 – 2019 Ngày thi: 06/6/2018 Môn thi: Toán (Hệ chuyên Toán) Thời gian làm bài: 150 phút ĐỀ CHÍNH THỨC Bài (2.5 điểm) 5x  1  x   x 1 x  x  1  x b Tìm cặp số thực (x; y) với y lớn thỏa mãn điều kiện x2  y  y  xy   a Cho c Cho rút gọn biểu thức A  số thực khác thỏa mãn điều kiện { Chứng minh Bài (1.5 điểm) a Chứng minh với số tự nhiên n khơng chia hết cho 81 b Một số nguyên dương gọi số may mắn số gấp 99 lần tổng tất chữ số Tìm số may mắn Bài (2.0 điểm) a Giải phương trình √ √  x  y  xy  b Giải hệ phương trình  2 x  y  Bài (3.0 điểm) Cho hình vng ABCD nội tiếp đường tròn (O) Gọi M điểm cạnh BC (M khác B C), N điểm cạnh CD cho BM = CN Gọi H, I giao điểm AM với BN, DC a Chứng minh tứ giác AHND nội tiếp MN vng góc với BI b Tìm vị trí điểm M để độ dài đoạn MN ngắn c Đường thẳng DM cắt đường tròn (O) P (P khác D) Gọi S giao điểm AP BD Chứng minh SM song song AC Bài (1.0 điểm) Trên biểu tượng Olympic có miền ký hiệu (như hình minh họa) Người ta điền số vào miền cho miền điền số, miền khác điền số khác tổng số hình trịn 14 e a b d c a Tính tổng số miền b, d, f h b Xác định cách điền thỏa mãn yêu cầu HẾT Ghi chú: Cán coi thi khơng giải thích thêm k f h g SỞ GIÁO DỤC VÀ ĐÀO TẠO QUẢNG NGÃI KỲ THI TUYỂN SINH VÀO LỚP 10 THPT NĂM HỌC 2018 - 2019 Ngày thi: 06/6/2018 ĐỀ CHÍNH THỨC Mơn: Tốn (Hệ chuyên Toán) Thời gian làm bài: 150 phút HƯỚNG DẪN CHẤM Bài (2.5 điểm) 5x  1  x   x 1 x  x  1  x b Tìm cặp số thực (x; y) với y lớn thỏa mãn điều kiện x2  y  y  xy   rút gọn biểu thức sau A  a Cho số thực khác thỏa mãn điều kiện { c Cho Chứng minh Điểm Tóm tắt cách giải 1.a Rút gọn biểu thức sau 0.25 điểm 0.25 điểm 0.25 điểm 0.25 điểm 1.b Tìm cặp số thực (x; y) với y lớn thỏa mãn điều kiện x2  y  y  xy   Phương trình viết lại x2 - 4yx + 5y2 + 2y - 3=0 Phương trình có nghiệm ’= -y2 - 2y +  3  y  0.25 điểm 0.25 điểm Vì y lớn nên y = 0.25 điểm  x2  x    ( x  2)2   x  0.25 điểm Vậy (x,y) = (2; 1) 1.c Cho minh số thực khác thỏa điều kiện { Chứng Cộng theo vế ta a + b + c = (1)+(2) ta a + b = c2-a2 = (c-a)(c+a) = (-b).(c-a) hay –c = (-b).(c-a) 0.25 điểm Tương tự ta có –b = (-a)(b-c) –a = (-c)(a-b) 0.25 điểm Nhân theo vế đẳng thức ta Bài (1.5 điểm) a Chứng minh với số tự nhiên n khơng chia hết cho 81 b Một số nguyên dương gọi số may mắn số gấp 99 lần tổng tất chữ số Tìm số may mắn Điểm Tóm tắt cách giải 2.a Chứng minh với số tự nhiên n hết cho 81 Giả sử tồn số tự nhiên n để suy không chia , hay => n=3k mà 0.25 điểm nên 0.25 điểm Nhưng k không chia hết cho với Vậy với số tự nhiên n khơng chia hết cho 81 0.25 điểm 2.b Một số nguyên dương gọi số may mắn số gấp 99 lần tổng tất chữ số Tìm số may mắn Giả sử số cần tìm ̅̅̅̅̅̅̅̅̅̅̅̅̅ => ̅̅̅̅̅̅̅̅̅̅̅̅̅ = 99( TH1 m kiểm tra trực tiếp suy vô nghiệm TH2 m Ta ln có { Do m ̅̅̅̅̅̅̅̅̅̅̅̅̅ 0.25 điểm suy bất đẳng thức khơng cịn 0.25 điểm TH3 m = Suy hay nên a1=1 Khi 0.25 điểm Suy Vậy số cần tìm 1782 hay a2 = 7, a4 = 2, a3 = a1 = Bài (2.0 điểm) a Giải phương trình √ √  x  y  xy  b Giải hệ phương trình  2 x  y  Điểm Tóm tắt cách giải √ 3.a Giải phương trình √ Điều kiện: Ta viết lại (√ ) (√ √ 0.25 điểm ) √ √ √ √ √ 0.25 điểm [ 0.25 điểm Mà phương trình √ √ vơ nghiệm, nên nghiệm phương trình ban đầu x= (thỏa điều kiện) 0.25 điểm  x  y  xy  3.b Giải hệ phương trình  2 x  y  0.25 điểm Hệ viết lại thành { Đặt { ta có hệ { 0.25 điểm 0.25 điểm Giải hệ phương trình ta a = b = Với { { suy { { 0.25 điểm Bài (3.0 điểm) Cho hình vng ABCD nội tiếp đường trịn (O) Gọi M điểm cạnh BC (M khác B C), N điểm cạnh CD cho BM = CN Gọi H, I giao điểm AM với BN, DC a Chứng minh tứ giác AHND nội tiếp MN vng góc với BI b Tìm vị trí điểm M để độ dài đoạn MN ngắn c Đường thẳng DM cắt đường tròn (O) P (P khác D) Gọi S giao điểm AP BD Chứng minh SM song song AC Điểm Tóm tắt cách giải A B S H O D M P I C N 4.a Ta có: BM = CN, AB = BC, B  C  900 Nên ABM  BCN (c.g.c) 0.25 điểm Mà BAM  BMA  90  CBN  BMA  90  BHM  90 0 Suy ADN  AHN  1800 , hay tứ giác ADNH nội tiếp 0.25 điểm  IH  BN Ta có BC  CD (gt)  BC  NI 0.25 điểm Do M trực tâm tam giác BIN nên NM  BI (đpcm) 0.25 điểm 4.b Đặt AB = a, BM = x  MC = a – x Ta có MNC vng C  MN2 = CM2 + NC2 0.25 điểm 2 = (a – x) + x = 2x – 2ax + a         =  x - ax  a    x - ax  a   a 2 2  1    x  a   a2  a2  2  Dấu “=” xảy x  a   x  0.25 điểm a 0.25 điểm a Suy MN  Do MN đạt giá trị nhỏ là: a a x 2 Vậy M trung điểm BC MN nhỏ 0.25 điểm 4.c T ó ∠DMC = 900 − ∠PDC mà ∠PDC =∠PAC (cùng chắn cung PC) nên ∠DMC = 900 − ∠PAC 0.25 điểm Do BD trung trực AC nên ∠SAC=∠SCA hay ∠PAC =∠SCA 0.25 điểm Suy ∠DMC = 900 − ∠SCA = ∠DSC Do tứ giác CMSD nội tiếp, mà ∠MCD=900 nên ∠MSD=900 0.25 điểm Hay MS vng góc DB, suy SM song song AC 0.25 điểm Bài (1.0 điểm) Trên biểu tượng Olympic có miền ký hiệu (như hình minh họa) Người ta điền số vào miền cho miền điền số, miền khác điền số khác tổng số hình trịn 14 a Tính tổng số miền b, d, f h b Xác định cách điền thỏa yêu cầu Tóm tắt cách giải Điểm 5.a Gọi a’, b’, , k’ số miền a, b, , k Mỗi hình trịn có tổng 14 nên hình trịn 5.14 = 70 0.25 điểm Khi cộng số miền b, d, f, h cộng hai lần nên b' + d’ + f’ + h’ = 70 - (1 + + … + 9) = 25 0.25 điểm 5.b Theo giả thiết a’ + b’ = h’ + k’ = 14 nên ta có hai cặp thỏa (5;9) (6;8) Do b’ + h’ 11, 13, 15, 17 Dễ thấy b’ + h’ = 11 b’ + h’ = 13 (mà b’ + d’ + f’ + h’ =25) khơng thể thỏa mãn Nếu b’ + h’=17 d’ + f’ = (d’;f’) cặp (1;7) khơng thể có cặp (7;9) (7;8) hình trịn Suy b’ + h’ = 15 0.25 điểm Không tính tổng quát, giả sử b’ = 9, h’ = a’ = 5, k’ = 8, d’ =3, f’ = 7, c’ = 2, e’ = 4, g’ = (hoặc đối xứng lại) 0.25 điểm Ghi : + Mỗi tốn có nhiều cách giải, học sinh giải cách khác mà cho điểm tối đa Tổ chấm thảo luận thống biểu điểm chi tiết cho tình làm học sinh + Bài Hình học, khơng có hình vẽ học sinh thực bước giải có logic cho nửa số điểm tối đa phần đó; vẽ hình sai mặt chất khơng cho điểm + Điểm câu tồn tính đến 0,25 khơng làm trịn số SỞ GIÁO DỤC VÀ ĐÀO TẠO QUẢNG NGÃI KỲ THI TUYỂN SINH VÀO LỚP 10 THPT NĂM HỌC 2018 – 2019 Ngày thi: 06/6/2018 Môn thi: Toán (Hệ chuyên Toán) Thời gian làm bài: 150 phút ĐỀ CHÍNH THỨC MA TRẬN ĐỀ Mức độ Phân mơn SỐ HỌC Các chủ đề Nhận biết Thông hiểu Vận dụng Thấp Cao Cộng Bài 2a Dấu hiệu chia hết 0,75 Tổng hợp Bài 2b 1,5 0,75 ĐẠI SỐ Giải phương trình, hệ phương trình Rút gọn biểu thức Bài 3, 1b 3,0 Bài 1.a 4,5 1,0 Tổng hợp Bài 1c HÌNH HỌC 0,5 Quan hệ vng góc, song song Bài 4.a Bài 4c 1,0 1,0 Cực trị hình học (GTNN đoạn thẳng) 3, Bài 4.b TỔ HỢP 1,0 Tổng hợp Bài 5a Bài 5b 1,0 Tổng cộng 2,0 0,5 0,5 6,25 1,75 10,0 SỞ GIÁO DỤC VÀ ĐÀO TẠO TỈNH ĐIỆN BIÊN Đề thức (Có 01 trang) KÌ THI TUYỂN SINH VÀO LỚP 10 THPT NĂM HỌC 2018 - 2019 Mơn: Tốn (chun) Ngày thi: 06/6/2018 Thời gian làm 150 phút, không kể thời gian giao đề ĐỀ BÀI Câu (2,0 điểm) Cho biểu thức P  x 2 x  3x  x    , ( x  0; x  25) x 1  x x  x  a) Rút gọn P Tìm số thực x để P  2 b) Tìm số tự nhiên x số phương cho P số nguyên Câu (1,5 điểm) a) Trong mặt phẳng tọa độ Oxy , cho đường thẳng (d ) : y  2 x  Parabol ( P) : y  x Tìm tọa độ giao điểm A, B (d ) ( P) Tính độ dài đường cao OH tam giác OAB b) Cho phương trình: x2  m2 x  m   (1), m tham số Tìm tất số tự nhiên m để phương trình (1) có nghiệm nguyên Câu (2,0 điểm) x 16   xy - y = a) Giải hệ phương trình:  y  xy - =  x b) Giải phương trình x  16  x    x Câu (2,5 điểm) Cho hình thang ABCD (AB//CD, AB  CD ) Gọi K , M trung điểm BD AC Đường thẳng qua K vng góc với AD cắt đường thẳng qua M vng góc với BC Q Chứng minh: a) KM // AB b) QD  QC Câu (1,0 điểm) Có tập hợp A tập hợp S  1,2,3 2018 y2  A thỏa mãn điều kiện A có hai phần tử x  A, y  A, x  y x y Câu (1,0 điểm) Trên đường tròn  O  lấy hai điểm cố định A C phân biệt Tìm vị trí điểm B D thuộc đường trịn để chu vi tứ giác ABCD đạt giá trị lớn .Hết KỲ THI TUYỂN SINH VÀO LỚP 10 THPT Năm học : 2018-2019 SỞ GIÁO DỤC VÀ ĐÀO TẠO TỈNH ĐIỆN BIÊN HƯỚNG DẪN GIẢI MÔN TOÁN CHUYÊN CÂU NỘI DUNG Ý Câu Cho biểu thức P  (2,0đ) x 2 x  3x  x    , ( x  0; x  25) x 1  x x  x  a) Rút gọn P Tìm số thực x để P  2 b) Tìm số tự nhiên x số phương cho P số nguyên x 2 x  3x  x     x 1  x x  x  P a ( 1.5  ( x  2)( x  5)  ( x  3)( x  1)  (3x  x  5) ( x  1)( x  5)  x  x  ( x  1)( x  5)  điểm) x    x  25 + Với x  12  x  144 x  , Ta có x số phương nên Khi P   điểm) x   5 x 2  1   x 5 x 5  x  ước Suy +) x   1  x  16 +) x    x  36 +) x    x  144 x   1;1;7  x 1  x 5 x 2 x 2  2   0 x 5 x 5 x  12  + Với b  3x  x  ( x  1)( x  2) x 2  ( x  1)( x  5) x 5 Ta có P  2   ( 0.5 x 2 x 3   x 1  x x 5  Vậy giá trị x cần tìm 16;36;144 Câu a) Trong mặt phẳng tọa độ Oxy cho đường thẳng (d ) : y  2 x  Parabol ( P) : y  x Tìm tọa độ giao điểm A, B (d ) ( P) Tính độ dài đường cao OH tam giác OAB (1.5 điểm b) Cho phương trình: x2  m2 x  m   (1), m tham số Tìm tất số tự nhiên m để phương trình (1) có nghiệm ngun Phương trình hồnh độ giao điểm (d ) ( P) là: x  2 x   x  x  x      x  3 + Với x   y  + Với x  3  y  Vậy tọa độ giao điểm (d ) ( P) A(1;1), B(3;9) Gọi C , D giao điểm (d ) trục Ox,Oy Khi 3  C  ;0  , D  0;3 2  a Đường cao OH tam giác OAB đường cao OH tam giác vng OCD (0.75 điểm) 3 OC.OD   Ta có OC  ; OD   OH  2 OC  OD 3   3 2 Vậy OH  b Phương trình có nghiệm nguyên   m4  4m  số phương ( 0.75 điểm) + Với m  , m    (loại) + Với m     22 (thỏa mãn) + Với m  2m(m  2)   2m2  4m      (2m2  4m  5)      4m   m4  2m2     m4   m2  1     m2  2   khơng phương Vậy m  giá trị cần tìm Câu (2 x 16   xy - y = a) Giải hệ phương trình:  y  xy - =  x b) Giải phương trình x  16  x    x điểm) ĐK: x  0; y  x 16  x 16  xy   xy     y y  Ta có   y  xy   y  x    x y x (1) (2) Giải (2)  6y2  6x  5xy  (2x  3y)(3x  2y)  a (1.0 điểm) 3y 3y 16   Thay vào (1) ta y 2 * Nếu 2x  3y   x  3y 23  (phương trình vô nghiệm) 2y * Nếu 3x  2y   x  Thay vào (1) ta y2   y  3  + Với y   x  (TM) + Với y  3  x  2 (TM) Vậy hệ phương trình có hai nghiệm:  x; y    2;3 ;  x; y    2; 3 b (1.0 ĐK: 1  x      (*)   x  1  x      x    x   điểm)     2x      x    2x    0   x      2x     x  4(TM )  x    Vậy phương trình cho có nghiệm x  Câu Cho hình thang ABCD (AB//CD, AB  CD ) Gọi K , M trung điểm BD AC Đường thẳng qua K vng góc với AD cắt đường thẳng qua M vng góc với BC Q Chứng minh: (2,5 điểm) a) KM // AB b) QD  QC I A K D a (1.0 điểm) E B M Q H R C Gọi I trung điểm AB , E  IK  CD , R  IM  CD  ABD  BDC  Xét hai tam giác KIB KED có  KB  KD   IKB  EKD  KIB  KED  IK  KE (1) Chứng minh tương tự có: MIA  MRC  MI  MR (2) Từ (1) (2) suy KM đường trung bình IER  KM // CD b (1.5 điểm) Do CD // AB (gt) Vậy KM // AB (đpcm) Ta có: IA=IB, KB=KD (gt)  IK đường trung bình  ABD  IK//AD hay IE//AD  QK  IE Suy QK đường trung trực ứng với cạnh IE IER Tương tự ta chứng minh QM đường trung trực ứng với cạnh IR IER Hạ QH  CD QH trung trực ứng với cạnh ER IER Do DE  RC  AB  QH đường trung trực đoạn CD Vậy QC  QD Câu 5( điểm) Có tập hợp A tập hợp S  1, 2,3 2018 thỏa mãn điều kiện A có hai phần tử x  A, y  A, x  y y2 A x y Với tập A tập S  1, 2,3 2018 thỏa mãn đề bài, gọi a b phần tử nhỏ lớn tập A(a, b  S , a  b) Ta chứng minh b  2a Thật vậy, giả sử b  2a , theo giả thiết c  Mà b  2a  b – a  a   c  nhỏ A Vậy b  2a a2  A ba a2 a2   a , mâu thuẫn với a phần tử ba a Gọi d phần tử lớn tập B  A \ b Ta chứng minh b  2d Thật vậy, giả sử b  2d , theo giả thiết d  b  e  Mà b  2d   b – d  d  e  Suy e  A e  B Do eb d2  A bd d2 d d d2  b  d  b2  bd  5d  4b2  4bd  d  (2b  d )2 bd (mâu thuẫn VP số phương, VT khơng số phương) Vậy b  2d  2d  b  2a  d  a Mà a  d ( a d phần tử nhỏ lớn B) nên a  d  b  2a Do A  a;2a Kiểm tra lại ta thấy A thỏa mãn đề Vì a  S 2a  S nên  2a  2018   a  1009 Vậy số tập A thỏa mãn đề 1009 tập Câu (1,0 điểm) Trên đường tròn (O) lấy hai điểm cố định A C phân biệt Tìm vị trí điểm B D thuộc đường trịn để chu vi tứ giác ABCD có giá trị lớn B' A' B C O A D D' Không tổng quát giả sử: AB  AC Gọi B ' điểm cung ABC  AB'  CB' Trên tia đối BC lấy điểm A’ cho: BA’  BA  AB  BC  CA' B'BC  B'AC  B'CA   Ta có: B'CA  B'BA  1800  B'BA  B'BA '  B'BC  B'BA '  180  A’BB’  ABB’  A ' B '  B ' A  B'A  B'C  B'A' B'C  A' C  AB  BC ( B’ A  B’C B’, A, C cố định) Dấu “=” xảy B trùng với B’ khơng đổi Tương tự gọi D’ điểm cung ADC ta có AD’  CD’  AD  CD Dấu “=” xảy D trùng với D’ Vậy chu vi tứ giác ABCD lớn B, D điểm cung AC đường tròn  O  SỞ GIÁO DỤC – ĐÀO TẠO THÀNH PHỐ ĐÀ NẴNG TRƯỜNG THPT CHUYÊN LÊ QUÝ ĐÔN KỲ THI TUYỂN SINH LỚP 10 THPT NĂM HỌC 2018-2019 Mơn thi: TỐN CHUN Thời gian: 150 phút ĐỀ THI CHÍNH THỨC Câu a) Cho biểu thức A   x  1 x 1 Chứng minh rằng: A   x2  x 2x  x  với x  0, x  x  x 1 x b) Tìm tất cặp số nguyên  x; y  thỏa mãn: y   x 1 x  Câu a) Chứng minh phương trình  ax2  2bx  c  bx  2cx  a  cx  2ax  b   ln có nghiệm với số thực a, b, c b) Trên mặt phẳng tọa độ Oxy cho  P  : y  x đường thẳng  d  : y  mx  2m, với m tham số Gọi A H giao điểm (d) với trục hồnh trục tung Tìm tất giá trị m để  d  cắt (P) hai điểm C D nằm hai phía trục tung cho C có hồnh độ âm BD  AC Câu  a) Giải phương trình: x  x  1  x x    4 x    x    y  y  1  10  b) Giải hệ phương trình:    x    x  y  y  1  Câu Cho tam giác ABC nội tiếp đường trịn tâm (O) có A góc tù, AB  AC H trực tâm tam giác Các đường cao AH , BH , CH cắt BC, CA, AB D, E, F a) CMR: AO vuông EF b) Gọi K giao điểm thứ hai AO với đường tròn ngoại tiếp tam giác OHD Chứng minh EF trung trực AK Câu Cho hai đường tròn  I , r   J , R  tiếp xúc với E  r  R  đường thẳng d tiếp tuyến E đường trịn Trên d lấy A C cho E nằm R  EA  EC Các tiếp tuyến thứ  I  vẽ từ A C cắt B, tiếp tuyến thứ hai từ  J  vẽ từ A C cắt D Chứng minh tồn điểm cách đường thẳng AB, BC, CD, DA Câu Cho x, y hai số tự nhiên thỏa mãn y   x2 Chứng minh x tổng bình phương hai số tự nhiên liên tiếp ĐÁP ÁN Câu 1.a Điều kiện : x  0, x  A 2   x  1 x2  x 2x  x   x 1 x  x 1 x  x 1     2 x x x 1 x  x 1 x 1   1 3   x x 1  x  x     x     4  2 4 1 Vậy A  Dấu "  " xảy  x    x  (tm) 4 b y 9x  9x     x  x   x  1 x   x  3x  9x   x  3x   x    x  1   x    x   Có:  x  4  x  1  9 x    x  1  x  1  5  x  1 Ta có: y   x 1   x   x   5  x  6  x 1   x    x   1  x  2 8   x   x   (ktm)   x  6  y    1 (tm) Thay vào biểu thức y ta có:  5 10  (tm)  x   y    1  (ktm)  x  2  y  1  6  Vậy cặp số thỏa mãn toán  x; y    6; 1 ;  0; 1 Vậy cặp số thỏa mãn toán :  x; y    6; 1 ;  0; 1 Câu a)  ax  2bx  c   Ta có:  ax  2bx  c  bx  2cx  a  cx  2ax  b   *  bx  2cx  a  cx  2ax  b   1  4b  4ac   b  ac       4c  4ba   c  ab   2   4a  4bc   a  bc   1       a  b  c  ab  ac  bc    a  b    a  c    b  c  2 (1) (2) (3)  1       Luôn tôn biểu thức    * có nghiệm với a, b, c b) Học sinh tự vẽ hình theo phần trình bày Phương trình hồnh độ giao điểm  P   d  : x2  mx  2m  x2  mx  2m  (*) Có:   m2  8m  d  cắt  P  hai điểm phân biệt  * có hai nghiệm phân biệt m      m2  8m     m  8 d  Ox  A  A  2;0  Ta có:  d  Oy  B  B  0;2m  C D nằm phía trục tung C có hồnh độ âm: x1  0; x2   x1x2   2m   m  Gọi E F thứ tự hình chiếu C lên trục Ox D lên trục Oy CE  y1  mx1  2m  m  x1   BF  yF  yB  y2  yB  mx2  2m  2m  mx2 Ta có: DF / /Ox CE / /Oy nên: m  x1   x 2     mx2 x2 ACE m  0  x2  x1   x1  x1   m m4  x      m   2m    18m  x  2m    2m  4m  18m  16  DBF ( g g )  AC CE   BD FB  m2  7m    m2  7m     m   m  1   m  8   m  (tm) (ktm) Vậy giá trị cần tìm m  Câu a) Giải phương trình…  x  x  1  x x     x  x  3x x   12   10 x  10 x  x x   24   x   x x   x  x  10 x  25    3x  x     x  5  2 x    2 3 x  x   x   2x2   2x   2 x    x      3 x  x    x   x   x   4 x     2 x    x    5  x    5    x  5  13  x       x  5  13 x   2 x  10 x  24       x   x  13    x        x   14 x  40 x  24    x   Vậy tập nghiệm phương trình S    13  5;2 4 x    x    y  y  1  10  b) Giải hệ phương trình:  x   x  y  y  1  (2)     Điều kiện xác định: x  2 Phương trình (2) tương đương với: (1)   x  2   x  2  y3  y   x   y   x  x   y  x    y  1   x     x  y     y  1   x    1      x   x  y      y  1   x      2   y  x2    Thay vào phương trình 1 ta được: y  3 y    y  y  10 Áp dụng BĐT Co si ta có VT  2 y.2   y    y    y   y  VP  y  y  10   y    y  y    y  1  Như phương trình có nghiệm  y    y   x  y     1 (tm) Vậy hệ phương trình có nghiệm  x; y    1;1 Câu a) Chứng minh AO  EF Xét tứ giác BEFC có BEC  BFC  900 Lại có hai góc hai góc có đỉnh liên tiếp nhìn cạnh BC  BEFC tứ giác nội tiếp (dhnb)  FEC  FBC (hai góc nội tiếp chắn cung FC) Xét (O) có AOC  2BFC (góc nội tiếp góc tâm chắn cung AC)  AOC  2BFC (góc nội tiếp góc tâm chắn cung AC)  AOC  2FEC Xét AOC cân O có: OAC  1800  AOC 1800  2.FBC   900  FEC 2  FEC  KAE  900  FEC  FEC  900 Do AOC  KAE (hai góc đối đỉnh)  AO  EF (dpcm) b) Gọi K giao điêm…… Xét HKA ODA ta có: KAH  DAO (hai góc đối đỉnh) HKA  ODA (hai góc nội tiếp chắn cung OH đường tròn ngoại tiếp OHD)  HKA ODA ( g  g ) AK AH    AK AO  AH HD AD AO Xét HEA CDA ta có: HEC  CDA  900 ; EAH  DAC (hai góc đối đỉnh)  HEA CDA ( g.g ) AH EA    AH AD  EA.CA CA DA AK AE  AK AO  EA.CA   AH AD    AC AO Xét AEF ACO ta có: AK AE EAK  OAC (hai góc đối đỉnh),  (cmt ) AC AO  AEF ACO (cgc)  KEA  COA  2FEC (hai góc tương ứng)  EF phân giác KEA  KEA cân K EF đường trung trực AK (dpcm) Câu Gọi M , N , P, Q tiếp điểm AB, BC, CD, DA với đường tròn (I) (J) hình vẽ Theo tính chất hai tiếp tuyến cắt ta có: BM  BN , AM  AE  AQ, DQ  DP, CP  CN  CE Nên tam giác có đỉnh A, B, C, D tam giác cân Phân giac góc tứ giác ABCD trung trực cạnh tứ giác MNPQ Ta có: BMN  AMQ  NMQ  GPB  NPC  NPQ  PNC  ANM  MNP  DQP  AQM  PQD  1800  1800  3600  NMQ  NPQ  MNP  PQM Do MNPQ tứ giác nội tiếp trung trực MN , NP, PQ PM đồng quy, tức phân giác góc tứ giác ABCd đồng quy điểm cách đường thẳng AB, BC, CD, DA Câu Ta có : y   x2  y   x  1 x  1 Dễ có: UCLN x  x  hai số lẻ liên tiếp  x  1và x  số nguyên tố Do 2 x   m  TH 1: 2 x   3n  m   2k  1  x  4k  4k   x  k   k  1 mn  y  2 x   3n  TH : 2 x   m  m  x    3n  mn  y  Số chia dư mà số phương khơng thể chia dư nên vô lý Vậy với x, y  thỏa mãn y   x2 x tổng hai số tự nhiên liên tiếp SỞ GIÁO DỤC – ĐÀO TẠO TỈNH PHÚ THỌ TRƯỜNG THPT CHUYÊN HÙNG VƯƠNG KỲ THI TUYỂN SINH LỚP 10 THPT NĂM HỌC : 2018-2019 Môn thi: TOÁN CHUYÊN Ngày thi: 03/06/2018 Câu 1 b c a) Cho a, b, c số thực đôi khác nhau: a   b   c   x Tính a P  x.abc x b) Cho x, y, z số thực dương thỏa mãn: x  y  z  9;  1   Tính giá y z trị nhỏ biểu thức: T  x3  y3  z  3xyz Câu Cho a số nguyên dương Biết nghiệm x1  x2  x3 phương trình: x  3x    a  x  a  a) CMR: Biểu thức A có giá trị không đổi: A   x1  x2   x12  x22  x32 b) Đặt Sn  x1n  x2n  x3n CMR: S số nguyên lẻ với số n tự nhiên Câu a) Tìm nghiệm nguyên dương phương trình: x  y  3  y  x  3 13x  28 x  24 2x  Câu Cho nửa đường tròn tâm (O) đường kính AB  2R , H điểm cố định OA  H  O; H  A Đường thẳng qua H vng góc với AB cắt nửa đường trịn b) Giải phương trình: x  2 x   C Gọi E giao điểm thay đổi cung AC  E  A; E  C  , F thay đổi cung BC  F  B; F  C  cho EHC  FHC a) Chứng minh tứ giác EHOF nội tiếp b) Gọi R ' bán kính đường trịn ngoại tiếp tứ giác EHOF Tính EHF R  R' c) Chứng minh đường thẳng EF qua điểm cố định Câu Trung tâm thành phố Việt Trì có tát 2019 bóng đèn chiếu sáng thị, chia gồm loại: Đèn ánh sáng trắng có 671 bóng, đèn ánh sáng vàng nhạt có 673 bóng, đèn ánh sáng đỏ có 675 bóng Vào dịp giỗ tổ Hùng Vương, người ta thực việc thay bóng đèn theo quy luật sau: Mỗi lần tháo bỏ bóng đèn khác loại thay vào bóng đèn thuộc loại cịn lại Hỏi đến lúc tất bóng đèn trung tâm thành phố loại không ĐÁP ÁN Câu Câu a b c Ta có: a   b   a  b  Tương tự ta có: b  c  bc bc ca a b ;c  a  ac ab   a  b  b  c  c  a   b c c a a b bc ac ab  abc    abc      abc  1 Nếu abc   P  x giả thiết tương đương với a  ac  b  ba  c  cb  x  x3   a  ac  b  ba  c  cb   abc  a  1 b  1 c  1   a  1 b  1 c  1  a  b  c  ab  ac  cb  3x  x3  abc  ab  ac  bc   a  b  c  ab  ac  bc  a  b  c  x  P   x3  3x      x  1  P  1 Nếu abc  1 , biến đổi hoàn toàn tương tự a  ac  b  ba  c  cb  x  x3   a  ac  b  ba  c  cb   abc  a  1 b  1 c  1   a  1 b  1 c  1  a  b  c  ac  ba  cb  3x  x3  abc  ab  ac  bc   a  b  c   ab  ac  bc  a  b  c   x  2 P   x3  3x     x   P  1 Vậy giá trị P P  P  1 Câu 1b Áp dụng BĐT AM-GM ta có: 1     Do dấu phải xảy x y z x yz xảy giả thiết hay x  y  z  Thay vào T ta T  162 Vậy giá trị nhỏ giá trị T 162 Câu a) Ta có : Phương trình ban đầu tương đương với x    x  1  x  x  a     x   a  x  1 a 1  Ta có:   a     a   x1   a   x1  x3     x2    x1 x3    a  1  a  x   a   (Theo định lý Viet) Thay vào biểu thức A ta được:      P  1 a 1 1  1 a 1 1 1 a 1  P   a 1 1 a 1  a 111 a 1  a 1 P9 Từ ta có điều phải chứng minh b) Đặt Qn  x1n  x3n n   Qo  n   Q1  x1  x3  Qn  x1n  x3n   x1  x3   x1n1  x3n1   x1 x3  x1n  x3n   2Qn1  aQn Theo nguyên lý Quy nạp Q số chẵn với số tự nhiên n Suy : Sn   Qn số lẻ Ta có điều phải chứng minh Câu a) Tìm nghiệm ngun dương phương trình… Ta có: x  y  3  y  x  3 y   x2      y  x   1 2  x2   y x Nếu x      y 1 y Nếu x      y  4(ktm) y Nếu x    x2    x2   y x  y   y  1 y   ( Do x  3)   x  3  x  9   x    x   10      x  1  x  x x x   x  Vậy nghiệm nguyên dương  x; y  hệ phương trình cho 1;1 ;  3;1 b) Giải phương trình: Điều kiện xác định: x  Quy đồng phương trình cho trở thành: x  x  1   x  1 x   13 x  28 x  24  x3  x   x    x   13 x  28 x  24   x3  12 x  28 x  24    2x 1  2x 1  2x 1    x  x  12 x    x    x      x  2   x  2   2x 1  x 1   x3  x  12 x    x   3 2x 1 2x 1 Xét hàm số f  t   2t  4t Với t1  t2  f  t1   f  t2   2t13  2t23  t1  t2   f  t1   f  t2    t1  t2   t12  t1t2  t22    t1  t2   f  t1   f  t2    t1  t2   2t12  2t1t2  2t22     f  t1   f  t2  x  x    x5  2 x  4x   2x 1 x  6x   Vậy phương trình cho có nghiệm nhất: x  Do x   x    Câu a) CMR: EHOF tứ giác nội tiếp Qua E kể dây cung ED vng góc với AB H thuộc trung trực ED  AHD  EHA Ta có: EHC  FHC  900  EHC  900  FHC  EHA  FHB  AHD  EHA  FHB , mà hai góc vị trí đối đỉnh nên D, H , F thẳng hàng Ta lại có: OEF cân O  OEF  OFE OEF  OFE  EOF  1800  2OFE  1800  EOF EOF  900  EDF  AHD  EHA Suy tứ giác EHOF tứ giác nội tiếp (tứ giác có góc ngồi góc đỉnh  OFE  900  đối diện nhau) b) Gọi R’ bán kính đường trịn ngoại tiếp… Gọi I tâm đường tròn ngoại tiếp tứ giác EHOF Vì R  R ' nên IO=IE=IF=EO=OF  IEO IOF tam giác Do vậy: EHF  EOF  1200 (hai góc nội tiếp chán cung EF) c) Chứng minh đường thẳng EF qua điểm cố định Kéo dài FE cắt BA J ta chứng minh J cố định Ta có: OE  OF  sdOF  sdOE  OFJ  OHF Xét tam giác OHF tam giác OFJ có: OHF  OFJ (cmt ) FOI chung  OHF OFJ ( g.g ) OH OF    OH OJ  OF  OC  OC  CJ OF OJ  CJ tiếp tuyến  O  C Vì C cố định nên J cố đinh Ta có điều phải chứng minh Câu Giả sử ta thắp tồn thành phố loại bóng đèn A, hai loại cịn lại bóng loại B C Khi số bóng B C 0, tức hiệu số bóng đèn B C Nếu lần thay trước ta thay bóng A, B thành bóng C hiệc số bóng đèn B C tặng giảm bóng Tương tự thay bóng A, C thành hai bóng đèn B hiệu số bóng đèn B C tăng giảm bóng Nếu thay hai bóng đèn B, C thành hai bóng đèn loại A hiệu số giữ ngun khơng đổi Vậy hiệu số bóng bóng B C 3k  k   Điều trái giả thiết đèn ánh sáng trắng có 671 bóng, đèn ánh sáng vàng nhạt có 673 bóng, đèn ánh sáng đỏ có 675 bóng tức hiệu số bóng đèn bóng Suy điều giả sử sai nên ta khơng thể thắp sáng tồn thành phố bóng đèn màu SỞ GIÁO DỤC VÀ ĐÀO TẠO BẾN TRE ĐỀ THI TUYỂN SINH VÀO LỚP 10 TRUNG HỌC PHỔ THÔNG CHUYÊN BẾN TRE NĂM HỌC 2018 – 2019 MƠN: TỐN (chun) Thời gian: 150 phút (khơng kể phát đề) ĐỀ CHÍNH THỨC Câu 1: (2,0 điểm) a b  a b a  b với a, b hai số thực dương  ab a) Rút gọn biểu thức P :  a  b a  b Cho biểu thức P    b) Tính giá trị biểu thức P a  2019  2018 b  2020  2019 Câu 2: (1,5 điểm) a) Cho p số nguyên tố lớn Chứng minh p  chia hết cho 24 b) Cho phương trình x2  2mx  m   với m tham số Tìm giá trị m để phương trình cho có hai nghiệm phân biệt x1; x2 thỏa đạt giá trị lớn x1  x22 Câu 3: (1,5 điểm) a) Giải phương trình: x3   x  3x   x  y   b) Giải hệ phương trình:  x  y  xy      Câu 4: (2,0 điểm) a) Tìm nghiệm nguyên phương trình: x3  xy   x  y b) Cho hai số thực dương a, b thỏa a  b  Tìm giá trị nhỏ biểu thức   a b Câu 5: (3,0 điểm) Cho nửa đường tròn  O; R  có đường kính AB Vẽ đường thẳng d tiếp tuyến T O  B Trên cung AB lấy điểm M tùy ý ( M khác A, B ), tia AM cắt đường thẳng d điểm N Gọi C trung điểm đoạn thẳng AM , tia CO cắt đường thẳng d điểm D a) Chứng minh tứ giác OBNC tứ giác nội tiếp b) Gọi E hình chiếu N đoạn thẳng AD Chứng minh ba điểm NE AD  2R N , O, E thẳng hàng ND  CO.CD c) Chứng minh CACN d) Xác định vị trí điểm M để 2AM  AN đạt giá trị nhỏ - Hết SỞ GIÁO DỤC VÀ ĐÀO TẠO BẾN TRE ĐỀ CHÍNH THỨC HƯỚNG DẪN CHẤM ĐỀ THI TUYỂN SINH VÀO LỚP 10 TRUNG HỌC PHỔ THÔNG CHUYÊN BẾN TRE NĂM HỌC 2018 – 2019 MƠN: TỐN (Chun) Đáp án Câu Câu P 1   a) P:  ab  ab   a b  a b Điểm (2 đ) 0,25   ab  a b  ab  a  b a  b  a b  a b  0,25   a  b a  b   a  b  a  b  0,25  a  b2 b)  b a  2019  1 2018  0,25 0,25 0,25 0,25 P  2018   2019 1  2018  2019 0,25 Đặt A  p    p  1 p  1 1,5 đ 0,25 Câu Do p  , p số nguyên tố nên p không chia hết cho  p  2k  với k  , k   A  4k  k  1 a) k , k  hai số tự nhiên liên tiếp nên k  k  1  A (1) Ta lại có p  3m  với m  , m  Nếu p  3m  A  3m  3m    A 0,25 Nếu p  3m  A  3m  3m    A Vậy A (2) Do  8,3  nên từ (1) (2) suy A 24 b)  15    m2  m    m     , với m  phương trình cho 2  ln có nghiệm phân biệt với m Nhận xét: đạt giá trị lớn  x12  x22 đạt giá trị nhỏ x1  x2 0,25 0,25 0,25 Ta có x12  x22   x1  x2   x1 x2  4m2  2m  2  31 31 31    2m     với m  Giá trị nhỏ x12  x22 2 4  0,25 Ghi m Câu 1,5 đ 0,25 ĐKXĐ: x  1 Pt cho tương đương:  x  1  x2  x  1   x2  x  1   x  1 (*) Đặt a  x  1, a  b  x  x  1, b  a) 1  (do x  x    x     , với x ) 2  Pt (*) trở thành: b  2a  ab   b  2a  b  a    b  2a (do b  a  ) Với b  2a ta b) Câu a) x2  x   x    37 x   x2  5x       37 x    37 Thử lại ta nhận nghiệm x  2 2 x  y  x  y2         x  y 1  xy    x  y   xy    x  y   2  x  y   x  y  xy   2   x2  y  8 y  y   x  y       x   y x   y   x  y    y     x  x  xy   x  y  y  x  1  x  x  Dễ thấy x  1 pt vơ nghiệm x3  x  2 Với x  1 ta y   x2  x  x 1 x 1 0,25  x   2  x  1    x   1 x 1   x   y  x   2  x  3  y  11 x 1   x   y  x   1  x  2  y  Vậy pt có nghiệm nguyên  x; y   3;11 ,  2;  ,  0;  , 1;1 Để x, y  0,25 0,25 0,25 0,25 2đ 0,25 0,25 0,25 0,25 Ta có a  b   b   a Do a, b    a  1  a  a Khi T     5 a 1 a a 1 a 1  a  a Áp dụng BĐT Cauchy cho hai số dương ; ta được: 1 a a 1  a  a 5  a 1 a 1  a  a Đẳng thức xảy    1  a   a  3a  8a   a 1 a  a   loạ i    a   nhậ n   Vậy T   a  b  3 0,25 0,25 T 2 b) b) 0,25 3đ Câu a) 0,25 Ta có OC  AM (tính chất đường kính dây cung)  OCN  90 0,25 OB  NB (tính chất tiếp tuyến)  OBN  90 0,25 Tứ giác OBNC có OCN  OBN  180  Tứ giác OBNC tứ giác nội tiếp Trong tam giác AND có DC, AB hai đường cao cắt O  O trực tâm tam giác AND 0,25 0,25 c) d)  NO  AD Mà NE  AD  N , O, E thẳng hàng 1 Ta có S AND  NE AD  AB.ND 2 NE AD   AB  R ND Xét hai tam giác vng CAO CDN Ta có CAO  CDN (hai góc có cạnh tương ứng vng góc)  CAO CDN (g.g) CA CO    CA.CN  CD.CO CD CN Tam giác ABN vng B có đường cao BM nên ta có AM AN  AB2  AM  AN  2 AM AN  2 AB  4R 2 AM  AN AB 2 Đẳng thức xảy   AM   AM  R 2  AM AN  AB  AOM vuông cân O  M điểm cung AB 2AM  AN đạt giá trị nhỏ R 0,25 0,25 0,25 0,25 0,25 0,25 0,25 0,25 SỞ GIÁO DỤC – ĐÀO TẠO TỈNH LÀO CAI KỲ THI TUYỂN SINH LỚP 10 THPT NĂM HỌC 2018-2019 Mơn thi: TỐN CHUYÊN Câu a  b3 a b 1) Rút gọn biểu thức biết a, b thực dương: P    a b a b b a 1 2) Cho số dương a, b số c khác thỏa mãn điều kiện    Chứng minh a b c : a  b  a  c  b  c x  3  2  3  2 3) Cho:  3   y  17  12  17  12 Tính giá trị biểu thức M   x  y    x  y  xy  1 Câu 1) Một công ty vận tải dự định dùng loại xe lớn để chở 20 rau theo hợp đồng Nhưng vào cơng việc cơng ty khơng cịn xe lớn nên thay xe có trọng lượng nhỏ so với xe lớn ban đầu Để đảm bảo thời gian hợp đồng công ty cần dùng nhiều xe Hỏi trọng tải xe nhỏ 2) Tìm tất giá trị nguyên m để phương trình: x2  3x  m   có nghiệm thỏa x1  x2  x1 x2  2019 số nguyên Câu Cho đường trịn (w) có tâm O điểm A nằm ngồi đường trịn (w) Qua A kẻ tiếp tuyến AK, AL tới (w) với K, L tiếp điểm Dựng tiếp tuyến d (w) E thuộc cung nhỏ KL Đường thẳng d cắt đường thẳng AL, AK tương ứng M, N Đường thẳng KL cắt OM P ON Q Chứng minh rằng: 1) AOL  AKL 2) MON  900  KAL 3) MQ vng góc ON 4) KQ.PL  EM EN a b c Câu Cho số thực dương a, b, c thỏa mãn điều kiện a  b  c    Chứng minh  a  b  c   8a2   8b2   8c2  Câu 1) Tìm tất cặp số nguyên x, y thỏa mãn y  xy  3x   2) Cho m, n số nguyên thỏa mãn  m  n   mn chia hết cho 225 CMR : mn chia hết cho 225 ĐÁP ÁN Câu 1) Rút gọn 3 a  b a b P   a b a b b a   a a b b a   a   b  a  b  a  b b  a b a a b b a a  a b b a b b  a b  a b   ab   a b a b   a b  ab a b  2) Cho số dương a, b… 1 1 1      1 c  a b   c  Ta có:       a b c  ab  ac  bc  ab  ac  bc   abc ab  ac  bc  ab  acbc2  a  c  b  c   c  ab  ac  bc  c   c2  c2   c  c  0(c  0) Vậy ab  ac  bc 3) Cho…… Ta có:     3 x   2   2 x  3  2  3  2       y  17  12  17  12  y  17  12  17  12   x3   2  3  2  2 3  2  3  2   2    y  17  12  3 17  12 17  12 17  12  17  12  17  12            x3   3x   y  24  y  M   x  y    x  y  xy  1  x3  3xy  x  y   y  3xy  x  y    x  y   x3  y   x  y    x  24  y  x  y  20 Câu 1) Một công ty… Giả sử xe nhỏ chở x   x  20 Khi xe lớn chở x  1(tấn) Theo đề ta có: 20 20  1 x x 1  20  x  1  20 x  x  x  x  x  20   x  4(tm)   x  5(ktm) Vậy trọng tải xe nhỏ 2) Tìm tất giá trị ngun…… Để phương trình có nghiệm  '    m    m   x1  x2   x1 x2  m  Áp dụng định lý Vi-et ta có:   x1  x2   x1 x2  m  Khi theo đề ta có:  Khi theo đề ta có: x1  x2  x1 x2  2019 số nguyên 25 x1  x2    x1 x2   m  4 2019   m     m    U (3) 2019 2019 m  m    m     m   1  m   Vậy giá trị m thỏa mãn m  5, m  Câu N K E Q O A P M L 1) Ta có: AKO  ALO  900  900  1800 (Tứ giác có tổng hai góc đối diện 1800 ) Do đó: AKL  AOL (hai góc nội tiếp chắn cung AL) (đpcm) 2) Ta có: OLM OEM ( g  g )  LOM  MOE  LOE KOE Vì tứ giác AKOL nội tiếp (cmt)  KOL  1800  KAL EON KON ( g  g )  KON  EON  Ta có ngay: MON  MOE  EON   MON  900   1 1 EOL  EOK  KOL  1800  KAL 2 2  KAL 3) Ta có: QOM  MON  QLM nên QOLM tứ giác nội tiếp)  MQO  1800  MLO  1800  900  900 hay MQ vng góc với ON (đpcm) 4) Tứ giác OEML có OLM  OEM  900  900  1800  OEML tứ giác nội tiếp  EMO  ELO  ELK  KLO  EOK  LKO  NOK  LKO  NQK Mà OEM  OML (tính chất hai tiếp tuyến cắt nhau)  NQK  LMO( OME ) Có NKQ  MLP (tính chất hai tiếp tuyến cắt nhau)  MLP QKN ( g.g )  ML LP   KQ.PL  ML.NL  EM EN (đpcm) QK KN Câu Ta có: 18  a  b  c    a  b  c    a  b  c    a  b  c  1         8a   9a    8b   9b    8c   9c  a b c       2 8a  8b  8c    9a   9b   9c a b c Áp dụng bất đẳng thức Cosi ta có: 8a  8b  8c   9a   9b   9c  8a   8b   8c  a b c  18  a  b  c   8a   8b   8c    a  b  c   8a   8b   8c  1(dpcm) Dấu “=” xảy a  b  c Câu 1) Coi phương trình cho phương trình bậc ẩn y có tham số x Ta có:   x2  12 x  Vì x, y    số phương  x  12 x   k  x  12 x   k    x  3  k    2k   k  2k   k    2 x   k    x  1 (tm)   x   k  k       2 x   k  1   x  2   (tm) x   k      k  Thay vào phương trình đề Với x  1  *  y  y     y  1   y  1(tm) Với x  2  *  y  y     y  2   y  2(tm) Vậy tập nghiệm phương trình cho  x; y    1;1 ;  2;2  2) Đặt A   m  n   mn 2 2  A  16  m  n   4mn  16  m  n    m  n    m  n      15  m  n    m  n  2  m  n   m  n  A 225  15      m  n  225  m  n   m  n  25  m  n   m  n  15 m 15   m  n  15      mn 225  m  n   m  n  15 n 15 Vậy  m  n   mn chia hết cho 225 mn chia hết cho 225 SỞ GIÁO DỤC VÀ ĐÀO TẠO HÀ NAM ĐỀ CHÍNH THỨC KỲ THI TUYỂN SINH LỚP 10 THPT CHUN Năm học 2018 - 2019 Mơn: Tốn (Đề chuyên) Thời gian làm bài: 150 phút (Đề thi có 01 trang) Câu (2,0 điểm) Cho biểu thức   1 a 1 a 1 Q      a  2a  (với  a  1) 2 a a  a   a  a   a    1) Rút gọn Q 2) So sánh Q Q3 Câu (2,0 điểm) 1) Giải phương trình  x9 3    x   x 2) Trong mặt phẳng tọa độ Oxy, cho Parabol (P) có phương trình y  x hai đường thẳng (d): y  m ; (d’): y  m2 (với  m  ) Đường thẳng (d) cắt Parabol (P) hai điểm phân biệt A, B; đường thẳng (d’) cắt Parabol (P) hai điểm phân biệt C, D (với hoành độ điểm A D số âm) Tìm m cho diện tích hình thang ABCD gấp lần diện tích tam giác OCD Câu (1,0 điểm) Tìm số nguyên dương x, y thỏa mãn x  3.2 y  Câu (3,0 điểm) Cho đường tròn (O) đường thẳng d cố định ((O) d khơng có điểm chung) Điểm P di động đường thẳng d Từ điểm P vẽ hai tiếp tuyến PA, PB (A, B thuộc đường tròn (O) ) Gọi H chân đường vng góc hạ từ điểm A đến đường kính BC, E giao điểm hai đường thẳng CP AH Gọi F giao điểm thứ hai đường thẳng CP đường tròn (O) 1) Chứng minh E trung điểm đoạn thẳng AH 2) Vẽ dây cung CN đường tròn (O) cho CN song song với AB Gọi I giao hai đường thẳng NF AB Chứng minh IF AF IA  IB  IB AC 3) Chứng minh điểm I thuộc đường cố định P di động d Câu (1,0 điểm) Một học sinh chấm tùy ý điểm phân biệt vào hình trịn bán kính Chứng minh ln tồn hai điểm A, B điểm cho thỏa mãn AB  Câu (1,0 điểm) Cho số thực dương x, y, z thỏa mãn xy  yz  zx  x  y  z Chứng minh x2 x3   y2 y3  z2  z3   - HẾT Họ tên thí sinh……………………………… Số báo danh……………………… Người coi thi số 1………………………………Người coi thi số 2.……………… SỞ GIÁO DỤC VÀ ĐÀO TẠO HÀ NAM KỲ THI TUYỂN SINH LỚP 10 THPT CHUYÊN Năm học 2018-2019 Mơn: Tốn (Đề chun) (Hướng dẫn chấm có 04 trang) HƯỚNG DẪN CHẤM Lưu ý: Các cách gải khác so với đáp án mà cho điểm tương ứng theo phần hướng dẫn chấm Điểm tổng cộng tồn làm trịn đến 0,25; 0,5; 0,75 Câu Điểm Nội dung ý Với < a < 1, ta có: Câu 1 (2,0đ) (1,0đ)   1 a 1 a 1 Q         a  2a   a a  a   a  a   a      1  a  1 a  a2          a  1 2  1 a  1 a a  1  a 1  a   1  a    a     1  a 1  a  1  a  1 a 1        a 1  1 a  1 a a a 1  a   a   a       1  a 1  a     1 a 1 a      a   a   1 a  1 a a 1  a   a   a 1  a 1  a   1  a   1  a   1  a  2a 1 a  1 a 1 a  1 a   1 a  1 a   1 a  1 a   2a 1 a  1 a 2a 1 a  1 a 0,5  0,5 1  a   1  a  0,5  1  a   1  a  2a 1  a    2a 1  a   a  2a Do  a    a   1    a  1    2 3 (1,0đ) Xét Q  Q   a  1  a  1   Vậy Q  Q x    9  x  9  x  Đk:  Với đk trên, pt cho tương đương với x   x  3  x  x   3 x     x  x   * Đặt a   x , b   x ta có a, b  Từ (*), ta có hệ phương trình (1,0đ) Câu (2,0đ) a  2b   2 a  b  18 1  2   Tính A  m ; m , B  m ; m , C  m; m2  , D  m; m2   m  m (  m  )   m  m   m  m   9m Tính SOCD  m3 ; S ABCD   m  m2  Do S ABCD  9.SOCD (1,0đ) Câu (1,0đ) 0,2 0,25 0,25 b  / Thay (1) vào (2) suy  2b  3  b2  18   b  3 Với b  3 loại 216 Với b   x  25  216  ;0  Thử lại, phương trình có tập nghiệm S    25   0,2 0,2  10m m  m  m    1 Đặt m  t   10t  t  t     t   10t  6t     t   2 1 Suy m  Kết luận, m  giá trị cần tìm 4 TH1: x  2k  1 k  , k   , ta có pt : 72 k 1  3.2 y  0,25 0,2 0,2 0,2 0,2 0,2 +)Nếu k  suy x  1, y  nghiệm cần tìm +)Nếu k  suy 3.2 y   73  y   y   mod4  Xét mod hai vế có: 72 k 1  49k  3 mod4  0,2 3.2 y   1 mod4  Suy pt vô nghiệm TH2: x  2k ( với k  , k  1) 72 k  3.2 y   72 k   3.2 y   7k  1 7k  1  3.2 y Do    mod3 kết hợp với (1) suy   k k Vậy 1      3.2   m m y m1  1 m 1  3.2 m 1 m   0,2 y Do 2m1  lẻ 2m1 suy 2m1   m   x     m1 y y    y   Thử lại, suy có hai cặp nghiệm 1;1 ,  2;4  thỏa mãn u cầu 0,2 ( khơng cần trình bày theo ngôn ngữ mod) Câu (4,0đ) (1,0đ) (khơng có vẽ hình học sinh khơng chấm bài) Do HA // PB (Cïng vu«ng gãc víi BC) EH CH nên theo định lý Ta let áp dụng cho CBP , ta có : PB CB Mặt khác, PO // AC (cïng vu«ng gãc víi AB) 1 0,2 0,2  POB  ACB (hai gãc ®ång vÞ) AH CH 0,2 (2)  PB OB Do CB = 2OB, kết hợp (1) (2) ta suy AH = 2EH hay E trung điểm cña 0,2 AH  AHC đồng dạng PBO Do ®ã: Ta có IBF  FCA; BFI  CFA suy BIF đồng dạng CAF IF AF Suy (1)  IB AC IF BF Tương tự, ta có (2)  IA BC (1,0đ) Chứng minh được: PFA đồng dạng PAC suy AF  AP (3) AC PC BF PB Tương tự : PBF đồng dạng PCB suy (4)  BC PC Từ (1), (2), (3), (4) PA  PB suy IA  IB Gọi M chân đường vng góc hạ từ O lên đường thẳng d Gọi K giao điểm hai đường thẳng OM AB Ta có P, I, O thẳng hàng OI  AB Chứng minh: OIK đồng dạng OMP OK OI OP.OI    OK  (1,0đ) OP OM OM OB 2 Mặt khác OP.OI  OB suy OK  cố định, suy điểm K cố định OM Vậy I thuộc đường trịn đường kính OK Câu (1,0 đ) Gọi điểm A, B, C, D, E, F tâm hình trịn O TH1 +) Nếu có điểm trùng với tâm O điều phải chứng minh hiển nhiên Ta giả sử điểm khác điểm O +) Nếu có hai điểm A, B O thẳng hàng O nằm đoạn AB, suy AB  TH2 Không tồn hai điểm A, B O thẳng hàng đồng thời O nằm đoạn thẳng AB Phản chứng, khoảng cách hai điểm điểm lớn Giả sử điểm xếp theo chiều ngược chiều kim đồng hồ A, B, C, D, E, F 0,2 0,2 0,2 0,2 0,2 0,2 0,2 0,2 0,2 0,2   Xét OAB có AB   max OA, OB  AOB  max ABO, BAO Suy AOB  AOB  ABO  BAO  1800  AOB  600 Tương tự : BOC  600 , COD  600 , DOE  600 , EOF  600 , FOA  600 Như : AOB  BOC  COD  DOE  EOF  FOA  3600 vô lý, suy điều phải chứng minh Vậy tình tốn ln x   x2  2x  x2  x   Ta có x3    x    x  x    2 2 y  y6 z z6 Tương tự y   ; z3   2 - Suy   x2 y2 z2 x2 y2 z2        (*) 2 x3  y3  z3   x  x 6 y  y 6 z  z 6 0,2 0,2 0,2 a b2 c  a  b  c      a, b, c, u, v, w  1 - Chứng minh u v w uvw Câu (1,0đ) - Áp dụng (1) (*) ta thu x2  y2  z2  x 8 y 8 z 8 Ta cần chứng minh 2 x  y  z  1 x  y  z   x  y  z   18 3 2 x  y  z  0,2 x  y  z   x  y  z   18  x  y  z   xy  yz  zx   18   x  y  z    x  y  z    x  y  z    xy  yz  zx   18  (3) (2) Lại xy  yz  zx  x  y  z nên ta kiểm tra  x  y  z   3 x  y  z   18  (4)   x  y  z  3 x  y  z    Thật ta có quan hệ  x  y  z   3 xy  yz  zx   3 x  y  z  2 0,2 nên x  y  z  3, từ (4) Từ (2), (3), (4) suy điều phải chứng minh Dấu “=”  x  y  z  - HẾT - 0,2 SỞ GIÁO DỤC – ĐÀO TẠO TỈNH BÌNH PHƯỚC KỲ THI TUYỂN SINH LỚP 10 THPT NĂM HỌC 2018-2019 Mơn thi: TỐN CHUN Câu  a 1 ab  a   a  ab  a    1 :    1 ab  ab   ab    ab   a) Rút gọn biểu thức: T   b) Cho x   Tính giá trị biểu thức H  x5  3x4  3x3  x2  20 x  2023 1 Câu Cho parabol  P  y  x đường thẳng d : y   m  1 x  m2  Với giá trị 2 m d cắt (P) hai điểm phân biệt A( x1; y1 ); B( x2 ; y2 ) cho biểu thức T  y1  y2  x1 x2 đạt giá trị nhỏ Câu a) Giải phương trình : x   x  14  x2   x  1 y  1  10 b) Giải hệ phương trình :   x  y  xy  1  Câu Cho đường trịn (O;R) có đường kính AB CD vng góc với Trên dây BC lấy M (M khác B C) Trên dây BD lấy N cho MAN  CAD , AN cắt CD K Từ M kẻ MH vng góc với AB (H thuộc AB) a) CMR: Tứ giác ACMH nội tiếp, ACMK nội tiếp b) Tia AM cắt (O) E (E khác A), tiếp tuyến E B đường tròn cắt F Chứng minh AF qua trung điểm HM c) CMR: MN tiếp xúc với đường tròn cố định M di chuyển dây BC (M khác B C) Câu a) Tìm tất số nguyên p cho 16 p  lập phương số nguyên dương b) Tìm tất số nguyên  a; b  thỏa mãn  a  b2    a  b   4 Câu a) Cho x; y hai số thực dương CMR: x2 y   x y y x b) Xét số thực a; b; c với b  a  c cho phương trình ax2  bx  c  có nghiệm thực m; n thỏa mãn  m, n  Tìm GTLN GTNN biểu thức M  a  b  2a  c  a(a  b  c) ĐÁP ÁN Câu a) Rút gọn biểu thức : Điều kiện xác định : a  0, b  0, ab   a 1 ab  a   a  ab  a  T     1 :    1 ab  ab   ab    ab    ab  1   ab  1 ab  1 :  ab  1 ab  1  a  1 ab  1   ab  a  ab  1   ab  1 ab  1  ab  1 ab  1    a 1   ab   ab  a  a b  a  ab   ab  ab  a b  a  ab  a b  a  ab   ab  ab  a b  a  ab  : ab  ab   ab a  2a b  ab ab     ab ab  2 a  2 a      b) cho…tính giá trị biểu thức… x     x     x    x2  4x   H  x5  3x  3x3  x  20 x  2023  x5  x  x3  x  x3  x   x  x  1  2018  x3  x  x  1  x  x  x  1   x  x  1  2018   x3  x   x  x  1  2018  2018 ( x  x   0) Vậy H  2018 x   Câu Ta có phương trình hồnh độ giao điểm hai đồ thị hàm số là: x   m  1 x  m2   x   m  1 x  2m2   0(*) 2 Đường thẳng (d) cắt  P  hai điểm  phương trình * có hai nghiệm   '    m  1  2m2    2m  m2    m  2 Với  m   d  cắt  P  hai điểm A  x1; y1  ; B  x2 ; y2   x1  x2   m  1  x1 x2  2m  Áp dụng hệ thức Vi-et ta có:  Ta có: A  x1; x12  , B  x2 ; x22  2     1  T  y1  y2  x1 x2  x12  x22  x1 x2   x1  x2   x1 x2   x1 x2  2   x1  x2   x1 x2     m  1  4m   2m  4m  2  m  2m  1  2  2  m  1  2 Vì  m  1  m  0;2 Đặt t  m   m  0;2  t  1;1  t  0;1  T    m  1   2t  t  0;1 m  m  Vậy MinT   t    m  1    Câu a) Giải phương trình: x   x 14  x2  Điều kiện xác định: x  x   x  14  x   x    x  14   x    x 1   x 1   x  14   x  14  x 1  x  14   x  3 x3    ( x  3)( x  3) x 1  x  14      x  3    x  3  x  14   x 1      x  3 x  3 x      x  (1)  x   x  14    30    10  30   x 1   x 1      x    30 Với x     3   3 6 x  14   x  14    x  14  12  30  3,36 16 VP(1)     5,33 3  VP  VT hay (1)VN  VT (1)  Vậy phương trình cho có nghiệm x   x  1 y  1  10 b) Giải hệ phương trình:   x  y  xy  1  Ta có:  x  1 y  1  10  x  y   xy 2   10   x  y xy   ( x  y )( xy  1)       x  y 2  xy   xy 2   10  ( x  y )( xy  1)  x  y  u Đặt  hệ phương trình trên: xy   v  2   u  v  10  u  v   2uv  10  u  v   16    uv    uv   uv   u  v  u  1, v   u  v   u  3, v   uv     u  v  4    u  v  4 u  1, v  3 uv       uv  u  3, v  1  x  y   x  y    VN  x  2; y    xy     xy   x  2; y   x  1; y   x  y   x  y       x  1; y  xy   xy   x  1; y  2        x  1; y  2    x  2; y    x  y  1   x  y  1  x   2; y   x  0; y    xy   3   xy  2      x  0; y   x  3; y    x  y  3   x  y  3  x  3; y     xy   1   xy    Vậy hệ phương trình có tập nghiệm S  1;2 ; 2;1 ; 1; 2 ;  2;1 ; 0;3 ; 3;0  Câu A D K O G H I C M N Q E B F P a) CMR Tứ giác ACMH nội tiếp, ACMK nội tiếp Ta có: ACB  ACM  900 (góc nội tiếp chắn nửa đường tròn)  AHM  900  MH  AB   ACM  AHM  900  900  1800  ACHM tứ giác nội tiếp (có tổng hai góc đối diện 1800 ) (dhnb) Ta có: MCK góc nội tiếp chắn cung BD  MCK  DB  450 Mà MAK  CAD  450  MAK  MCK  450  ACMK tứ giác nội tiếp (hai đỉnh kề cạnh AC nhìn đoạn MK góc nhau) (dhnb) b) Tia AM cắt (O) E………………… Gọi AF cắt MH I, AM cắt BF P MH AH (định lý Ta let)  PB AB IH AH IH MH  AH  IH//FB  (định lý Ta let)      FB AB FB PB  AB  MH // PB AB nên ta có : Ta có: AEB  900 (góc nội tiếp chắn nửa đường trịn (O))  BEP  900 (Hai góc kề bù) Theo tính chất tiếp tuyến FE, FB cắt nên FE  FB, FEB  FBE (hai góc tạo tia tiếp tuyến dây cung chắn cung BE) Mặt khác: FEP  900  FEB (hai góc phụ nhau) FPE  900  FBE ( PEB vuông E)   FEP  FPE  900  FEB   PEF cân F  FE  FP (hai cạnh bên tam giác cân)  FE  FB  FP  BP  FP  FB  FB IH MH    MH  IH FB FB Do AF qua trung điểm I MH c) CMR MN tiếp xúc với……………… Do tứ giác ACMK nội tiếp : ACM  MKN  900 (hai góc đối diện 900 ) Gọi G giao điểm AM DC Ta có: BCD vng cân B  BDC  450 (tính chất tam giác vng cân) Xét tứ giác ADNG có: NDG  GAN  450  ADNG tứ giác nội tiếp (hai góc nhìn đoạn thẳng góc nhau)  ADN  AGN  1800  AGN  900 hay MGN  900 (hai góc kề bù) Vì MKN  MGN  900  MCKN nội tiếp (hai góc nhìn đoạn thẳng góc nhau)  AMN  AKC (góc ngồi đỉnh góc đỉnh đối diện) Mà AMC  AKC (hai góc nội tiếp chắn cung AC )   AMC  AMN  AKC  Kẻ AQ  MN Q Khi ta có: AMC  AMQ(ch  gn)  AQ  AC (hai cạnh tương ứng) Áp dụng định lý Pytago ta có AC  R2  R  R không đổi A điểm cố định nên M di chuyển dây BC MN ln tiếp xúc với đường trịn A; R   đường tròn cố định (đpcm) Câu a) Do 16 p  lẻ nên ta đặt 16 p    2n  1  8n3  12n2  6n   p  n  4n2  6n  3 (*)  n  * Ta có:  4n2  6n  3 tam thức bậc vô nghiệm, số lẻ lớn không phân n  tích thành tích số tự nhiên  *    4n  6n   p Thử lại ta được: 16.307    2.8  1  4913 thỏa mãn Vậy p  307 số nguyên tố thỏa mãn toán b) Tìm tất số nguyên…… Nhân hai vế phương trình với 12 ta được:  p  307 36  a  b2   84  a  b   48   6a     6b    50  52  52  12  2  6 a     6b    6a   5   a  b  2(tm)  6b     a     a  , b  2(ktm)    6b   5    a  2, b  (ktm)  6a   5   6b   5  1  a  ; b  (ktm)   6 a   3   6a    25     b     a  ; b  (ktm)   6b  2  25  a    6a   1 3        6a  2   b   a  1; b  (ktm)  6b         a  b    6b  2  49    6 a     a    a  ; b  0(ktm)  6b   7   6a   49  b          a  1; b   6a   1    6b        b     a  0; b  (ktm)    6 a      a  ; b  (ktm)  6b    3  a        a  ; b  1(ktm)  6b      a  0, b   6 a     6b   1  6a   7   6b   1 Vậy tập nghiệm phương trình cho  a; b    0;1 ; 1;0  ;  2;2  Câu a) Với x, y  ta có: x2 y   x  y  x3  y  xy ( x  y )  ( x  y )( x  xy  y )  xy  x  y  y x  x2  xy  y  xy  x  xy  y    x  y   x, y x  y  x, y  Vậy BĐT chứng minh, dấu "  " xảy   b) Theo đề ta có phương trình ax2  bx  c  có hai nghiệm m, n   m, n  1  a  b  m  n   a Áp dụng định lý Vi-et ta có:  mn  c  a c  b        a  b  2a  c    a  a   1  m  n   mn  M  b c a  m  n  mn 1  a a 1  m  n   Vì  mn  2; mn   M  1 m  n Vậy MaxM   mn   c  Ta lại có:  m, n   m  n  1  n  m  1  mn    mn  m  n 1  1  m  n  1  m  n  a  b  c  Vậy MinM   m  n    a  c M   m  n  1 SỞ GIÁO DỤC VÀ ĐÀO TẠO ĐỀ THI TUYỂN SINH LỚP 10 TRƯỜNG THPT CHUYÊN NAM ĐỊNH Năm học 2016 - 2017 Mơn: TỐN (chung) - ĐỀ ĐỀ CHÍNH THỨC Dành cho học sinh thi vào lớp chuyên tự nhiên Thời gian làm bài: 120 phút (Đề thi gồm: 01 trang) Câu (2,0 điểm) 1) Tìm điều kiện xác định biểu thức A  x   3 x 2) Tính giá trị biểu thức B  x  x   x với x   3) Tính bán kính đường trịn ngoại tiếp hình vng ABCD , biết cạnh AB  cm 4) Tìm tọa độ giao điểm đường thẳng y   x  parabol y  x Câu (1,5 điểm) Cho biểu thức P  1) Chứng minh P   x2 x  x x 2 x 2 x 1 (với x  0; x  )  x 1 x 2 x 3 x 2 2) Chứng minh x  0; x  P  Câu (2,5 điểm) 1) Cho phương trình x   m  1 x  2m   (với m tham số) a) Tìm giá trị tham số m để phương trình cho có hai nghiệm phân biệt x1 , x2 thỏa mãn x12  x22   x1 x2 b) Tìm giá trị tham số m để phương trình cho có nghiệm lớn 2  2 x  y  xy  x  y  2) Giải hệ phương trình    x  y    x  Câu (3,0 điểm) Cho hình chữ nhật ABCD , kẻ AH vng góc với BD H HE, HF vng góc với AB, AD E F Gọi K , M trung điểm HD , BC I giao điểm AH với EF 1) Chứng minh I trực tâm tam giác ABK 2) Chứng minh tứ giác ABMK tứ giác nội tiếp 3) Chứng minh AH  BE.BD.DF Câu (1,0 điểm) Xét x, y, z số thực dương thỏa mãn xy  yz  zx  Tìm giá trị nhỏ biểu thức S  1   x  yz  y  zx  z  xy  2 - HẾT Họ tên thí sinh:…………………………… Họ tên, chữ ký GT 1:…………………………… Số báo danh:………………………………… Họ tên, chữ ký GT 2:……………………… … ĐÁP ÁN VÀ HƯỚNG DẪN CHẤM SỞ GIÁO DỤC VÀ ĐÀO TẠO NAM ĐỊNH ĐỀ THI TUYỂN SINH LỚP 10 TRƯỜNG THPT CHUN Năm học 2016 - 2017 Mơn: TỐN (chung) - Đề ĐỀ CHÍNH THỨC Dành cho học sinh thi vào lớp chuyên tự nhiên (Hướng dẫn chấm gồm 03 trang) Câu Câu 1) 2) Điểm (2,0đ) Nội dung Biểu thức A  x   Ta có B   x  3 x 1  x  xác định    3 x x  x   x  x3  x   0,5 Với x   , ta có B       3) Đường trịn ngoại tiếp hình vng ABCD có đường kính AC  cm 0,5 AC  cm 2 x  Xét phương trình x   x   x  x      x  2 Với x   y  1; với x  2  y  Tọa độ giao điểm cần tìm 1;1  2;4  Suy bán kính đường trịn R  4) Câu 1) Ta có P      x  1 x  2  x  1 x  2  3x  x P   x 2    x  1 x 1 0,5  x  2 (1,5đ) x 1 0,25 3x  x  x  x   x      2) 0,5   x 1 x 2  0,25 x  x 3  x  1 x  1 x 1  x  3  x  2 0,25 x 2 x 3 x 2 x 3  1 x 2 x 2 Với x  0; x  ta có x 22 0,25 0,25 1 3   1  P x 2 x 2 0,25 Câu 1.a) (2,5đ) Ta có    m  1   2m    m  6m    m  3 0,25 Phương trình có hai nghiệm phân biệt x1 , x2     m  0,25 2 x  x  m 1 Theo hệ thức Viet ta có   x1.x2  2m  x12  x22   x1 x2   x1  x2   3x1 x2  0,25   m  1   2m     m2  4m   m   m  Đối chiếu điều kiện ta m  giá trị cần tìm 1.b) 2) 0,25 x  x   m  1 x  2m     x   x  m  1     x  m  Phương trình có nghiệm lớn m    m  2  1 2 x  y  xy  x  y    2   2x  y    2x  Điều kiện: x  y  (1)   x  y  x  y    x  y     x  y  x  y  1  0,25 0,25 0,25  x  y x  y   x  y  Thế y  x vào (2) ta 0,25 3x   x  0,25 x    x  4 x  11x   Với x   y  (thỏa mãn điều kiện) Vậy hệ cho có nghiệm  x; y    2;2  0,25 (3 đ) Câu 1) M B Tứ giác AEHF hình chữ nhật, suy I trung điểm AH C H E 0,25 K I A F  IK đường trung bình AHD  IK song song AD  KI  AB ( AD  AB ) Xét ABK có KI  AB AI  BD (giả thiết), suy I trực tâm ABK D 0,25 0,25 0,25 2) 3) (1) BI  AK ( I trực tâm ABK ) IK song song AD , IK  AD ( IK đường trung bình AHD )  IK song song BM , IK  BM Do tứ giác BMKI hình bình hành  BI song song MK (2) 0,25 Từ (1) (2) suy MK  AK hay AKM  900 0,25 ABM  AKM  900  900  1800 , tứ giác ABMK tứ giác nội tiếp 0,25 Vì ABD vng A có AH đường cao nên AH  BH DH Do AH  BE.BD.DF  AH AH  BE.BD.DF  AH BH DH  BE.BD.DF BH BE  AH DH  BE.DF  AH DH  BE.DF ( HE song song AD ) BD BA 0,25  AH DH  DF  AH DH  AB.DF BA ABH đồng dạng với DHF (g.g), 0,25 0,25 (*) 0,25 AH DF   AH DH  AB.DF AB DH 0,25 Suy (*) Vậy AH  BE.BD.DF (1 đ) Câu Ta có 1   x  yz  x  yz  2( xy  yz  zx) x  xy  yz  zx   x  y  x  z  Tương tự, ta có S  S 1  0,25   x  y  x  z   y  z  y  x   z  x  z  y  yz xz xy    xz  yz  xy  yz   xy  xz  yz  xz   yz  xy  xz  xy  Với a, b ta có  a  b     a  b  2  a  b  4ab  ab  Áp dụng bất đẳng thức ta được: yz xz xy S   2  xy  yz  zx   xy  yz  zx   xy  yz  zx  4 xy  yz  zx S    xy  yz  zx  xy  yz  zx 0,25 0,25 Đẳng thức xảy x  y  z  0,25 Vậy giá trị nhỏ S Lưu ý: + Các cách giải khác đáp án đúng, phù hợp với chương trình THCS, ban giám khảo thống cho điểm thành phần tương ứng + Điểm toàn tổng điểm câu khơng làm trịn HẾT SỞ GIÁO DỤC VÀ ĐÀO TẠO TRÀ VINH KỲ THI TUYỂN SINH VÀO LỚP 10 THPT CHUYÊN NĂM HỌC 2018 – 2019 MÔN THI: TỐN Thời gian: 150 phút (khơng kể thời gian phát đề) Bài (2,0 điểm) Cho biểu thức: Q   x  1  2 x  y  x  y2 x  y : với x  y   x  x2  y  Rút gọn Q Xác định giá trị Q x  y Bài (1,0 điểm) Cho đường thẳng (d ) : y  ax  b Tìm a, b biết đường thẳng (d) tiếp xúc với parabol ( P) : y  x điểm A(1;1) Bài (2,0 điểm) x2 Giải phương trình:  x2    x2  x2  y  y  Giải hệ phương trình:  xy  x   Bài (1,0 điểm) Với a, b, c độ dài ba cạnh tam giác Chứng minh phương trình sau ln có nghiệm: (b2  c2  a ) x2  4bcx  (b2  c  a )  Bài (1,0 điểm) Cho x, y, z ba số thực dương thỏa mãn: x  y  z  2 x3  y  z Chứng minh:     x  y y  z z  x2 xyz Bài (3,0 điểm) Từ điểm A nằm bên ngồi đường trịn (O; R), vẽ hai tiếp tuyến AB, AC với đường tròn (B, C tiếp điểm) Trên cung nhỏ BC lấy điểm M, vẽ MI  AB, MK  AC ( I  AB, K  AC ) Chứng minh AIMK tứ giác nội tiếp đường tròn Vẽ MP  BC ( P  BC ) Chứng minh MPK  MIP Xác định vị trí điểm M cung nhỏ BC để tích MI MK MP đạt giá trị lớn HẾT HƯỚNG DẪN GIẢI VÀ BIỂU ĐIỂM DỰ KIẾN: Câu Phần Q   1) Bài (2,0đ)    Nội dung   x x y :  1  2 2 x  y  x  y  x  x  y Điểm x  x2  y x  x2  y   y x2  y x2  y x x x2  y x x2  y  x2  x2  y  y x2  y y  x y  1.5 x2  y 2 x  y x  y x y x y Vậy Q  x y với x  y  x y Thay x  y (thỏa mãn ĐK) vào biểu thức Q, ta được: 2) Q 3y  y 3y  y  2y 4y  2 0.5 x  y Vì đường thẳng (d ) : y  ax  b qua điểm A(1;1) nên ta có:  a  b  b  a  (1) Xét phương trình hồnh độ giao điểm (d) (P): ax  b  x2  x2  ax  b  (2) Thay (1) vào (2) được:  x  1 x  ax  a    ( x  1)( x   a)    x  a 1 Vậy Q  Bài (1,0đ) 1.0 Vì (d) tiếp xúc với parabol ( P) : y  x điểm A(1;1) nên phương trình (2) có nghiệm kép x1  x2  1  1  a   a  2  b  2   1 Vậy a  2; b  1 Bài (2,0đ) 1) x2  x    x  x  x   16  x ĐK:  x  2 (1) 1.0 Đặt y  x  ( y  0)  x  y  Phương trình (1) trở thành: y   y  16   y     y  2   y2  y    y2  y    y (do y   y   0)  y2  y    ( y  2)(2 y  3)   y   (do y   0) 3 Với y  , ta có: 2 25 3 x      x2  x 2 Kết hợp với điều kiện  x    y Vậy nghiệm phương trình cho x   2  x  y  y  (1)  (2)  xy  x  Với x  , phương trình (2) trở thành  (vơ lí) Với x  , ta có:  x2  y  y    x  ( y  1)  2 x  y  y 1      1  xy  x   y  1  y 1  x x   2) 1  x      x   x (do x  0)  x 1.0   x  1   x    x  1 Với x   y    y  1  y0 Với x  1  y   1 Vậy nghiệm hệ phương trình ( x, y)  (1;2),(1;0) Bài (1,0đ) (b2  c2  a ) x2  4bcx  (b2  c  a )  Vì a, b, c độ dài ba cạnh tam giác nên: a, b, c  0; b  c  a  0; a  b  c  0; a  c  b  Xét trường hợp: + TH1: b2  c2  a  (1) (2) 1.0 Phương trình (1) trở thành: 4bcx   x  (do b, c  0)  Phương trình (1) có nghiệm + TH2: b2  c2  a   Phương trình (1) phương trình bậc hai Xét  '  (2bc)2  (b2  c  a )2  (2bc  b  c  a )(2bc  b  c  a ) 2   b  c   a   a   b  c        a  b  c  b  c  a  a  b  c  a  c  b  Kết hợp với (2)   '   Phương trình (1) có nghiệm * Kết luận: Vậy phương trình (1) ln có nghiệm Vì x, y, z  0; x  y  z  nên: 2 x3  y  z    3 x2  y y  z z  x2 xyz (1) x2  y  z x2  y  z x2  y  z x2 y2 z2       3 x2  y y2  z2 z  x2 yz zx xy z2 x2 y2 x2 y2 z2  1 1 1    3 x  y2 y  z2 z  x 2 yz zx xy Bài (1,0đ) x2 y2 z2 x2 y2 z2      y  z z  x x  y 2 yz zx xy z2 z2 2 Lại có: ( x  y )   x  y  xy   x  y 2 xy y2 y2 z2 z2 Tương tự, ta có:  ;  z  x 2 zx x  y 2 xy  (2)  (1) (đpcm)  (2) B I M P O A 0.25 Bài (3,0đ) K C 1) Tứ giác AIMK có: AIM  AKM  900 (GT)  AIM  AKM  180  AIMK tứ giác nội tiếp 0.75 2) Chứng minh tương tự phần 1), ta có tứ giác BIMP, CKMP nội tiếp Tứ giác BIMP nội tiếp  I1  B2 Tứ giác CKMP nội tiếp  C2  P2   Mà B2  C2   sđMC     P2  I1 (đpcm) Chứng minh tương tự phần 2), ta có P1  K1  MPK  MIP có: P2  I1; K1  P1  MPK # MIP (g.g) MP MK   MP  MI.MK MI MP  MI.MK.MP  MP MP  MP3 Do đó, tích MI.MK.MP lớn  MP3 lớn  MP lớn  M điểm cung nhỏ BC Vậy M điểm cung nhỏ BC tích MI.MK.MP đạt giá trị lớn 1.0  3) Thầy Nguyễn Mạnh Tuấn Trường THCS Cẩm Hoàng – Cẩm Giàng – Hải Dương 1.0 SỞ GIÁO DỤC VÀ ĐÀO TẠO THANH HĨA ĐỀ CHÍNH THỨC Đề thi gồm 01 trang KỲ THI VÀO LỚP 10 THPT CHUYÊN LAM SƠN NĂM HỌC 2018 - 2019 Mơn: TỐN (Dành cho thí sinh thi vào lớp chuyên Toán) Thời gian làm bài: 150 phút (Không kể thời gian giao đề) Ngày thi: 09 tháng năm 2018 Câu I: (2,0 điểm)      Tính giá trị biểu thức: P  1  1   1             2018  Cho hai số thực a , b thỏa mãn hệ thức a3  3a  5a  17  b3  3b2  5b  11  Chứng minh a  b  Câu II: (2,0 điểm) Giải phương trình: x  x   x  1(1  x) 1  1 2 x y Giải hệ phương trình:   x   y   xy   Câu III: (2,0 điểm) Tìm tất cặp số nguyên  x; y  thỏa mãn: x2019  y 2019  y 1346  y 673  2 Cho n số nguyên dương tùy ý, với số nguyên dương k đặt Sk  1k  2k   nk Chứng minh S2019 S1 Câu IV: (3,0 điểm) Cho tam giác nhọn ABC có AB  AC Gọi D, E, F chân đường cao kẻ từ A, B, C tam giác, P giao điểm đường thẳng BC EF Đường thẳng qua D song song với EF cắt đường thẳng AB, AC, CF Q, R, S Chứng minh tứ giác BQCR tứ giác nội tiếp PB DB  Chứng minh D trung điểm QS PC DC Khi B, C cố định điểm A thay đổi thỏa mãn điều kiện trên, chứng minh đường tròn ngoại tiếp tam giác PQR qua điểm cố định Câu V: (1,0 điểm) Một giải đấu bóng chuyền có n đội tham gia ( n  , n  ), luật đấu sau: Mỗi đội đấu với tất đội khác trận Sau trận đấu, đội thắng điểm, đội thua điểm; cịn hai đội hịa đội điểm Sau giải đấu đội xếp hạng theo điểm số từ cao xuống thấp (hai đội điểm xếp hạng) Hỏi chênh lệch điểm lớn đội xếp thứ hạng liền bao nhiêu? Hết -Họ tên thí sinh: Số báo danh: Chữ ký giám thị 1: Chữ ký giám thị 2: ĐÁP ÁN CHI TIẾT Câu I.1 n(n  1) (n  1)(n  2) Từ suy ra:   , (n  , n  1)     n n(n  1) Cho n giá trị từ 2,3, , 2018 nhân biểu thức ta được: 1.4 2.5 2017.2020 1.2.3 2017 4.5 2020 P  2.3 3.4 2018.2019 2.3.4 2018 3.4 2019 2020 1010 Vậy P   3.2018 3027 Với n  , n  1, ta có:     n  Câu I.2 Ta có:  a  1   a  1  14   b  1   b  1  14  3 Cộng vế hai biểu thức ta được:  a  1   b  1   a  1   b  1  3 x  a 1 Đặt  đẳng thức trở thành: x3  y3  x  y   ( x  y)( x2  xy  y  2)   y  b 1 y  3y2    0)  x  y  (do x  xy  y    x    2  Vậy (a 1)  (b 1)   a  b  Câu II.1 Điều kiện xác định: x  Ta có: x2  x   x  1(1  x)  x2  x x   x   2( x  x  1)   Đặt x  x   y (điều kiện y  )  y  1 Phương trình trở thành y  y      y  (do y  ) y  1  x  1  x     x   x  Khi : x  x    x    x    x  x  10   x   Vậy phương trình có nghiệm x  Câu II.2  x, y   Điều kiện xác định:  xy    x   0; y    Hệ phương trình cho tương đương với hệ: 2 2 2 2   (1) x  y  x y x  y  x y  2  2 2 2 (2)    x y  xy   ( x  y )  x y  x  y   xy    xy  1 Từ (2) ta có: x y  xy      xy  +Với xy  , thay vào (1) suy x    x   x   x Từ suy hệ có nghiệm: ( x; y)  2; ; ( x; y)   2;  (thỏa mãn điều kiện)  +Với xy  1 , thay vào (1) suy x      (vô nghiệm) x2 Vậy hệ có nghiệm: ( x; y)      2; ; ( x; y)   2;  Câu III.1 Giả sử  x; y  cặp số nguyên thỏa mãn phương trình cho Đặt a  x673 , b  y 673 ta có a, b  thỏa mãn a3  b3  b2  b  Ta thấy: a3   b  1   b  1    b  1 a3  b3   b  1 b   3 +) Nếu b  b  2  b  1 b    Suy ra:  b 1  a3  b3  b   a  b điều khơng thể xảy a, b  +) Nếu 2  b  1, b  suy b 2;  1;0;1 Với b  2  y  2  y  Với b   a   x673   x  673   y  1  x  Với b  1  a    673 (thỏa mãn)  x   y  1  673 673   y  x  Với b   a    673 (thỏa mãn)  x   y   Vậy cặp số nguyên thỏa mãn  x; y   1;1 ( x; y)  1; 1 Câu III.2 n(n  1) Để chứng minh toán ta cần chứng tỏ 2S2019 n(n  1) Ta có S1     n  Ta có nhận xét sau: Với a, b nguyên dương  a 2019  b2019  (a  b) Thật : a2019  b2019  (a  b)(a 2018  a 2017b   ab2017  b2018 ) (a  b) Xét hai trường hợp: +) Nếu n lẻ: Từ nhận xét ta có 2S2019  2n2019  12019  (n  1)2019   22019  (n  2) 2019   2S2019  n  2019  n  2019        n       2(12019  n2019 )  22019  (n  1)2019    n  2019  n  2019   n  2019  n   2019              (n  1)           Mặt khác n n  nguyên tố nên 2S2019 n(n  1) +) Nếu n chẵn: Ta có 2S2019  2n2019  12019  (n  1)2019   22019  (n  2) 2019    n  2019  n  2019   n 2019  n  2019               n           2S2019  2(12019  n2019 )  22019  (n  1)2019    n  2019  n  2019   n  2019  n   2019                (n  1)           Suy 2S2019 n(n  1) Vậy S2019 S1 Câu IV.1 A E F R H S P B D M C Q Do AB  AC nên Q nằm tia đối tia BA R nằm đoạn CA, từ Q, C nằm phía đường thẳng BR Do tứ giác BCEF nội tiếp nên AFE  BCA Do QR song song với EF nên AFE  BQR Từ suy BCA  BQR hay tứ giác BQCR nội tiếp Câu IV.2 Gọi H trực tâm tam giác ABC Dễ thấy tứ giác AEHF nội tiếp Suy FEH  FAH  BAD Tương tự DEH  DCH  BCF Mà BAD  BCF (cùng phụ với ABC ) Suy FEH  DEH , hay EB đường phân giác góc E tam giác DEP ED BD Theo tính chất đường phân giác ta có :  EP BP ED CD Mặt khác AC  BE nên AC phân giác DEP , nên  EP CP CD BD PB DB Suy :    CP BP PC DC Chứng minh tương tự ta có FC phân giác góc DFE , hay DFS  SFE Mà SFE  FSD (so le QR EF ) Vậy DFS  FSD hay tam giác DFS cân D  DF  DS Ta lại có, tính chất phân giác ngồi nên DFQ  AFE Mà AFE  FQD (đồng vị) Suy tam giác DFQ cân D  DF  DQ Từ suy DS  DQ , hay D trung điểm QS Câu IV.3 Gọi M trung điểm BC Ta chứng minh DP.DM  DQ.DR từ suy đường trịn ngoại tiếp tam giác PQR qua M cố định Thật vậy, tứ giác BQCR nội tiếp nên DQ.DR  DB.DC (4) PB DB Theo câu 2) ta có   DB.PC  DC.PB  DB  DP  DC   DC  DP  DB  PC DC  DC  DB   DB.DC  DP     Do M trung điểm BC AB  AC nên DC  DB  DM Do DP.DM  DB.DC (5) Từ (4) (5) ta DP.DM  DQ.DR Suy tứ giác PQMR nội tiếp hay đường tròn ngoại tiếp tam giác PQR qua trung điểm BC cố định Câu V Kí hiệu Di , i  1, n đội bóng thứ i d i điểm số đội Di sau giải đấu Khơng tính tổng quát, giả sử d1  d2   dn Xét hiệu di  di 1 , i  1, n  , ta có di  di 1  0,  i  1, n  Giả sử đội xếp hạng s đội xếp hạng s  có chênh lệch điểm lớn nhất, nghĩa hiệu d s  d s 1 lớn số hiệu Ta có nhận xét: Sau trận đấu, dù kết nào, tổng số điểm hai đội tham gia thi đấu Chia đội bóng làm hai nhóm Nhóm gồm đội D1 , , Ds nhóm gồm đội cịn lại Ds 1 , , Dn s ( s  1) Khi s đội nhóm đấu với trận nhận s(s  1) điểm Ngoài đội thuộc nhóm đấu với đội thuộc nhóm tất (n  s)s trận nhận không 2(n  s)s điểm (vì số (n  s)s trận có trận mà đội thuộc nhóm thua) Do tổng điểm mà s đội nhóm nhận khơng q s(s 1)  2(n  s)s  (2n  s  1)s (2n  s  1) s Từ suy d s  (1)  2n  s  s (n  s)(n  s  1) Lại có: Các đội thuộc nhóm đấu với trận nhận (n  s)(n  s  1) điểm Do (n  s)(n  s  1) số điểm d s 1 đội Ds 1 lớn  n  s  1, hay d s 1  n  s  ns (2) Từ (1) (2) suy ra: ds  ds 1  (2n  s  1)  (n  s  1)  n Dấu ‘=’ xảy đội vô địch thắng tất đội 2(n  1) điểm, tất đội lại khác hịa (và thua đội vơ địch), đội nhận n  điểm Vậy max(di  di 1 )  n SỞ GIÁO DỤC VÀ ĐÀO TẠO THÁI BÌNH ĐỀ CHÍNH THỨC Đề thi gồm 01 trang ĐỀ THI TUYỂN SINH VÀO LỚP 10 THPT CHUN THÁI BÌNH NĂM HỌC: 2018-2019 MƠN THI: TỐN (Dành cho thí sinh thi chun Tốn, Tin) Câu (2,0 diểm) 1) Cho phương trình x2  2mx  m2  2m   (1) (với m tham số) Tìm m để phương trình (1) có hai nghiệm khơng âm x1; x2 Tính theo m giá trị biểu thức P  x1  x2 tìm giá trị nhỏ P 2) Cho hàm số y  x2  Tìm tất giá trị x nguyên x2 Câu (2 điểm) 1) Cho số a; b; c thỏa mãn điều kiện a  2b  5c  Chứng minh phương trình ax2  bx  c  có nghiệm 2) Giải phương trình:  x3  x  3  x3 : 3 Câu (1 điểm) Hai nến chiều dài làm chất liệu khác nhau, nến thứ cháy hết với tốc độ giờ, nến thứ hai cháy hết với tốc độ Hỏi phải bắt đầu đốt lúc chiều để chiều phần lại nến thứ hai dài gấp đơi phần cịn lại nến thứ nhất?  Câu (1,0 điểm) Cho biểu thức x   x  y    y  2018 Tìm giá trị nhỏ biểu thức P  x  y Câu (3,5 điểm) 1) Cho tam giác ABC có AB  4, AC  3, BC  , đường cao AH Trên nửa mặt phẳng bờ BC chứa điểm A vẽ hai nửa đường tròn đường kính BH HC Hai nửa đường trịn cắt AB, AC E, F a) Tính diện tích nửa đường trịn đườn kính BH b) Chứng minh tứ giác BEFC nội tiếp đường thẳng EF tiếp tuyến chung hai đường trịn đường kính BH CH 2) Cho nửa đường trịn đường kính AB  2R Tìm kích thước hình chữ nhật MNPQ có hai đỉnh M, N thuộc đường tròn , hai đỉnh P, Q thuộc đường kính AB cho điện tích MNPQ lớn Câu (0,5 điểm) Cho a,b,c ba số thức dương thỏa mãn điều kiện : 1   1 a b c Tìm giá trị lớn biểu thức P 5a  2ab  2b  5b  2bc  2c  5c  2ca  2a ĐÁP ÁN Câu 1) Phương trình có hai nghiệm khơng âm m   m  m  2m    '       S    2m   m  P   m  2m       m  1   Gọi x1; x2 hai nghiệm phương trình cho, ta có: m2 (luon dung )  x1  x2  2m (định lý Vi-et)  x x  m  m   P  x1  x2   P  x1  x2  x1x2  2m  m2  2m  Với m  ta có: P  2m  m  m     2       P2   P  2 Dấu "  " xảy  m  Vậy Min 2) P  2 m  x2  x2   6   x2 x2 x2 x2 Để y    x  2 U  6  1; 2; 3; 6 Ta có: y  x2 x -1 -3 -1 tm tm Vật tập hợp giá trị -2 -3 -6 -4 -5 -8 tm tm tm tm tm x để y nguyên 3; 1; 4;0; 5;1; 8;4 tm Câu 1) a  2b  5c   b  a  5c 2 a  10ac  25c a  6ac  25c  a  3c   16c   b  4ac   4ac    0a; b; c 4 2  Phương trình ax2  bx  c  ln có nghiệm 3 2) Giải phương trình  x3  x  3  x3 :  x3  x  3  x3 : 32 3  x    x  1  x  x  3  x   x  x  3     x  1  x 1 3 3 3  x   x  Dễ thấy    1x  1;     1  x 1  x 1 x  1 Để phương trình có nghiệm x2  x    x  x     x  1   vô nghiệm Vậy phương trình cho vơ nghiệm Câu Giả sử chiều dài hai nến L  cm  Gọi thời gian đốt hai nến để đượ phần cịn lại nến thứ hai gấp đơi phần lai nến thứ x (giờ)  x   Theo đề ta có, đốt độ dài nến thứ thứ hai L L , (cm) Trong x độ dài nến thứ thứ hai đốt xL xL , (cm)  Độ dài nến thứ thứ hai lại sau đốt x (giờ) là: xL xL L  , x  (cm) Theo đề ta có phương trình xL xL  x 2x   2 L    1     5x 12  1 x   2, (tm) 12 L Vậy phải đốt hai nến 2,4 hay phải đốt hai nến lúc  2,4  1,6 =1 36 phút chiều để yêu cầu toán Câu Từ giả thiết ta có: x   x  2018 y   y2    2018 y   y Tương tự ta có: y   y  2018  x  x y  1  y 2   Cộng vế hai phương trình ta được: 2019  x  y   2017 Xét A     x2   y  x2   y     x  y  1  x 1  y   A   x  y  1  xy     x  y   VP  2017   x  y  2  VT  2019( x  y )  2017   x  y   2019 P  2017  P  20192 P  2017   P   P2  4.2017 P 2.4036 2017 2017 2018  2018 2018 Dấu "  " xảy  x  y  2017 2018 4036   2018   y2  y  Vậy MinP  2017 2018 2017 2018 x  y  2018 4036 Câu A E F B O1 H O2 C 1) a) Tính diện tích nửa đường trịn đường kính BH Ta có: AB2  AC  BC  ABC vuông A( định lý Pytago đảo) Áp dụng hệ thức lượng trịn tam giác vng ta có: BH  AB 16  BC  Diện tích nửa đường trịn đường kính BH 2  BH    32 S           dvdt    5 25 b) Chứng minh tứ giác BEFC nội tiếp và…… Gọi O1 O2 trung điểm BH CH Dễ nhận thấy AEHF hình chữ nhật (tứ giác có góc vng)  CAH  FEH Mà CAH  ABC (cùng phụ với BAH )  FEH  ABC Mà ABC nội tiếp chăn cung HE đường tròn  O1  ; FEH tạo dây cung EH EF vị trí góc tia tiếp tuyến dây cung  EF tiếp tuyến đường trịn đường kính BH Chứng minh tương tự ta có EF tiếp tuyến đường trịn đường kính CH Vì AEHF hình chữ nhật  AEF  AHF Mà AHF  ACB (cùng phụ với FHC )  AEF  ACB Mà AEF  BEF  1800  ACB  BEF  1800  Tứ giác BEFC tứ giác nội tiếp (Tứ giác có tổng hai góc đối 180 ) 2) Cho nửa dường trịn dường kính AB=2R……… M A N Q O P Đặt MN  x ta có: AQ  R  x; BQ  R  x Áp dụng hệ thức lượng tam giác vng AMB ta có: MQ  AQ.BQ  R  x Khi ta có: SMNPQ  MN MQ  x R  x  x  R  x  R Dấu xảy  x  R  x  x  R  x  x   MN  x  R 2; MQ  x  Câu R R B 1    5a  2ab  2b  27 27. 5a  2ab  2b2   5a  2ab  2b 2  27  1     2  5a  2ab  2b 27  Chứng minh tương tự ta có: 5b  2bc  2c 2  27  1  27  1   ;      2 2 2  5b  2bc  2c 27  5c  2ca  2a  5c  2ca  2c 27  27  1 1  P       5a  2ab  2b   5b  2bc  2c   5c  2ca  2a     Sử dụng BĐT 11 1      ta có: x y  z 9 x y z  1 1 1     2  2  2 2  5a  2ab  2b  3a   2ab  a    a  2b   3a 2ab  a a  2b  1 1  1   1                3a  ab ab a   a b b   1 2    2  2  9a 9ab 9b  Ta lại có :  Cauchy  1   2 2 9ab 9 a b  1  1              5a  2ab  2b2  9a 9a 9a 9b   3a 3b  Chứng minh tương tự: 1    2 2 5b  2bc  2c  3b 3c  1      5c  2ca  2a  3c 3a  1   2 5a  2ab  b 5b  2bc  2c 5c  2ca  2a     1   1 1                    3a 3b   3b 3c   3c 3a   a b c   P 27  1      9 9 a  b  c  Dấu "  " xảy   1 abc      a b2 c2 Vậy Pmax  3 SỞ GIÁO DỤC VÀ ĐÀO TẠO TỈNH TIỀN GIANG ĐỀ THI CHÍNH THỨC (Đề thi có 01 trang, gồm 05 bài) KỲ THI TUYỂN SINH LỚP 10 Năm học 2018-2019 Mơn thi: TỐN CHUN TIN Thời gian làm bài: 120 phút (không kể thời gian phát đề) Ngày thi: 06/6/2018  Bài I (3 điểm): Rút gọn biểu thức A  29  12  Giải phương trình 10 52  x  2 x  4  3x2  x  21 2  2 x  xy  y  y  Giải hệ phương trình  x  y    Bài II (3 điểm): Trong mặt phẳng Oxy, cho Parabol  P  : y  x đường thẳng  d  : x  y  12  a) Tìm tọa độ giao điểm A B  d   P  b) Tìm tọa độ điểm C nằm  P  cho tam giác ABC vuông C Giả sử x1 , x2 nghiệm phương trình x2  2mx   Xác định m để x14  x2  32 Bài III (1 điểm): Hai máy cày làm việc 12 cày khu đất Nếu máy cày thứ làm 42 10 nghỉ sau máy cày thứ hai làm 22 hai máy cày 25% khu đất Hỏi làm máy cày bao lâu? Bài IV (3 điểm): Cho đường trịn tâm O đường kính AB  2R điểm C nằm đường tròn cho CA  CB Gọi I trung điểm OA Vẽ đường thẳng d vng góc với AB I , cắt tia BC M cắt đoạn AC P ; AM cắt đường tròn  O  điểm thứ hai K a) Chứng minh tứ giác BCPI nội tiếp đường tròn b) Chứng minh điểm B, P, K thẳng hàng c) Các tiếp tuyến A C đường tròn  O  cắt Q Tính diện tích tứ giác QAIM theo R biết BC  R Hết Trang Giải Bài I: A  29  12  5  20  2.2 5.3    5 52 2  3   52   35  2 10  x  2 x  4  3x2  x  21  x   x        x   x     x   x    Điều kiện xác định       3x  x  21  3  x   x     3  x  x      x  2 x     x  4 x  10  x  2 x  4  3x2  6x  21  10  x  2 x  4   x  2 x    t  Đặt t   x   x   , t  Phương trình trở thành 10t  3t   3t  10t     t    x  1   n  + t    x   x     x  x      x  1   n   3  82 x  n  1 + t    x   x     x  x     3  3  82  n x   2 x  xy  y  y  1   2  2  x  y  Từ (1) ta có y  y  x  3   x  Ta xem phương trình bậc hai theo biến y (x tham số) 2    x  3    x   x  x    8x  x  x   3x  1  2 x   3x  x   3x   x  y   x 1 2 + Nếu y  x  Thay vào phương trình (2) ta Suy phương trình có nghiệm y  x2   x     x  x  8x    3x  x   (phương trình vơ nghiệm) + Nếu y   x  Thay vào phương trình (2) ta x2    x  1   x    x   y  1 Vậy tập nghiệm S   2; 1 Bài II: 1 a) Ta có  d  : y  x  Phương trình hồnh độ giao điểm x   y  x  x6   x  4  y  Vậy giao điểm A  6;9  , B  4; 4 b) Trang Cách   Gọi C  c ; c    P  c  6, c  4 điểm cần tìm   1  Ta có AB  125 ; AC   c     c    c  c  12c  117 ; 4  16 2 2 1  BC   c     c    c  c  8c  32 4  16 Tam giác ABC vuông C AB2  AC  BC  125  c  c  12c  117  c  c  8c  32 16 16 1 1  c  c  4c  24   c  c  c  c  4c  8c  12c  24  8 4 1  c3  c    c  c     c    12  c    1    c    c3  c  4c  12   8  c    1  c  c  4c  12  8 c   n    c  4  l  c  l   2 Vậy C  2;1 điểm thỏa đề Cách 2:   Gọi C  c ; c    P  c  6, c  4    13  Ta gọi M trung điểm AB, suy M 1;   2 5 Ta có ABC vng C nên MC  AB  (đường trung tuyến ứng với cạnh huyền nửa cạnh huyền) 2 2  13  125 Suy  c  1   c    2 4 1 1  c  c  2c  12   c  c  c  c  2c  4c  6c  12  16 16 8 1  c3  c    c  c    2c  c     c    16  x   n  1    c    c  c  2c      x   l   16   x  4 l   Vậy điểm C  2;1 điểm thỏa đề Trang Ta có  '  m2  Phương trình có nghiệm x1 , x2   '   m2    m  m  2 b c Theo định lý viet ta có S  x1  x2    2m ; P  x1.x2   a a x14  x2  32   x12  x2   x12 x2  32   S  P   P  32  S  4S P  P  32 2  16m4  16m2  2.42  32  16m4  64m2  32  32  m2  m2     2  m  Kết hợp với điều kiện có nghiệm ta có m  2 m  Bài III: Gọi x (giờ) thời gian máy cày làm xong khu đất y (giờ) thời gian máy cày làm xong khu đất Điều kiện x, y  12 1 Mỗi máy máy làm tương ứng khu đất x y 12 12 1 Do máy cáy 12 khu đất nên ta có phương trình   10 x y 10 Nếu máy làm 42 máy làm 22 làm 25%  khu đất nên ta có 42 22 phương trình   x y 12 12 1 1   x  y  10   x  300   x 300   Suy  1 42 22 y  200        x  y 200 y Vậy máy làm 300 xong khu đất Máy làm 200 xong khu đất Bài IV: Trang M Q C K P A I B O a) Xét tứ giác BCPI có: ACB  900 (góc nội tiếp chắn nửa đường trịn) Và PIB  900 (giả thiết) Suy tứ giác BCPI nội tiếp đường trịn đường kính BP b) Xét tam giác MAB có: MI  AB AC  MB , suy MI , AC đường cao Mà P giao điểm MI , AC Nên P trực tâm tam giác MAB Ta lại có BKA  900 (góc nội tiếp chắn nửa đường tròn) Nên BK  MA  BK đường cao thứ tam giác MAB Do BK qua điểm P hay B, P, K thẳng hàng c) Ta có AQ / / MI (do vng góc với AB) nên QAIM hình thang vng BC  R nên OBC Do ABC  600 Ta có QA, QC tiếp tuyến  O  nên QAC  QCA  ABC  600 (góc nội tiếp góc tạo tiếp tuyến dây cung chắn cung) Do QAC ABC vng C có AC  AB  BC   2R   R  R  QA  R Trang R BI  R 2 3R Xét tam giác MIB vng I có: MI  BI tan ABC  R.tan 600  2  3R  R 3  R  QA  IM  AI   5R Vậy diện tích hình thang vng QAIM là: SQAIM    2 Ta có I trung điểm bán kính OA nên AI  Trang ... 3.4 2018. 2019       2018 2018 .2019      P  1  1       2.3  3.4   2018. 2019  2.3  3.4  2018. 2019   2.3 3.4 2018. 2019 10 4074340  2.3 3.4 2018. 2019 1.4... TẠO BÌNH ĐỊNH Đề thức KỲ THI TUYỂN SINH VÀO LỚP 10 THPT CHUYÊN NĂM HỌC 2018 – 2019 Môn thi: TỐN (Chun Tốn) Ngày thi: 03/ 6/ 2018 Thời gián làm bài: 150 phút (không kể thời gian phát đề) Bài 1: (2,0điểm)... – ĐÀO TẠO TỈNH CÀ MAU KỲ THI TUYỂN SINH LỚP 10 THPT NĂM HỌC 2018- 2019 Môn thi: TOÁN CHUYÊN Thời gian: 150 phút Ngày thi: 03/06 /2018 ĐỀ THI CHÍNH THỨC Câu 1: Rút gọn biểu thức sau a) A  20  45

Ngày đăng: 28/12/2020, 19:04

Xem thêm:

TỪ KHÓA LIÊN QUAN

w